SlideShare a Scribd company logo
1 of 436
AIIMS NOVEMBER – 2015
QUESTIONS & EXPLANATORY ANSWERS (INCLUDING IMAGE BASED
QUESTIONS)
BY:DR MANJUNATH,
MBBS MD (MAULANAAZAD MEDICAL COLLEGE),
ALL INDIAFACULTY
DIRECTOR, DOCTORSACADEMY, DAVANAGERE
ANDSHIVAMOGGA .
REGULAR COURSE/FOUNDATION COURSE,
CONTACT: 9686252725, 9632241911 .WWW.DOCTORSACADEMYDVG.COM
DR MANJUNATH, DOCTORS ACADEMY, DAVANAGERE 1
AIIMS NOV 2015-QUESTIONS & ANSWERS WITH EXPLANATIONS-SOLVED BY
DR: MANJUNATH, MBBS, MD (MAMC NEW DELHI) DIRECTOR
DOCTORS ACADEMY DAVANAGERE & SHIMOGA
(www.doctorsacademydvg.com)
ANATOMY
1. WHICH OF THE
FOLLOWING MUSLE
HELPS IN PROTRUSION
OF MANDIBLE?
A) TEMPORALIS
B)LATERAL PTERYGOID
C) MEDIAL PTERYGOID
D)BUCCINATOR
• ANSWER: B) LATERAL PTERIGOID
DR MANJUNATH, DOCTORS ACADEMY, DAVANAGERE 2
AIIMS MAY 2015-QUESTIONS & ANSWERS WITH EXPLANATIONS-SOLVED BY
DR: MANJUNATH, MBBS, MD (MAMC NEW DELHI) DIRECTOR
DOCTORS ACADEMY DAVANAGERE & SHIMOGA
(www.doctorsacademydvg.com)-
• THE LATERAL PTERYGOID MUSCLE IS ALSO KNOWN AS PTERYGOIDEUS
EXTERNUS OR EXTERNAL PTERYGOID MUSCLE.
•
• GROSS ANATOMY
• THE LATERAL PTERYGOID IS A SHORT, THICK MUSCLE, SOMEWHAT CONICAL
IN FORM, WHICH EXTENDS ALMOST HORIZONTALLY, POSTERIORLY AND
LATERALLY BETWEEN THE INFRATEMPORAL FOSSA AND THE CONDYLE OF
THE MANDIBLE . IT ARISES BY TWO HEADS: AN UPPER (SUPERIOR) AND A
LOWER (INFERIOR).
• THE SUPERIOR PART ARISES FROM THE LOWER PART OF THE LATERAL
SURFACE OF THE GREATER WING OF THE SPHENOID AND FROM THE
INFRATEMPORAL CREST. IT INSERTS IN THE TMJ CAPSULE AND TMJ DISC
.
• THE INFERIOR PART ARISES FROM THE LATERAL SURFACE OF THE LATERAL
PTERYGOID PLATE AND INSERTS INTO A DEPRESSION IN FRONT OF THE
NECK OF THE CONDYLE OF THE MANDIBLE; THE PTERYGOID FOVEA.
DR MANJUNATH, DOCTORS ACADEMY, DAVANAGERE 3
AIIMS MAY 2015-QUESTIONS & ANSWERS WITH EXPLANATIONS-SOLVED BY
DR: MANJUNATH, MBBS, MD (MAMC NEW DELHI) DIRECTOR
DOCTORS ACADEMY DAVANAGERE & SHIMOGA
(www.doctorsacademydvg.com)-
• ACTIONS OF LATERAL PTERIGOID
• THE SUPERIOR PART IS ACTIVE DURING RETRUSION (OPPOSITE OF
PROTRUSION) AND IPSILATERAL JAW MOVEMENT. IT IS ALSO
ESSENTIAL IN PULLING THE CAPSULE AND DISC FORWARD DURING
MOUTH OPENING, THEREBY MAINTAINING NORMAL RELATIONSHIP
BETWEEN THE CONDYLE OF THE MANDIBLE AND THE TMJ DISC.
• THE INFERIOR PART IS RESPONSIBLE FOR OPENING OF THE MOUTH,
PROTRUSION AND CONTRALATERAL JAW MOVEMENT.
• HYPERACTIVITY OF THE LPM MUSCLE HAS BEEN DESCRIBED IN TMJ
INTERNAL DERANGEMENT , ESPECIALLY WITH LONGSTANDING
ANTERIOR DISPLACEMENT OF THE DISC WITHOUT RECAPTURE.
THICKENING OF THE TENDON (INFERIOR PART) CAN GIVE RISE TO THE
"DOUBLE DISC SIGN".
• NERVE SUPPLY: MANDIBULAR NERVE
DR MANJUNATH, DOCTORS ACADEMY, DAVANAGERE 4
AIIMS MAY 2015-QUESTIONS & ANSWERS WITH EXPLANATIONS-SOLVED BY
DR: MANJUNATH, MBBS, MD (MAMC NEW DELHI) DIRECTOR
DOCTORS ACADEMY DAVANAGERE & SHIMOGA
(www.doctorsacademydvg.com)
DR MANJUNATH, DOCTORS ACADEMY, DAVANAGERE
MUSCLES NERVE SUPPLY ACTION SPECIAL REMARKS
MASSETER ANTERIOR DIVISION OF
MANDIBULAR NERVE
ELEVATES THE MANDIBLE
RETRACTS THE MANDIBLE
SIDE MOVEMENT
EFFECTOR OF JAW JERK
IT IS HYPERTROPHIED IN
BRUXISM
TEMPORALIS ANTERIOR DIVISION OF
MANDIBULAR NERVE
ELEVATES MANDIBLE
PROTRACTION & SIDE TO SIDE MOVEMENT.
IT IS A FAN SHAPED MUSCLE
LATERAL PTERYGOID ANTRIOR DIVISION OF
MANDIBULAR NERVE
MOST IMPORTANT MUSCLE FOR OPENING
OF MOUTH ALONG WITH SUPRAHYOID
MUSCLES
LEFT & RIGHT MUSCLES CONTRACT
TOGETHER TO CAUSE PROTRUSION OF JAW
IF ONLY ONE LATERAL PTERYGOID
CONTRACTS, JAW IS PULLED MEDIALLY
TOWARD THE OPPOSITE SIDE.
MEDIAL PTERYGOID BRANCH FROM TRUNK OF
MANDIBULAR NERVE
CONTRACTION OF LATERLA & MEDIAL
PTERIGOID TOGETHER PROVIDE HELP IN
GRINDING FOOD BETWEEN TEETH OF THE
SAME SIDE.
NERVE TO MEIDAL PTERIGOID
SUPPLIES TENSOR VELI
PALATINI & TENSOR TYMPANI
THROUGH OTIC GANGLION
5
AIIMS MAY 2015-QUESTIONS & ANSWERS WITH EXPLANATIONS-SOLVED BY
DR: MANJUNATH, MBBS, MD (MAMC NEW DELHI) DIRECTOR
DOCTORS ACADEMY DAVANAGERE & SHIMOGA
(www.doctorsacademydvg.com)
2. AXILLARY NERVE IS NOT
INJURED IN WHICH OF THE
FOLLOWING CONDITIONS?
A. FRACTURE OF SURGICAL
NECK HUMERUS
B. INTRAMUSCULAR INJECTION
C. IMPROPER USE OF CRUTCHES
D. SHOULDER DISLOCATION
• ANS: C. IMPROPER USE OF CRUTCHES
• REF: SNELL’S 9ED/354
DR MANJUNATH, DOCTORS ACADEMY, DAVANAGERE 6
AIIMS MAY 2015-QUESTIONS & ANSWERS WITH EXPLANATIONS-SOLVED BY
DR: MANJUNATH, MBBS, MD (MAMC NEW DELHI) DIRECTOR
DOCTORS ACADEMY DAVANAGERE & SHIMOGA
(www.doctorsacademydvg.com): AXILLARY NERVE
• THE AXILLARY NERVE IS ONE OF FIVE TERMINAL BRANCHES
OF THE BRACHIAL PLEXUS, SUPPLYING MOTOR AND
SENSORY BRANCHES TO THE SHOULDER.
•
• SUMMARY
• ORIGIN: POSTERIOR CORD OF THE BRACHIAL PLEXUS
• COURSE: PASSES OUT OF AXILLA THROUGH THE
QUADRANGULAR SPACE TO THE UPPER ARM
• MAJOR BRANCHES: SUPERIOR LATERAL CUTANEOUS NERVE
OF THE ARM
• MOTOR SUPPLY: DELTOID AND TERES MINOR MUSCLES
• SENSORY SUPPLY: SKIN OVERLYING DELTOID MUSCLE
DR MANJUNATH, DOCTORS ACADEMY,
DAVANAGERE
7
AIIMS MAY 2015-QUESTIONS & ANSWERS WITH EXPLANATIONS-SOLVED BY
DR: MANJUNATH, MBBS, MD (MAMC NEW DELHI) DIRECTOR
DOCTORS ACADEMY DAVANAGERE & SHIMOGA
(www.doctorsacademydvg.com)
DR MANJUNATH, DOCTORS ACADEMY,
DAVANAGERE
8
AIIMS MAY 2015-QUESTIONS & ANSWERS WITH EXPLANATIONS-SOLVED BY
DR: MANJUNATH, MBBS, MD (MAMC NEW DELHI) DIRECTOR
DOCTORS ACADEMY DAVANAGERE & SHIMOGA
(www.doctorsacademydvg.com)
DR MANJUNATH, DOCTORS ACADEMY, DAVANAGERE
HIGH YIELDING INFORMATION:
NERVE DAMAGED EFFECT OF LESION
UPPER TRUNK OF BRACHIAL PLEXUS (ERB’S PALSY) POLICEMAN’S TIP HAND OR PORTER’S TIP HAND
LOWER TRUNK OF BRACHIAL PEXUS (KLUMPKE’S
PALSY)
CLAW HAND, HORNER’S SYNDROME
LONG THORACIC NERVE WINGED SCAPULA
MEDIAN POINTING INDEX, APE THUMB
UNLAR CLAW HAND (ULNAR CLAW HAND)
MEIDAN + ULNAR CLAW HAND (COMPLETE CLAW HAND)
RADIAL WRIST DROP
9
AIIMS MAY 2015-QUESTIONS & ANSWERS WITH EXPLANATIONS-SOLVED BY
DR: MANJUNATH, MBBS, MD (MAMC NEW DELHI) DIRECTOR
DOCTORS ACADEMY DAVANAGERE & SHIMOGA
(www.doctorsacademydvg.com)
3. BUCCINATOR IS PIERCED
BY ALL OF THE
FOLLOWING EXCEPT ?
A. BUCCAL BRANCH OF
MANDIBULAR
B. LABIAL BRANCH OF FACIAL
ARTERY
C. PAROTID DUCT
D. MUCUS GLAND OF MOLARS
ANS: B. LABIAL BRANCH OF FACIAL ARTERY
(Ref: BDC vol3, 6ed/p63)
DR MANJUNATH, DOCTORS ACADEMY, DAVANAGERE 10
AIIMS MAY 2015-QUESTIONS & ANSWERS WITH EXPLANATIONS-SOLVED BY
DR: MANJUNATH, MBBS, MD (MAMC NEW DELHI) DIRECTOR
DOCTORS ACADEMY DAVANAGERE & SHIMOGA
(www.doctorsacademydvg.com)
DR MANJUNATH, DOCTORS ACADEMY, DAVANAGERE
STRUCTURES PIERCING BUCCINATOR ARE:
PAROTID DUCT:
PAROTID DUCT
BUCCAL BRANCH OF MANDIBULAR NERVE
4-5 MOLAR MUCUS GLANDS LYING ON BUCCOPHARYNGEAL FASCIA AROUND PAROTID
DUCT
EMERGES AT ANTERIOR BORDER OF GLAND
RUNS OVER MASSETER
PIERCES BUCCINATOR TO ENTER MOUTH OPPOSITE UPPER SECOND MOLAR
11
AIIMS MAY 2015-QUESTIONS & ANSWERS WITH EXPLANATIONS-SOLVED BY
DR: MANJUNATH, MBBS, MD (MAMC NEW DELHI) DIRECTOR
DOCTORS ACADEMY DAVANAGERE & SHIMOGA
(www.doctorsacademydvg.com)
4. WHICH OF THE
FOLLOWING ARTERY
DOES NOT SUPPLY
RECTUM?
A. SUPERIOR RECTAL ARTERY
B. INFERIOR RECTAL ARTERY
C. MIDDLE RECTAL ARTERY
D. MIDDLE COLIC ARTERY
ANS: D. MIDDLE COLIC ARTERY
Ref: BDC vol2, 6th ed/267-269
DR MANJUNATH, DOCTORS ACADEMY, DAVANAGERE 12
AIIMS MAY 2015-QUESTIONS & ANSWERS WITH EXPLANATIONS-SOLVED BY
DR: MANJUNATH, MBBS, MD (MAMC NEW DELHI) DIRECTOR
DOCTORS ACADEMY DAVANAGERE & SHIMOGA
(www.doctorsacademydvg.com)
DR MANJUNATH, DOCTORS ACADEMY, DAVANAGERE
BLOOD SUPPLY OF RECTUM:
NAME OF ARTERY ARISES FROM SUPPLIES
SUPERIOR-RECTAL A INFERIOR MESENTERIC ARTRY MAIN SUPPLY FOR UPPER 2/3 OF
THE RECTUM
MIDDLE RECTAL A ANT DIV OF INTERNAL ILIAC
ARTERY
MIDDLE THRID OF RECTUM
INFERIOR RECTAL A AORTA DISTAL THIRD OF RECTUM
MEDIAN SACRAL A TERMINAL MIDLINE BRANCH OF
THE AORTA
POSTERIOR WALL OF ANORECTAL
JUNCTION ON SACRORECTAL
FASCIA
13
AIIMS MAY 2015-QUESTIONS & ANSWERS WITH EXPLANATIONS-SOLVED BY
DR: MANJUNATH, MBBS, MD (MAMC NEW DELHI) DIRECTOR
DOCTORS ACADEMY DAVANAGERE & SHIMOGA
(www.doctorsacademydvg.com)
5. IDENTIFY VAGUS NERVE IN THE GIVEN DIAGRAM OF
TRANSVERSE SECTION OF THORAX?
A. VAGUS NERVE
B. AORTA
C. THORACIC DUCT
D. IVC
Ans: A VAGUS NERVE
Ref: Netter’s Anatomy Atlas 5th/27
DR MANJUNATH, DOCTORS ACADEMY,
DAVANAGERE
14
AIIMS MAY 2015-QUESTIONS & ANSWERS WITH EXPLANATIONS-SOLVED BY
DR: MANJUNATH, MBBS, MD (MAMC NEW DELHI) DIRECTOR
DOCTORS ACADEMY DAVANAGERE & SHIMOGA
(www.doctorsacademydvg.com): HIGH YIELDING POINTS
DR MANJUNATH, DOCTORS ACADEMY, DAVANAGERE
LONGEST INTRACRANIAL COURSE → TROCHLEAR NERVE (ALL INDIA REPEAT)
LONGEST COURSE OVERALL AND MOST WIDELY DISTRIBUTED → VAGUS NERVE
SMALLEST (THINNEST) CRANIAL NERVE → TROCHLEAR NERVE
LARGEST (THICKEST) CRANIAL NERVE → TRIGEMINAL NERVE
CRANIAL NERVES CARRYING PARASYMPATHETIC FIBERS → 3, 7, 9, 10
THE ONLY CRANIAL NERVE DECUSSATING COMPLETELY BEFORE EMERGING → TROCHLEAR
NERVE
COMMONEST CRANIAL NERVE AFFECTED IN SPINAL ANESTHESIA → ABDUCENT NERVE
CRANIAL NERVE MOST COMMONLY INVOLVED IN INTRACRANIAL IN INTRACRANIAL
ANEURYSM → OCULOMOTOR NERVE
CRANIAL NERVE MOST COMMONLY INVOLVED IN BASED SKULL FRACTURE → FACIAL
NERVE
CRANIAL NERVE MOST COMMONLY INVOLVED IN RAISED INTRACRANIAL TESNION →
ABDUCENT NERVE
15
6. WHICH OF THE FOLLOWING IS NOT INVOLVED IN
FORMATION OF THORACIC DIAPHRAGM?
A. CENTRAL TENDON
B. BODY WALL
C. PERICARDIOPERITONEAL FOLD
D. SOMATIC PLEURIPERITONEUM
ANS: C. PERICARDIOPERITONEAL FOLD
REF: BDC 6TH ED, VOL1/193-194
DR MANJUNATH, DOCTORS ACADEMY,
DAVANAGERE
AIIMS MAY 2015-QUESTIONS & ANSWERS WITH EXPLANATIONS-SOLVED BY
DR: MANJUNATH, MBBS, MD (MAMC NEW DELHI) DIRECTOR
DOCTORS ACADEMY DAVANAGERE & SHIMOGA
(www.doctorsacademydvg.com)
16
AIIMS MAY 2015-QUESTIONS & ANSWERS WITH EXPLANATIONS-SOLVED BY
DR: MANJUNATH, MBBS, MD (MAMC NEW DELHI) DIRECTOR
DOCTORS ACADEMY DAVANAGERE & SHIMOGA
(www.doctorsacademydvg.com): EXTRA EDGE
DR MANJUNATH, DOCTORS ACADEMY, DAVANAGERE
EMBRYONIC STRUCTURE PART OF DIAPHRAGM
SEPTUM TRANSVERSUM STERNAL PART & CENTRAL TENDINOUS
REGION
PLEUROPERITONEAL MEMBRANES DORSAL PAIRED PORTION
DORSAL MESENTARY OF ESOPHAGUS UNPAIRED CRURAL PART
LATERAL BODY WALLS PERIPHERAL COSTAL PORTION
17
AIIMS MAY 2015-QUESTIONS & ANSWERS WITH EXPLANATIONS-SOLVED BY
DR: MANJUNATH, MBBS, MD (MAMC NEW DELHI) DIRECTOR
DOCTORS ACADEMY DAVANAGERE & SHIMOGA
(www.doctorsacademydvg.com)
DR MANJUNATH, DOCTORS ACADEMY, DAVANAGERE
MAJOR OPENINGS IN DIAPHRAGM: SIM: SAVING INTO MEMEORY
OPENING PART OF DIAPHRAGM SHAPE STRUCTURES
PASSING
VENAE CAVAL T8
CENTRAL TENDON QUADRILATE
RAL
1.INFERIOR VENA CAVA
2. BRANCHES OF RIGHT
PHRENIC NERVE
OESOPHAGEAL T8
MUSCULAR PORTION
DERIVED FROM RIGHT CRUS
ELLIPTICAL 1.OESOPHAGUS
2.OESOPHAGUS
BRANCH OF
LEFT GASTRIC VESSELS
3. VAGUS/GASTRIC
NERVES
AORTIC
(OSSEOAPANEUROTIC)
T12
BETWEEN RIGHT AND
LEFT CRUS (POSTERIOR
TO DIAPHRAGM)
ROUND 1. AORTA
2. THORACIC DUCT
3. AZYGOUS VEIN
18
7. 8 YEAR OLD BOY PRESENTED TO OPD
WITH HYPEREXTENSION OF METACARPO
PHALANGEAL JOINT AND FLEXION OF
DISTAL PHALAGEAL JOINT OF LITTLE AND
RING FINGER. WHICH NERVE IS
INVOLVED?
A. ULNAR NERVE
B. RADIAL NERVE
C. ANTERIOR INTEROSSEOUS
NERVE
D. POSTERIOR INTEROSSEOUS
NERVE
ANS- A. ULNAR NERVE (DIAGNOSIS-ULNAR CLAW HAND)
Ref: BDC 5th/e vol 1, p110-111
DR MANJUNATH, DOCTORS ACADEMY, DAVANAGERE
AIIMS MAY 2015-QUESTIONS & ANSWERS WITH EXPLANATIONS-SOLVED BY
DR: MANJUNATH, MBBS, MD (MAMC NEW DELHI) DIRECTOR
DOCTORS ACADEMY DAVANAGERE & SHIMOGA
(www.doctorsacademydvg.com)
19
AIIMS MAY 2015-QUESTIONS & ANSWERS WITH EXPLANATIONS-SOLVED BY
DR: MANJUNATH, MBBS, MD (MAMC NEW DELHI) DIRECTOR
DOCTORS ACADEMY DAVANAGERE & SHIMOGA
(www.doctorsacademydvg.com): ULNAR NERVE-EVERY THING YOU SHOULD KNOW
DR MANJUNATH, DOCTORS ACADEMY, DAVANAGERE
THE ULNAR NERVE IS ONE OF THE TERMINAL BRANCHES OF THE BRACHIAL
PLEXUS AND HAS MOTOR AND SENSORY SUPPLY TO THE FOREARM AND HAND.
• GROSS ANATOMY
• ORIGIN
• THE ULNAR NERVE ORIGINATES AS A TERMINAL BRANCH OF THE MEDIAL CORD
OF THE BRACHIAL PLEXUS WITH NERVE ROOT FIBRES FROM C8-T1.
• COURSE
• ARM
• IN THE ARM, THE ULNAR NERVE RUNS MEDIAL TO THE AXILLARY ARTERY AND
SUBSEQUENTLY THE BRACHIAL ARTERY ON THE CORACOBRACHIALIS MUSCLE IN
THE ANTERIOR COMPARTMENT. THE NERVE PASSES TO THE POSTERIOR
COMPARTMENT THROUGH THE MEDIAL INTERMUSCULAR SEPTUM DISTALLY
RUNNING WITH THE SUPERIOR ULNAR COLLATERAL ARTERY. FURTHER ON, IT
RUNS BETWEEN MEDIAL HEAD OF TRICEPS BRACHII MUSCLE AND THE MEDIAL
INTERMUSCULAR SEPTUM TO PASS POSTERIOR TO THE MEDIAL HUMERAL
EPICONDYLE IN THE SUPERFICIAL CONDYLAR GROOVE (CUBITAL TUNNEL).
20
AIIMS MAY 2015-QUESTIONS & ANSWERS WITH EXPLANATIONS-SOLVED BY
DR: MANJUNATH, MBBS, MD (MAMC NEW DELHI) DIRECTOR
DOCTORS ACADEMY DAVANAGERE & SHIMOGA
(www.doctorsacademydvg.com): ULNAR NERVE
DR MANJUNATH, DOCTORS ACADEMY, DAVANAGERE
• FOREARM
• THE ULNAR NERVE ENTERS THE FOREARM FROM THE ARM
VIA THE TWO HEADS OF FLEXOR CARPI ULNARIS (FCU)
MUSCLE. IT SUBSEQUENTLY LIES SUPERFICIAL TO FLEXOR
DIGITORUM PROFUNDUS AND DEEP TO FCU AND MEDIAL
TO THE ULNAR ARTERY. AT THE WRIST, THE ULNAR NERVE
RUNS LATERAL TO THE TENDON OF FCU.
• HAND
• THE ULNAR NERVE ENTERS THE HAND SUPERFICIAL TO
THE FLEXOR RETINACULUM AND INSIDE GUYON'S CANAL.
THEN IT DIVIDES INTO ITS TERMINAL BRANCHES AT THE
LEVEL OF THE PISIFORM BONE.
• PRIOR TO PASSING THE FLEXOR RETINACULUM AT THE
WRIST, THE ULNAR NERVE GIVES OFF THE DORSAL
CUTANEOUS BRANCH.
21
AIIMS MAY 2015-QUESTIONS & ANSWERS WITH EXPLANATIONS-SOLVED BY
DR: MANJUNATH, MBBS, MD (MAMC NEW DELHI) DIRECTOR
DOCTORS ACADEMY DAVANAGERE & SHIMOGA
(www.doctorsacademydvg.com)
DR MANJUNATH, DOCTORS ACADEMY, DAVANAGERE
CUBITAL TUNNEL SYNDROME IS A TYPE OF ULNAR NERVE COMPRESSION NEUROPATHY
WHICH CAN OCCUR FROM A PATHOLOGICAL COMPRESSION OF THE ULNAR NERVE AT
THE CUBITAL TUNNEL.
• EPIDEMIOLOGY
• IT IS THE SECOND MOST COMMON PERIPHERAL NEUROPATHY OF THE UPPER
EXTREMITY.
• CLINICAL PRESENTATION
• ULNAR NERVE COMPRESSION CAN RESULT IN ALTERED SENSATION IN THE LITTLE AND
RING FINGERS. IN MANY PATIENTS, SENSORY LOSS IS OFTEN THE FIRST SYMPTOM TO
BE REPORTED. AS THE CONDITION PROGRESSES, THEY MAY BE CLUMSINESS IN THE
HAND, AS THE ULNAR NERVE IS THE PRINCIPAL MOTOR SUPPLY TO THE INTRINSIC
MUSCLES OF THE HAND. IN WELL‐ESTABLISHED CASES, THERE MAY BE MARKED
WASTING OF THE SMALL MUSCLES OF THE HAND AND THE ULNAR‐SIDED MUSCLES OF
THE FOREARM.
22
AIIMS MAY 2015-QUESTIONS & ANSWERS WITH EXPLANATIONS-SOLVED BY
DR: MANJUNATH, MBBS, MD (MAMC NEW DELHI) DIRECTOR
DOCTORS ACADEMY DAVANAGERE & SHIMOGA
(www.doctorsacademydvg.com)
DR MANJUNATH, DOCTORS ACADEMY, DAVANAGERE 23
AIIMS MAY 2015-QUESTIONS & ANSWERS WITH EXPLANATIONS-SOLVED BY
DR: MANJUNATH, MBBS, MD (MAMC NEW DELHI) DIRECTOR
DOCTORS ACADEMY DAVANAGERE & SHIMOGA
(www.doctorsacademydvg.com): EXTRA EDGE
DR MANJUNATH, DOCTORS ACADEMY, DAVANAGERE 24
AIIMS MAY 2015-QUESTIONS & ANSWERS WITH EXPLANATIONS-SOLVED BY
DR: MANJUNATH, MBBS, MD (MAMC NEW DELHI) DIRECTOR
DOCTORS ACADEMY DAVANAGERE & SHIMOGA
(www.doctorsacademydvg.com)
DR MANJUNATH, DOCTORS ACADEMY, DAVANAGERE 25
8. INCORRECT REGARDING LOCATION OF OTIC
GANGLION IS?
A. ANTERIOR TO MIDDLE MENINGEAL ARTERY
B. LATERAL TO MANDIBULAR NERVE
C. INFERIOR TO FORAMEN OVALE
D. LATERAL TO TENSOR VELI PALATINE
• ANS: B. LATERAL TO MANDIBULAR NERVE ( OTIC GANGLION IS MEDIAL TO
MANDIBULAR NERVE)
• Ref: Gray’s 3rd/988-989
DR MANJUNATH, DOCTORS ACADEMY, DAVANAGERE
AIIMS MAY 2015-QUESTIONS & ANSWERS WITH EXPLANATIONS-SOLVED BY
DR: MANJUNATH, MBBS, MD (MAMC NEW DELHI) DIRECTOR
DOCTORS ACADEMY DAVANAGERE & SHIMOGA
(www.doctorsacademydvg.com)
26
AIIMS MAY 2015-QUESTIONS & ANSWERS WITH EXPLANATIONS-SOLVED BY
DR: MANJUNATH, MBBS, MD (MAMC NEW DELHI) DIRECTOR
DOCTORS ACADEMY DAVANAGERE & SHIMOGA
(www.doctorsacademydvg.com)
DR MANJUNATH, DOCTORS ACADEMY, DAVANAGERE
OTIC GANGLION :
IT IS A SMALL, OVAL, REDDISH-GREY GANGLION SITUATED JUST BELOW
FORAMEN OVALE.
IT IS A PERIPHERAL PARASYMPATHETIC GANGLION
RELATED TOPOGRAPHICALLY TO MANDIBULAR NERV, BUT FUNCTIONALLY
WITH GLSSOPHARYNGEAL NERVE.
NEAR ITS JUNCTION WITH TRIGEMINAL MOTOR ROOT, MANDIBULAR NERVE
LIES LATERAL TO IT.
TENSOR VELI PALATINI LIES MEDIALLY, SEPARATING THE GANGLION FROM
CARTILAGINOUS PART OF THE PHARYNGOTYMPANIC TUBE
MIDDLE MENINGEAL ARTERY IS POSTERIOR TO THE GANGLION.
OTIC GANGLION USUALLY SURROUNDS THE ORIGIN OF THE NERVE TO MEDIAL
PTERYGOID.
27
AIIMS MAY 2015-QUESTIONS & ANSWERS WITH EXPLANATIONS-SOLVED BY
DR: MANJUNATH, MBBS, MD (MAMC NEW DELHI) DIRECTOR
DOCTORS ACADEMY DAVANAGERE & SHIMOGA
(www.doctorsacademydvg.com)
DR MANJUNATH, DOCTORS ACADEMY, DAVANAGERE 28
9. TRUE STATEMENT ABOUT OSTEOBLASTS ARE
ALL EXCEPT?
A. DERIVED FROM OSTEOPROGENITOR
B. EXPRESS NEUROPEPTIDE RECEPTORS
C. HAVE SMOOTH PLASMA MEMBRANE OUTLINE
D. REGULATED BY BONE MORPHOGENIC FACTOR
ANS: C. HAVE SMOOTH PLASMA MEMBRANE OUTLINE
Ref: I.B Singh Histology 7/ed/pg 89-90
DR MANJUNATH, DOCTORS ACADEMY, DAVANAGERE
AIIMS MAY 2015-QUESTIONS & ANSWERS WITH EXPLANATIONS-SOLVED BY
DR: MANJUNATH, MBBS, MD (MAMC NEW DELHI) DIRECTOR
DOCTORS ACADEMY DAVANAGERE & SHIMOGA
(www.doctorsacademydvg.com)
29
10. ALL OF THE
FOLLOWING
ARTICULATED
ARTICULATE WITH
MAXILLA EXCEPT?
A. LACRIMAL
B. NASAL
C. ZYGOMATIC
D. SPHENOID
ANS: D. SPHENOID (SPHENOID HAS NO
ARTICULATION)
Ref: Gray’s Anantomy 3rd e/p780-786
DR MANJUNATH, DOCTORS ACADEMY,
DAVANAGERE
AIIMS MAY 2015-QUESTIONS & ANSWERS WITH EXPLANATIONS-SOLVED BY
DR: MANJUNATH, MBBS, MD (MAMC NEW DELHI) DIRECTOR
DOCTORS ACADEMY DAVANAGERE & SHIMOGA
(www.doctorsacademydvg.com)
30
• BIOCHEMISTRY
12. FOR THE TRANSCRIPTION OF SELENO-CYSTEINE,
CODON IS ?
A. UGA
B. UAA
C. UAG
D. GUA
ANS: A. UGA (SELENOCYSTEINE IS 21 ST AMINOACID HELPS IN DELAYING AGEING AND CANCER AND
HEART DISEASE)
REF: HARPER 29/e p270
DR MANJUNATH, DOCTORS ACADEMY, DAVANAGERE
AIIMS NOVEMBER 2015-QUESTIONS & ANSWERS WITH EXPLANATIONS-SOLVED BY
DR: MANJUNATH, MBBS, MD (MAMC NEW DELHI) DIRECTOR
DOCTORS ACADEMY DAVANAGERE & SHIMOGA
(www.doctorsacademydvg.com)
31
AIIMS MAY 2015-QUESTIONS & ANSWERS WITH EXPLANATIONS-SOLVED BY
DR: MANJUNATH, MBBS, MD (MAMC NEW DELHI) DIRECTOR
DOCTORS ACADEMY DAVANAGERE & SHIMOGA
(www.doctorsacademydvg.com)
DR MANJUNATH, DOCTORS ACADEMY, DAVANAGERE 32
DR MANJUNATH, DOCTORS ACADEMY, DAVANAGERE
WHAT IS IT? IT IS AN L-AMINO ACID FOUND IN SOME PEROXIDASES & REDUCTASES WHERE IT PARTICIPATES
IN CATALYSIS OF ELECTRON TRANSFER REACTIONS.
STRUCTURE AS ITS NAME IMPLIES, A SELENIUM ATOM REPLACES THE SULFUR OF ITS STRUCTUAL ANALOG,
CYSTEINE.
SINCE SELENOCYSTEINE IS INSERTED INTO POLYPEPTIDES DURING TRANSLATION, IT IS
COMMONLY REFERRED TO AS “21ST AMINO ACID.”
PK Pk3 OF SELENOCYSTEINE, 5.2, IS 3 UNITS LOWER THAN THAT OF CYSTEINE
UNIQUE
FEATURE
UNLIKE THE OTHER 20 GENETICALLY ENCODED AMINO ACIDS, SELENOCYSTEINE IS NOT
SPECIFIED BY A SIMPLE THREE-LETTER CODON
CLINICAL
SIGNIFICANCE:
SELENOCYSTEINE IS PRESENT AT THE ACTIVE SITE OF SEVERAL HUMAN ENZYMES THAT CATALYZE
REDOX REACTIONS.
EXAMPLES: THIOREDOXIN REDUCTASE, GLUTATHIONE PEROXIDASE & DEIODINASE THAT
CONVERTS THYROXINE TO THIIODOTHYRONINE.
SIGNIFICANTLY , REPLACEMENTOF SELENOCYSTEINE BY CYSTEINE CAN SIGNIFICANTLY DECREASE
CATALYTIC ACTIVITY.
IMPAIRMENTS IN HUMAN SELENOPROTEINS HAVE BEEN IMPLICATED IN TUMORIGENESIS,
ATHEROSCLEROSIS & SELENIUM DEFICIENCY CARDIOMYOPATHY (KESHAN DISEASE).
AIIMS NOV 2015-QUESTIONS & ANSWERS WITH EXPLANATIONS-SOLVED BY
DR: MANJUNATH, MBBS, MD (MAMC NEW DELHI) DIRECTOR
DOCTORS ACADEMY DAVANAGERE & SHIMOGA
(www.doctorsacademydvg.com): SELENOCYSTEINE
33
13. WHICH ONE OF THE
REACTION DOES NOT REQUIRE
BIOTIN?
A. ACETYL-COA TO MALONY-COA
B. PYRUVATE TO OXALOACETATE
C. GLUTAMINE TO GAMMA
CARBOXY GLUTAMATE
D. PROPIONYL COA TO METHYL
MALONYL COA
ANS- C. GLUTAMINE TO GAMMA CARBOXY
GLUTAMATE
Ref: Harper’s 26/e p153-156
DR MANJUNATH, DOCTORS ACADEMY, DAVANAGERE
AIIMS MAY 2015-QUESTIONS & ANSWERS WITH EXPLANATIONS-SOLVED BY
DR: MANJUNATH, MBBS, MD (MAMC NEW DELHI) DIRECTOR
DOCTORS ACADEMY DAVANAGERE & SHIMOGA
(www.doctorsacademydvg.com)
BIOTIN DEFICIENCY
34
DR MANJUNATH, DOCTORS ACADEMY, DAVANAGERE
AIIMS MAY 2015-QUESTIONS & ANSWERS WITH EXPLANATIONS-SOLVED BY
DR: MANJUNATH, MBBS, MD (MAMC NEW DELHI) DIRECTOR
DOCTORS ACADEMY DAVANAGERE & SHIMOGA
(www.doctorsacademydvg.com)
REACTIONS REQUIRING BIOTIN AS CO-ENZYME:
BIOTIN ENZYME BIOCHEMICAL FUNCTION
ACETYL-COA CARBOXYLASE LIPID SYNTHESIS FROM ACETATE
METHYLMALONYL-COA
CARBOXYLTRANSFERASE
PROPINONIC ACID SYNTHESIS BY
RUMEN BACTERIA
3-METHYLCROTONYL-COA CARBOXYLASE LEUCINE CATABOLISM
PROPINYL-COA CARBOXYLASE CONVERSION OF AMINO ACIDS AND
PROPIONATE TO GLUCOSE IN THE LIVER
PYRUVATE CARBOXYLASE LIVER GLUCONEOGENESIS
35
AIIMS NOV 2015-QUESTIONS & ANSWERS WITH EXPLANATIONS-SOLVED BY
DR: MANJUNATH, MBBS, MD (MAMC NEW DELHI) DIRECTOR
DOCTORS ACADEMY DAVANAGERE & SHIMOGA
(www.doctorsacademydvg.com)
DR MANJUNATH, DOCTORS ACADEMY, DAVANAGERE 36
AIIMS MAY 2015-QUESTIONS & ANSWERS WITH EXPLANATIONS-SOLVED BY
DR: MANJUNATH, MBBS, MD (MAMC NEW DELHI) DIRECTOR
DOCTORS ACADEMY DAVANAGERE & SHIMOGA
(www.doctorsacademydvg.com)
DR MANJUNATH, DOCTORS ACADEMY, DAVANAGERE 37
AIIMS MAY 2015-QUESTIONS & ANSWERS WITH EXPLANATIONS-SOLVED BY
DR: MANJUNATH, MBBS, MD (MAMC NEW DELHI) DIRECTOR
DOCTORS ACADEMY DAVANAGERE & SHIMOGA
(www.doctorsacademydvg.com)
DR MANJUNATH, DOCTORS ACADEMY, DAVANAGERE 38
AIIMS NOVEMBER 2015-QUESTIONS & ANSWERS WITH EXPLANATIONS-SOLVED BY
DR: MANJUNATH, MBBS, MD (MAMC NEW DELHI) DIRECTOR
DOCTORS ACADEMY DAVANAGERE & SHIMOGA
(www.doctorsacademydvg.com): BIOTIN FOOD SOURCES
DR MANJUNATH, DOCTORS ACADEMY, DAVANAGERE 39
14. HEME IS NOT
SYNTHESIZED IN?
A. RBC
B. OSTEOCYTE
C. LIVER PARENCHYMA
D. ERYTHROPOETIC PRECURSOR
CELLS IN BONE MARROW
ANS: A. RBC
Ref: Lippincott’s Illustrated Reviews:
Biochemistry 4/e p21
DR MANJUNATH, DOCTORS ACADEMY, DAVANAGERE
AIIMS MAY 2015-QUESTIONS & ANSWERS WITH EXPLANATIONS-SOLVED BY
DR: MANJUNATH, MBBS, MD (MAMC NEW DELHI) DIRECTOR
DOCTORS ACADEMY DAVANAGERE & SHIMOGA
(www.doctorsacademydvg.com)
40
AIIMS NOV 2015-QUESTIONS & ANSWERS WITH EXPLANATIONS-SOLVED BY
DR: MANJUNATH, MBBS, MD (MAMC NEW DELHI) DIRECTOR
DOCTORS ACADEMY DAVANAGERE & SHIMOGA
(www.doctorsacademydvg.com)
15. GLYCOGEN PHOSPHORYLASE IS REGULATED BY ALL
EXCEPT?
A. PROTEIN KINASE
B. CALMODULIN
C. C-AMP
D. GLYCOGENIN
ANS- D. GLYCOGENIN
Ref: Harper’s 26/e p145-147
DR MANJUNATH, DOCTORS ACADEMY, DAVANAGERE 41
AIIMS MAY 2015-QUESTIONS & ANSWERS WITH EXPLANATIONS-SOLVED BY
DR: MANJUNATH, MBBS, MD (MAMC NEW DELHI) DIRECTOR
DOCTORS ACADEMY DAVANAGERE & SHIMOGA
(www.doctorsacademydvg.com)
EXPLANATION:
CYCLIC AMP INTEGRATES THE REGULATION OF GLYCOGENOLYSIS & GLCOGENESIS
THE PRINCIPAL ENZYMES CONTROLLING GLYCOGEN METABOLISM-GLYCOGEN
PHOSPHORYLASE AND GLYCOGEN SYNTHASE-ARE REGULATED BY ALLOSTERIC MECHANISMS
AND COVALENT MODIFICATION BY REVERSIBLE PHOSPHORYLATION AND
DEPHOSPHOSPHORYLATION IS INCREASED IN RECPONSE TO:
 CYCLIC AMP (cAMP) FORMED FROM ATP BY ADENYLYL CYCLASE AT THE INNER SURFACE
OF CELL MEMBRANES IN RESPONSE TO HORMONES SUCH AS EPINEPHRINE,
NOREPINEPHRINE, AND GLUCAGEN.
 cAMP IS HYPDRLYZED BY PHOSPHODIESTERASE, SO TERMINATING HORMONE ACTION; IN
LIVER INSULINE INCREASES THE ACTIVITY OF PHOSPHODIESTERASE.
DR MANJUNATH, DOCTORS ACADEMY, DAVANAGERE 42
AIIMS NOV 2015-QUESTIONS & ANSWERS WITH EXPLANATIONS-SOLVED BY
DR: MANJUNATH, MBBS, MD (MAMC NEW DELHI) DIRECTOR
DOCTORS ACADEMY DAVANAGERE & SHIMOGA
(www.doctorsacademydvg.com)
16. IN A PATIENT WITH LEAD TOXICITY, WHICH HEME
PATHWAY INTERMEDIATE WILL INCREASE IN URINE?
A. ALA
B. PORPHOBILINOGEN
C. UROPORPHOBILINOGEN
D. COPROPORPHYRIN
ANS: A. ALA
(Ref: Harper 29/e p309-311)
DR MANJUNATH, DOCTORS ACADEMY, DAVANAGERE
CHILD WITH LEAD POISONING
43
AIIMS NOVEMBER 2015-QUESTIONS & ANSWERS WITH EXPLANATIONS-SOLVED BY
DR: MANJUNATH, MBBS, MD (MAMC NEW DELHI) DIRECTOR
DOCTORS ACADEMY DAVANAGERE & SHIMOGA
(www.doctorsacademydvg.com)
EXPLANATION:
HEME SYNTHESIS:
STARTS WITH SUCCINYL-CoA, DERIVED FROM THE CITRIC ACID CYCLE IN MITOCHONDRIA,
AND THE AMINO ACID GLYCINE.
PYRIDOXAL PHOSPHATE IS ALSO NECESSARY IN THIS REACTION TO “ACTIVE” GLYCIN.
THE PRODUCT OF THE CONDENSATION REACTIOB BETWEEN SUCCINYL-CoA AND GLYCINE IS
α-AMINO-β-KETOADIPIC ACID, WHICH IS RAPIDLY DECARBOXYLATED TO FORM α
AMINOLEVULINATE (ALA).
DR MANJUNATH, DOCTORS ACADEMY, DAVANAGERE
• THIS REACTION REQUENCE IS CATALYZED BY ALA SYNTHASE, THE RATE-CONTROLLING ENZYME IN
PORPHYRIN BIOSYNTHESIS IN MAMMALIN LIVER. SYNTHESIS OF ALA OCCURS IN MITOCHONDRIA.
•IN THE CYTOSOL, TWO MOLECULES OF ALA ARE CONDENSED BY THE ENZYME ALA DEHYDRATASE
TO FORM TWO MOLECULES OF WATER AND ONE OF PORPHOBILINOGEN (PBG).
•ALA DEHYDRATASE IS A ZINC-CONTAINING ENZYME AND IS SENSITIVE TO INHIBITION BY LEAD, AS
CAN OCCUR IN KEAD POISONING.
•IT IS FOR THIS REASON THAT ALA GETS INCREASED IN LEAD POISONING.
44
AIIMS NOVEMBER 2015-QUESTIONS & ANSWERS WITH EXPLANATIONS-SOLVED BY
DR: MANJUNATH, MBBS, MD (MAMC NEW DELHI) DIRECTOR
DOCTORS ACADEMY DAVANAGERE & SHIMOGA
(www.doctorsacademydvg.com)
17. GLUCOSE 6 PHOSPHATE PRODUCED IN CYTOPLASM OF
HEPATOCYTE DOES NOT GET ACTED UPON BY GLUCOSE 6
PHOSPHATASE ENZYME BECAUSE?
A. IT IS THEROMODYNAMICALLY VIABLE ONLY WHEN GLUCONEOGENESIS
HAS STARTED
B. IT REQUIRES ACTIVATION BY PYRUVATE KINASE
C. GLUCOSE 6 PHOSPHATASE IS PRESENT IN ENDOPLASMIC RETICULUM
D. STERIC HINDERANCE BY ALBUMIN
ANS: C. GLUCOSE 6 PHOSPHATASE IS PRESENT IN ENDOPLASMIC RETICULUM
(Ref: Chatterjee 5/e p488; Harper 29/e p19-21; 26/e p147)
DR MANJUNATH, DOCTORS ACADEMY,
DAVANAGERE
45
AIIMS MAY 2015-QUESTIONS & ANSWERS WITH EXPLANATIONS-SOLVED BY
DR: MANJUNATH, MBBS, MD (MAMC NEW DELHI) DIRECTOR
DOCTORS ACADEMY DAVANAGERE & SHIMOGA
(www.doctorsacademydvg.com)
EXPLANATION:
DR MANJUNATH, DOCTORS ACADEMY, DAVANAGERE
•THE CONVERSION GLUCOSE 6-PHOSPHATE TO GLUCOSE IS CATALYZED BY GLUCOSE-6-PHOSPHATASE.
•IT IS PRESENT IN LIVER AND KIDNEY BUT ABSENT FROM MUSCLE AND ADIPOSE TISSUE, WHICH,
THEREFORE, CANNOT EXPORT GLUCOSE INTO THE BLOODSTREAM.
•GLUCOSE-6-PHOSPHATASE CONSISTS OF 357 AMINO ACIDS, AND IS ANCHORED TO THE
ENENDOPLASMIC RETICULUM (ER) BY NINE TRANSMEMBRANE HELICIES.
•ITS N-TERMINAL AND ACTIVE SITE ARE FOUND ON THE LUMEN SIDE OF THE ER AND ITS C-TERMINUS
PROJECTS INTO THE CYTOPLASM.
•THE TRANSFER OF THE GLUCOSE 6-PHOSPHATE IS CARRIED OUT BY A TRANSPORTER PROTEIN (T1)
AND THE ENDOPLASMIC RETICULUM (ER) CONTAINS STRUCTURES ALLOWING THE EXIT OF THE
PHOSPHATE GROUP (T2) AND GLUCOSE (T3).
46
AIIMS MAY 2015-QUESTIONS & ANSWERS WITH EXPLANATIONS-SOLVED BY
DR: MANJUNATH, MBBS, MD (MAMC NEW DELHI) DIRECTOR
DOCTORS ACADEMY DAVANAGERE & SHIMOGA
(www.doctorsacademydvg.com)
18. WHICH IS NOT
GLUCOGENIC?
A. ACETYL COA
B. PYRUCATE
C. LACTATE
D. OXALOACETETE
ANS- A. ACETYL COA (ACETYL COA ENTERS KREB
S CYCLE FOR ATP PRODUCTION)
(Ref: Chatterjee 5/e p448: Harper 29/e p19-20;
26/e p153-155) DR MANJUNATH, DOCTORS ACADEMY,
DAVANAGERE
47
AIIMS NOVEMBER 2015-QUESTIONS & ANSWERS WITH EXPLANATIONS-SOLVED BY
DR: MANJUNATH, MBBS, MD (MAMC NEW DELHI) DIRECTOR
DOCTORS ACADEMY DAVANAGERE & SHIMOGA
(www.doctorsacademydvg.com)
EXPLANATION:
GLUCONEOGENESIS
DR MANJUNATH, DOCTORS ACADEMY, DAVANAGERE
WHAT IS IT? IT IS THE TERM USED TO INCLUDE ALL PATHWAYS RESPONSIBLE FOR
CONVERTING NON-CARBOHYDRATE PRECUSORS TO GLUCOSE OR
GLYCOGEN RESERVES.
MAJOR SUB-STRATES GLUCOGENIC AMINO ACIDS, LACTATE, GLYCEROL & PROPIONATE
SITE LIVER AND KIDNEY ARE THE MAJOR GLUCONEOGENIC TISSUES.
PURPOSE • IT MEETS THE NEEDS OF BODY FOR GLUCOSE WHEN CARBOHYDRATE IS NOT
AVAILABLE IN SUFFICIENT AMOUNT FROM DIET OR FROM GLYCOGEN.
A SUPPLY OF GLUCOSE IS NECESSARY ESPECIALLY FOR CNS & RBCs.
HYPOGLYCEMIA CAUSES BRAIN DYSFUNCTION,
GLUCOSE IS ALSO IMPORTANT IN MAINTAINING LEVEL OF INTERMEDIATES OF
CITRIC ACID CYCLE
GLUCONEOGENESIS ALSO CLEARS LACTATE PRODUCED BY MUSCLE & RBCs AND
GLYCEROL PRODUCED BY ADIPOSE TISSUE.
48
AIIMS MAY 2015-QUESTIONS & ANSWERS WITH EXPLANATIONS-SOLVED BY
DR: MANJUNATH, MBBS, MD (MAMC NEW DELHI) DIRECTOR
DOCTORS ACADEMY DAVANAGERE & SHIMOGA
(www.doctorsacademydvg.com)
19. WHICH IS NOT A METHOS
OF PROTEIN PRECIPITATION?
A. SALTING OUT WITH METALS
B. ACETONE & ALCOHOL
C. CHANGING PH OTHER THAN
ISOELECTRIC PH
D. TRICHLORO ACETIC ACID
ANS: C. CHANGING PH OTHER THAN
ISOELECTRIC PH
(Ref: Chatterjee 5/e p448: Harper 29/e p19-21)
DR MANJUNATH, DOCTORS ACADEMY, DAVANAGERE 49
AIIMS MAY 2015-QUESTIONS & ANSWERS WITH EXPLANATIONS-SOLVED BY
DR: MANJUNATH, MBBS, MD (MAMC NEW DELHI) DIRECTOR
DOCTORS ACADEMY DAVANAGERE & SHIMOGA
(www.doctorsacademydvg.com)
EXPLANATION:
METHODS OF PROTEIN PRECIPITATION:
DR MANJUNATH, DOCTORS ACADEMY, DAVANAGERE
•SALTING OUT:
•EXCESS SALT PRECIPITATES PROTEINS BECAUSE MOST OF WATER MOLECULES BECOME
TIED UP IN FORMING HYDRATION SHELLS AROUND SALT IONS.
•SO, LESS WATER IS AVAILABLE TO DISSOLVE PROTEINS
•AMMONIUM SULFATE IS MOST COMMONLY USED REAGENT FOR SALTING OUT PROTEIN
•THE SALT CONCETRATION AT WHICH PROTEIN PRECIPITATES DIFFERS FROM ONE
PROTEIN TO ANOTHER. EG GLOBULIN PRECIPITATE AT HALF SATURATION WHILE
ALBUMIN PRECIPITATES AT FULL SATURATION OF AMMONIUM SULPHATE.
•BY ORGANC SOLVENTS(PROTEIN IS LEAST SOLUBLE AT ISOELECTRIC PH)
•BY HEAVY METALS (LEAD, CADMIUM, MERCURY)
50
AIIMS NOVEMBER 2015-QUESTIONS & ANSWERS WITH EXPLANATIONS-SOLVED BY
DR: MANJUNATH, MBBS, MD (MAMC NEW DELHI) DIRECTOR
DOCTORS ACADEMY DAVANAGERE & SHIMOGA
(www.doctorsacademydvg.com)
PHYSIOLOGY
21. Na-IODIDE TRANSPORTER
IS PRESENT IN ALL EXCEPT?
A. PITUITARY
B. SALIVARY
C. PLACENTA
D. THYROID
ANS- A. PITUITARY
(Ref: Ganong 23/e p303)
DR MANJUNATH, DOCTORS ACADEMY, DAVANAGERE 51
AIIMS NOVEMBER 2015-QUESTIONS & ANSWERS WITH EXPLANATIONS-SOLVED BY
DR: MANJUNATH, MBBS, MD (MAMC NEW DELHI) DIRECTOR
DOCTORS ACADEMY, DAVANAGERE & SHIMOGA.
(www.doctorsacademydvg.com)
EXPLANATION:
DR MANJUNATH, DOCTORS ACADEMY, DAVANAGERE
“THE SALIVARY GLAND, THE GASTRIC MUCOSA, THE PLACENTA, THE CILIARY BODY OF
THE EYE, THE CHOROID PLEXUS, THE MAMMARY GLANDS, AND CERTAIN CANCERS
DERIVED FROM THESE TISSUES ALSO EXPRESS NA IODIDE TRANSPOTER, WHICH CAN
TRANSPORT IODIDE AGAINST A CONCENTRATION GRADIENT.”
BUT THE TRANSPORTER IN THESE TISSUES IS NOT AFFECTED BY TSH.
THE PHYSIOLOGIC SIGNIFICANCE OF ALL THESE EXTRATHYROIDAL IODIDE-
CONCENTRATING MECHANISM IS OBSCURE.
THEY MAY PROVIDE PATHWAY FOR RADIOABLATION OF NA IODIDE TRANSPORTER-
EXPRESSING CANCER CELLS USING IODIDE RADIOIDOTOPES.
THIS APPROACH IS ALSO USEFUL FOR THE ABLATION OF THYROID CANCERS.
52
AIIMS NOVEMBER 2015-QUESTIONS & ANSWERS WITH EXPLANATIONS-SOLVED BY
DR: MANJUNATH, MBBS, MD (MAMC NEW DELHI) DIRECTOR
DOCTORS ACADEMY DAVANAGERE & SHIMOGA
(www.doctorsacademydvg.com)
22. ATRIAL NATRIURETIC PEPTIDE EXERTS ITS ACTION ON
ALL EXCEPT?
A. INHIBITION OF Na+ REABSORPTION IN PCT
B. DILATE AFFERENT ARTERIOLE
C. MESANGIAL CELL CONTRACTION
D. INHIBITION OF Na+ REABSORPTION IN MEDULARRY COLLECTING DUCT
ANS: C. MESANGIAL CELL CONTRACTION
(ANP is a powerful vasodilator, and a protein (polypeptide) hormone secreted by heart
muscle cells. It is involved in the homeostatic control of body water, sodium, potassium
and fat (adipose tissue). It is released by muscle cells in the upper chambers (atria) of
the heart (atrial myocytes) in response to high blood volume. ANP acts to reduce the
water, sodium and adipose loads on the circulatory system).
(Ref: Ganong 23/e p675)
DR MANJUNATH, DOCTORS ACADEMY, DAVANAGERE 53
AIIMS NOVEMBER 2015-QUESTIONS & ANSWERS WITH EXPLANATIONS-SOLVED BY
DR: MANJUNATH, MBBS, MD (MAMC NEW DELHI) DIRECTOR
DOCTORS ACADEMY DAVANAGERE & SHIMOGA
(www.doctorsacademydvg.com)
EXPLANATION:
ACTIONS OF ATRIAL NATRIURETIC PEPTIDE (ANP) & BRAIN NATRIURETIC PEPTIDE (BNP)
DR MANJUNATH, DOCTORS ACADEMY, DAVANAGERE
• THEY ACT ON THE KIDNEY TO INCREASE NA+ EXCRETION
• THEY APPEAR TO PRODUCE THIS EFFECT BY DILATING AFFERENT ARTERIOLES AND
RELAXING MESANGIAL CELLS. BOTH OF THESE ACTIONS INCREASE GLOMERULAR
FILTRATION
•THEY ACT ON THE RENAL TUBULES TO NHIBIT NA+ REABSORPTION.
•OTHER ACTIONS INCLUDE AN INCREASE IN CAPILLARY PERMEABILITY, LEADING TO
EXTRAVASATION OF FLUID AND A DECLINE IN BLOOD PRESSURE.
•THEY RELAX VASCULAR SMOOTH MUSCLE IN ARTERIOLES AND VENULES
54
AIIMS NOV 2015-QUESTIONS & ANSWERS WITH EXPLANATIONS-SOLVED BY
DR: MANJUNATH, MBBS, MD (MAMC NEW DELHI) DIRECTOR
DOCTORS ACADEMY DAVANAGERE & SHIMOGA
(www.doctorsacademydvg.com)
23. INTERSTITIAL FLUID VOLUME CAN BE
DETERMINED BY ?
A. RADIOACTIVE SODIUM AND RADIOIODINE LABELLED ALBUMIN
B. RADIOACTIVE WATER AND RADIOLABELLED ALBUMIN
C. RADIOACTIVE SODIUM AND RADIOACTIVE WATER
D. DIIODOTHALMIUM IODIONE
ANS- A. RADIOACTIVE SODIUM AND RADIOIODINE LABELLED ALBUMIN
(Ref: Ganong 24e p3-5, Guyton 12/534-536, 541)
DR MANJUNATH, DOCTORS ACADEMY, DAVANAGERE 55
AIIMS NOV 2015-QUESTIONS & ANSWERS WITH EXPLANATIONS-SOLVED BY
DR: MANJUNATH, MBBS, MD (MAMC NEW DELHI) DIRECTOR
DOCTORS ACADEMY DAVANAGERE & SHIMOGA
(www.doctorsacademydvg.com)
EXPLANATION:
MEASUREMENT OF BODY FLUID VOLUME
DR MANJUNATH, DOCTORS ACADEMY, DAVANAGERE
VOLUME INDICATORS
TOTAL BODY WATER 3H2O, 2H2O, ANTIPYRINE
EXTRACELLULAR FLUID 22NA, 125I-OTHAMAMATE, THIOSULFATE, INULIN
EXTRACELLULAR FLUID (CALCULATED AS TOTAL BODY WATER-EXTRACELLULAR FLUID VOLUME)
PLASMA VOLUME 125I-ALBUMIN, EVANS BLUE DYE (T-1824)
BLOOD VOLUME 51CR-LABELED RED BLOOD CELLS, OR CALCULATED AS BLOOD VOLUME= PLASMA
VOLUME/(1-HEMATOCRIT)
INTERSTITIAL FLUID (CALCULATE AS EXTRACELLULAR FLUID VOLUME-PLASM VOLUME)
56
AIIMS NOV 2015-QUESTIONS & ANSWERS WITH EXPLANATIONS-SOLVED BY
DR: MANJUNATH, MBBS, MD (MAMC NEW DELHI) DIRECTOR
DOCTORS ACADEMY DAVANAGERE & SHIMOGA
(www.doctorsacademydvg.com)
24. THE CLOT FORMED IN NOT STABLE UNLESS EXTENSIVE CROSS
LINKING OCCURS. THIS IS DONE BY?
A. PLASMIN
B. THROMBIN
C. FACTOR XIII
D. HMWK
ANSWER: C. FACTOR XIII
REF: GANONG 23/E P531
57DR MANJUNATH, DOCTORS ACADEMY, DAVANAGERE
AIIMS NOV 2015-QUESTIONS & ANSWERS WITH EXPLANATIONS-SOLVED BY
DR: MANJUNATH, MBBS, MD (MAMC NEW DELHI) DIRECTOR
DOCTORS ACADEMY DAVANAGERE & SHIMOGA
(www.doctorsacademydvg.com)
58DR MANJUNATH, DOCTORS ACADEMY, DAVANAGERE
AIIMS NOV 2015-QUESTIONS & ANSWERS WITH EXPLANATIONS-SOLVED BY
DR: MANJUNATH, MBBS, MD (MAMC NEW DELHI) DIRECTOR
DOCTORS ACADEMY DAVANAGERE & SHIMOGA
(www.doctorsacademydvg.com)
25. PHYSIOLOGICAL CHANGES IN LAPAROSCOPY INCLUDE ALL
EXCEPT?
A. INCREASED PCWP
B. INCREASED ICP
C. DECREASED FRC
D. INCREASED PH
ANSWER: D. INCREASED PH (THERE IS DECREASE IN PH UE TO METABOLIC ACIDOSIS)
REF: WYLIE CBURCBILL-DAVIDSON’S A PRACTICE OF ANAESTHESIA 7/E P896
59DR MANJUNATH, DOCTORS ACADEMY, DAVANAGERE
AIIMS NOV 2015-QUESTIONS & ANSWERS WITH EXPLANATIONS-SOLVED BY
DR: MANJUNATH, MBBS, MD (MAMC NEW DELHI) DIRECTOR
DOCTORS ACADEMY DAVANAGERE & SHIMOGA
(www.doctorsacademydvg.com)
26. ALL OF THE STATEMENTS ARE TRUE ABOUT SMOOTH
MUSLE CONTRACTION EXCEPT ?
A. TENSION DEVELOPED IS PROPORTIONAL TO OR AFFECTED BY FREQUENCY OF
SPIKE POTENTIALS
B. MUSCLE CONTRACTION IS RELATED TO THE AMPLITUDE OF BASAL ELECTRICAL
RHYTHUM (BER)
C. FREQUENCY IS AROUND 6/MIN
D. THRESHOULD IS -50mV
ANS- B. MUSCLE CONTRACTION IS RELATED TO THE AMPLITUDE OF BER (BER ITSELF RARELY
CAUSES MUSLE CONTRACTION)
(Ref: Ganong 24/e p498-499, 23/e p471)
DR MANJUNATH, DOCTORS ACADEMY, DAVANAGERE 60
AIIMS NOV 2015-QUESTIONS & ANSWERS WITH EXPLANATIONS-SOLVED BY
DR: MANJUNATH, MBBS, MD (MAMC NEW DELHI) DIRECTOR
DOCTORS ACADEMY DAVANAGERE & SHIMOGA
(www.doctorsacademydvg.com)
29. IN THE FORMULA,
CLEARANCE C=U x V/P,U
STANDS FOR?
A. URINARY CONCENTRATION IN
GM/24 HRS
B. URINARY CONCENTRATION IN
MG/100ML
C. MICROALBIMINURIA
D. URINE OSMOLARITY
ANS- D. URINE OSMOLARITY (REF: GUYTON’S
12/E .663)
Ref: Guyton’s 12/e p663
DR MANJUNATH, DOCTORS ACADEMY, DAVANAGERE 61
AIIMS NOVEMBER 2015-QUESTIONS & ANSWERS WITH EXPLANATIONS-SOLVED BY
DR: MANJUNATH, MBBS, MD (MAMC NEW DELHI) DIRECTOR
DOCTORS ACADEMY DAVANAGERE & SHIMOGA
(www.doctorsacademydvg.com)
30. ANATOMICAL DEAD SPACE IS MEASURED WITH ?
A. SINGLE BREATH NITROGEN TEST OR NITROGEN WASH OUT
B. BOHLERSETHOD
C. SPIROMETER
D. XENON ISOTOPES
ANS: A. SINGLE BREATH NITROGEN TEST
REF: GUYTON’S 12/E P883-885
(Nitrogen washout (or Fowler's method) is a test for measuring anatomic dead space in
the lung during a respiratory cycle , as well as some parameters related to the closure
of airways.)
DR MANJUNATH, DOCTORS ACADEMY, DAVANAGERE 62
AIIMS NOVEMBER 2015-QUESTIONS & ANSWERS WITH EXPLANATIONS-SOLVED BY
DR: MANJUNATH, MBBS, MD (MAMC NEW DELHI) DIRECTOR
DOCTORS ACADEMY DAVANAGERE & SHIMOGA
(www.doctorsacademydvg.com)
DR MANJUNATH, DOCTORS ACADEMY, DAVANAGERE 63
AIIMS NOVEMBER 2015-QUESTIONS & ANSWERS WITH EXPLANATIONS-SOLVED BY
DR: MANJUNATH, MBBS, MD (MAMC NEW DELHI) DIRECTOR
DOCTORS ACADEMY DAVANAGERE & SHIMOGA
(www.doctorsacademydvg.com)
PATHOLOGY
31. HISTOPATHOLOGY OF
SCHWANNOMA,FOLLOWING IS
SEEN?
A. ANTOMY A WITH VEROCAY BODY
B. ROSETTES
C. ANTOMY B WITH VEROCAY BODY
D. PALLISADING
ANS: A. ANTOMY A WITH VEROCAY BODY
REF:ROBBIN’S 9/E P1314; COMPLETE REVIEW OF
PATHOLOGY 1/E P710
DR MANJUNATH, DOCTORS ACADEMY, DAVANAGERE 64
AIIMS NOVEMBER 2015-QUESTIONS & ANSWERS WITH EXPLANATIONS-SOLVED BY
DR: MANJUNATH, MBBS, MD (MAMC NEW DELHI) DIRECTOR
DOCTORS ACADEMY DAVANAGERE & SHIMOGA
(www.doctorsacademydvg.com)
DR MANJUNATH, DOCTORS ACADEMY, DAVANAGERE
FEATURES SCHWANNOMAS NEUROFIBROMAS
GENERAL
CHARACTERISTICS
 BENIGN TUMOR
 ARISE DIRECTLY FROM
PERIPHERAL NERVES.
 BENIGN TUMORS,
HETEROGEOUS IN
COMPOSITION
 NEOPLASTIC SCHWANN
CELLS MIXED WITH
PERINURIAL LIKE CELLS,
FIBROBLASTS, MAST CELLS,
AND CD34+ SPINCLE CELLS
GROSS  WELL-CIRCUMSCRIBED
&ENCAPSULATED
 BUT ASSOCIATED NERVE
WITHOUT INVADING
 NON-ENCAPSULATED MASS
MORPHOLOGY CONSISTS OF:
 ANTONI A: CELLULAR AREAS
 ANTONI B: LOOSE
EDEMATOUS AREAS
3 TYPES:
 LOCALIZED CUTANEOUS
NEURROFIBROMA
 DIFFUSE NEUROFIBROMA
 PLEXIFORMNEUROFIBROMA
65
AIIMS NOVEMBER 2015-QUESTIONS & ANSWERS WITH EXPLANATIONS-SOLVED BY
DR: MANJUNATH, MBBS, MD (MAMC NEW DELHI) DIRECTOR
DOCTORS ACADEMY DAVANAGERE & SHIMOGA
(www.doctorsacademydvg.com): PATHOLOGY OF SCHWANNOMAS
• Schwannomas are benign encapsulated neoplasms of schwann cells (WHO
grade I). They arise eccentrically from their parent nerve. They are composed
of two cell types: Antoni A and Antoni B.
• The Antoni A cells are densely packed and arranged in fascicles; acellular areas
lie between opposing rows of parallel nuclei (Verocay bodies).
• The Antoni B cells are less compact and are prone to cystic degeneration.
• PATHOLOGIC VARIANTS INCLUDE :
• conventional (most common type): fibrous capsule, hyaline vessels, Antoni A
and loose textured Antoni B areas (Verocay bodies)
• cellular schwannomas (predominantly Antoni A tissue without Verocay bodies)
• melanotic schwannomas (dense melanin pigment)
• plexiform schwannomas (do not undergo malignant change, unlike plexiform
neurofibromas)
DR MANJUNATH, DOCTORS ACADEMY, DAVANAGERE 66
AIIMS NOVEMBER 2015-QUESTIONS & ANSWERS WITH EXPLANATIONS-SOLVED BY
DR: MANJUNATH, MBBS, MD (MAMC NEW DELHI) DIRECTOR
DOCTORS ACADEMY DAVANAGERE & SHIMOGA
(www.doctorsacademydvg.com): VESTIBULAR SCHWANNOMA
DR MANJUNATH, DOCTORS ACADEMY, DAVANAGERE 67
AIIMS NOVEMBER 2015-QUESTIONS & ANSWERS WITH EXPLANATIONS-SOLVED BY
DR: MANJUNATH, MBBS, MD (MAMC NEW DELHI) DIRECTOR
DOCTORS ACADEMY DAVANAGERE & SHIMOGA
(www.doctorsacademydvg.com)
32. A 30 YEAR OLD SOFTWARE ENGINEER CAME
TO OPD WITH CHIEF COMPLAINS OF
HEARTBURN. ON ENDOSCOPIC BIOPSY, THE
LESION SHOWS THE FOLLOWING (FIGURE
BELOW). IDENTIFY THE LESION, STAIN HAS
DONE FOR THE WHAT AND WHAT
ADDITIONAL FEATURES SHOULD BE LOOKED
FOR?
A. ADENOCACINOMA; PAS; MALIGNANCY
B. BARRETTS OESOPHAGUS; MUCIN STAIN;
DYSPLASIA
C. SQUAMOUS CELL CARCINOMA;
CYTOKERATIN, SQUAMOUS PEARLE
D. INFECTION; FUNGAL STAIN; INCLUSIO BODY
ANS: B. BARRETTS OESOPHAGUS; MUCIN STAIN; DYSPLASIA
REF: ROBBIN’S 9/757; COMPLETE REVIEW OF PATHOLOGY
1/E P482
DR MANJUNATH, DOCTORS
ACADEMY, DAVANAGEREDR. MANJUNATH, DOCTORS ACADEMY 68
AIIMS NOVEMBER 2015-QUESTIONS & ANSWERS WITH EXPLANATIONS-SOLVED BY
DR: MANJUNATH, MBBS, MD (MAMC NEW DELHI) DIRECTOR
DOCTORS ACADEMY DAVANAGERE & SHIMOGA
(www.doctorsacademydvg.com)
EXPLANATION:
DR MANJUNATH, DOCTORS ACADEMY, DAVANAGERE
Barrett esophagus represents progressive metaplasia of oesophageal stratified
squamous cell epithelium to columnar epithelium.
Although the exact number varies, 90-100% of oesophgeal adenocarcinoma is thought
to arise from this metaplasia.
Although patients with Barrett oesophagus have a 30x risk of developing oesophageal
adenocarcinoma, the annual risk of developing adenocarcinoma depends on the
degree of histological dysplasia, but may be ~1% (range 0.1-2%), and the absolute risk
is low.
69
AIIMS NOVEMBER 2015-QUESTIONS & ANSWERS WITH EXPLANATIONS-SOLVED BY
DR: MANJUNATH, MBBS, MD (MAMC NEW DELHI) DIRECTOR
DOCTORS ACADEMY DAVANAGERE & SHIMOGA
(www.doctorsacademydvg.com)
33. THE GIVEN FIGURE SHOWS WHICH OF
THE FOLLOWING?
A. AMYLOIDOSIS-GREY; VIABLE WHITE NECROTIC
B. NUTMEG LIVER-RED AREAS ARE VIABLE
PERICENTRAL AREAS; WHITE AREAS ARE
PERIPORTAL NECROTIC AREAS
C. RED AREAS ARE NECROTIC AREAS NEAR CENTRAL
VEIN, WHITE AREAS ARE VIABLE, FIBROTIC
PERIPORTAL AREA
D. AMYLOIDOSIS-NECROTIC WHITE PERIPORTAL VIABLE
GREY PERICENTRAL AREAS
ANS: C. RED AREAS ARE NECROTIC AREAS NEAR
CENTRAL VEIN, WHITE AREAS ARE VIABLE,
FIBROTIC PERIPORTAL AREA (CHRONIC HEAPTIC
CONGESTION)
REF: ROBBIN’S 9/129; COMPLETE REVIEW OF
PATHOLOGY 1/E P86
DR MANJUNATH, DOCTORS
ACADEMY, DAVANAGEREDR. MANJUNATH, DOCTORS ACADEMY 70
AIIMS NOVEMBER 2015-QUESTIONS & ANSWERS WITH EXPLANATIONS-SOLVED BY
DR: MANJUNATH, MBBS, MD (MAMC NEW DELHI) DIRECTOR
DOCTORS ACADEMY DAVANAGERE & SHIMOGA
(www.doctorsacademydvg.com)
EXPLANATION:
DR MANJUNATH, DOCTORS
ACADEMY, DAVANAGEREDR. MANJUNATH, DOCTORS ACADEMY
ACUTE PULMONARY CONGESTION:
 ENGORGED ALVEOLAR CAPILLARIES
 ALVEOLAR SEPTALEDEMA
 FOCAL INTRA ALVEOLAR HEMORRHAGE
CHRONIC PULMONARY CONGESTION
 SEPTA ARE THICKENED AND FIBROTIC
 HEMOSIDERIN-LADEN MACROPHAGES(HEART FAILURE CELLS)
ACUTE HEPATIC CONGESTION
 CENTRILOBULAR HEPATOCYTES: ISCHEMIC NECROSIS
 PERIPORTAL HEPATOCYTES-FATTY CHANGE
CHRONIC PASSIVE HEPATIC CONGESTION
 NUTMEG LIVER: CENTRILOBULAR REGIONS ARERED-BROEN AGAINST SURROUNDING ZONES
OF UNCONGESTED TAN LIVER
 INITIALLY CENTRILOBULAR NECROSIS & HEMOSIDERIN LADEN MACROPHAGES.
 LATER: HEPATIC FIBROSIS CALLED CARDIAC CIRRHOSIS.
71
AIIMS NOVEMBER 2015-QUESTIONS & ANSWERS WITH EXPLANATIONS-SOLVED BY
DR: MANJUNATH, MBBS, MD (MAMC NEW DELHI) DIRECTOR
DOCTORS ACADEMY DAVANAGERE & SHIMOGA
(www.doctorsacademydvg.com)
34. WHICH OF THE FOLLOWING IS A IMMUNE
PRIVILEGED REGION?
A. AREA POSTREMA
B. SEMINIFEROUS TUBULES
C. KIDNEY
D. OPTIC NERVE
ANS: B. SEMINIFEROUS TUBULES
[REF: ROBBIN’S 9/215-216]
DR MANJUNATH, DOCTORS
ACADEMY, DAVANAGEREDR. MANJUNATH, DOCTORS ACADEMY 72
AIIMS NOVEMBER 2015-QUESTIONS & ANSWERS WITH EXPLANATIONS-SOLVED BY
DR: MANJUNATH, MBBS, MD (MAMC NEW DELHI) DIRECTOR
DOCTORS ACADEMY DAVANAGERE & SHIMOGA
(www.doctorsacademydvg.com)
EXPLANATION:
DR MANJUNATH, DOCTORS
ACADEMY, DAVANAGEREDR. MANJUNATH, DOCTORS ACADEMY
IMMUNE-PRIVILEGED SITES:
 TESTIS, EYE AND BRAIN
 TISSUES IN WHICH THESE ANTIGENS ARE LOCATED DO NOT COMMUNICATE
WITH THE BLOOD AND LYMPH
 DIFFICULT TO INDUCE IMMUNE RESPONSES TO ANTIGEN INTRODUCED INTO
THESE SITES
 PROLONGED TISSUE INFLAMMATION ON INJURY & RELEASE OF ANTIGEN FROM
THESE SITES; POST-TRAUMATIC ORCHITIS & UVEIRIS
73
AIIMS NOVEMBER 2015-QUESTIONS & ANSWERS WITH EXPLANATIONS-SOLVED BY
DR: MANJUNATH, MBBS, MD (MAMC NEW DELHI) DIRECTOR
DOCTORS ACADEMY DAVANAGERE & SHIMOGA
(www.doctorsacademydvg.com)
35. PATHOLOGIST EXAMINES BIOPSY
FROM A PATIENT PRESENTING WITH
BLEEDING PER RECTUM WITH A PAST
HISTORY OF INTUSSUPTION FOR THE
PAST 6MONTHS. HIS OBTAINED
FINDING HAS BEEN SHOWN BELOW.
IDENTIFY THE PATHOLOGY?
A. TUBULE VILLOUS ADENOMA
B. ADENO CARCINOMA
Ç. HAMARTOMA
D. JUVENILE POLYPOSIS SYNDROME
ANS: A. TUBULE VILLOUS ADENOMA
REF: ROBBIN’S 9/808-809; COMPLETE REVIEW OF
PATHOLOGY 1/E P517
DR MANJUNATH, DOCTORS
ACADEMY, DAVANAGEREDR. MANJUNATH, DOCTORS ACADEMY 74
AIIMS NOVEMBER 2015-QUESTIONS & ANSWERS WITH EXPLANATIONS-SOLVED BY
DR: MANJUNATH, MBBS, MD (MAMC NEW DELHI) DIRECTOR
DOCTORS ACADEMY DAVANAGERE & SHIMOGA
(www.doctorsacademydvg.com)
EXPLANATION:
ADENOMAS CAN BE CLASSIFIED AS: (BASED ON THEIR ARCHITECTURE)
DR MANJUNATH, DOCTORS
ACADEMY, DAVANAGEREDR. MANJUNATH, DOCTORS ACADEMY
TUBULAR SMALL, PEDUNCULATED POLYPS COMPOSED OF ROUNDED,
OR TUBULAR, GLANDS
TUBULO-VILLOUS
(PICTURE)
MIXTURE OF TUBULAR AND VILLOUS ELEMENTS
VILLOUS LARGER AND SESSILE, ARE COVERED BY SLENDER VILLI
75
AIIMS NOVEMBER 2015-QUESTIONS & ANSWERS WITH EXPLANATIONS-SOLVED BY
DR: MANJUNATH, MBBS, MD (MAMC NEW DELHI) DIRECTOR
DOCTORS ACADEMY DAVANAGERE & SHIMOGA
(www.doctorsacademydvg.com)
36. WHICH ONE IS BEST PROGNOSTIC FACTOR FOR
ALL?
A. HYPERPLOIDY
B. ORGANOMEGALY
C. TLC MORE THAN 50,000/UL
D. ENVIROMENTAL FACTORS
ANS: A
[REF: ROBBIN’S 9/590-592]
DR MANJUNATH, DOCTORS
ACADEMY, DAVANAGEREDR. MANJUNATH, DOCTORS ACADEMY 76
AIIMS NOVEMBER 2015-QUESTIONS & ANSWERS WITH EXPLANATIONS-SOLVED BY
DR: MANJUNATH, MBBS, MD (MAMC NEW DELHI) DIRECTOR
DOCTORS ACADEMY DAVANAGERE & SHIMOGA
(www.doctorsacademydvg.com)
EXPLANATION:
PROGNOSTIC FACTORS IN ACUTE LYMPHOBLASTIC LEUKEMIA
DR MANJUNATH, DOCTORS
ACADEMY, DAVANAGEREDR. MANJUNATH, DOCTORS ACADEMY
DETERMINANTS FAVORABLE UNFAVORABLE
WBC/UL <10,000 >2,00,000
AGE 2-9 YR <1 Y,>10 Y
GENDER FEMALE MALE
ETHNICITY WHITE BLACK
L.NODE, LIVER. SPLEEN
ENLARGEMENT
ABSENT MASSIVE
TESTICULAR
ENLARGEMENT
ABSENT PRESENT
CENTRAL NERVOUS
SYSTEM LEUKEMIA
ABSENT PRESENT
77
AIIMS NOVEMBER 2015-QUESTIONS & ANSWERS WITH EXPLANATIONS-SOLVED BY
DR: MANJUNATH, MBBS, MD (MAMC NEW DELHI) DIRECTOR
DOCTORS ACADEMY DAVANAGERE & SHIMOGA
(www.doctorsacademydvg.com)
37. 10 YEAR CHILD PRESENTED WITH BILATERAL
CERVICAL LYMPHADENOPATHY. LYMPHNODE
BIOPSY WAS PERFORMED, WHICH SHOWED
CELLS AS GIVEN IN THE FIGURE. WHICH OF THE
FOLLOWING IS TRUE REGARDING THIS
CONDITION?
A. HODGKIN LYMPHOMA;EBV AND EMBRYO
CELL
B. NON HODGKIN LYMPHOMA; HIV AND GIANT
B CELL
C. TB, MYCOBACTERIA AND TINY GRANULOMA
D. HODGKIN LYMPHOMA; EBV AND REED
STERNBERG CELL
ANS: D. HODGKIN LYMPHOMA; EBV AND REED
STERNBERG CELL
REF: ROBBIN’S 9/608-611; COMPLETE REVIEW OF
PATHOLOGY 1/E P285, 761
DR MANJUNATH, DOCTORS
ACADEMY, DAVANAGEREDR. MANJUNATH, DOCTORS ACADEMY 78
AIIMS NOVEMBER 2015-QUESTIONS & ANSWERS WITH EXPLANATIONS-SOLVED BY
DR: MANJUNATH, MBBS, MD (MAMC NEW DELHI) DIRECTOR
DOCTORS ACADEMY DAVANAGERE & SHIMOGA
(www.doctorsacademydvg.com)
EXPLANATION:
Hodgkin’s Lymphoma:
DR MANJUNATH, DOCTORS
ACADEMY, DAVANAGEREDR. MANJUNATH, DOCTORS ACADEMY
• USUALLY PRESENTS AS SOLITARY OR GENERALIZED LYMPHADENOPATHY AND
MOST COMMONLY OCCURS IN YOUNG ADULTS, ALTHOUGH ANY AGE GROUP MAY
BE AFFECT ED.
• THE DISEASE APPEARS TO SPREAD IN A CONTIGUOUS FASHION, AND MOST
PATIENT PRESENT WITH DISEASE LIMITED TO THE LYMPH NODES OR TO THE
LYMPH NODES AND SPLEEN
• THE FIGURE GIVEN IN THIS EQUESTION IS “CLASSIC”HODGKIN LYMPHOMA.
• THE BACKGROUND IS PRIMARY LYMPHOCYTES, AND THE REED-STERNBERG CELLS
79
AIIMS NOVEMBER 2015-QUESTIONS & ANSWERS WITH EXPLANATIONS-SOLVED BY
DR: MANJUNATH, MBBS, MD (MAMC NEW DELHI) DIRECTOR
DOCTORS ACADEMY DAVANAGERE & SHIMOGA
(www.doctorsacademydvg.com)- HODKINS LYMPHOMA
DR MANJUNATH, DOCTORS
ACADEMY, DAVANAGEREDR. MANJUNATH, DOCTORS ACADEMY 80
AIIMS NOVEMBER 2015-QUESTIONS & ANSWERS WITH EXPLANATIONS-SOLVED BY
DR: MANJUNATH, MBBS, MD (MAMC NEW DELHI) DIRECTOR
DOCTORS ACADEMY DAVANAGERE & SHIMOGA
(www.doctorsacademydvg.com)
38. IRON METABOLISM AND
REGULATION ARE IMPORTANT FOR
RBC PRECURSOR CELL. WHICH OF
THE FOLLOWING HELPS IN
REGULATION OF IRON
METABOLISM BUT IS NOT SPECIFIC
FOR IRON?
• A. HEPCIDIN
• B. DMT-1
• C. FERROPORTIN
• D. FERRITIN
•
ANS- B. DMT-1
[REF: WINTROBE 13ED/PG 811]
DR MANJUNATH, DOCTORS
ACADEMY, DAVANAGEREDR. MANJUNATH, DOCTORS ACADEMY 81
AIIMS NOVEMBER 2015-QUESTIONS & ANSWERS WITH EXPLANATIONS-SOLVED BY
DR: MANJUNATH, MBBS, MD (MAMC NEW DELHI) DIRECTOR
DOCTORS ACADEMY DAVANAGERE & SHIMOGA
(www.doctorsacademydvg.com)
EXPLANATION:
DR MANJUNATH, DOCTORS
ACADEMY, DAVANAGEREDR. MANJUNATH, DOCTORS ACADEMY
 *HEPHAESTIN IS SIMILAR TO CERULOPLASMIN, THE COPPER-CARRYING PROTEIN.
 **DIVALENT METAL TRANSPORTER TYPE 1 (DMT-1): ALSO KNOEN AS NATURAL RE-
SISTANCE MACROPHAGE-ASOCIATED PROTEIN TYPE 2 (NRAMP 2) OR DCT-1
 HEPCIDIN: PRINCIPAL IRON REGULATORY HORMONE; AND IS NEGATIVELY
REGULATED BY FERROPORTIN
 DMT1 HAS A STRICT REQUIREMENT FOR DIVALENT CATIONS, AND IT WILL NOR
TRANSPORT THE FE3+ FROM OF IRON
82
AIIMS NOVEMBER 2015-QUESTIONS & ANSWERS WITH EXPLANATIONS-SOLVED BY
DR: MANJUNATH, MBBS, MD (MAMC NEW DELHI) DIRECTOR
DOCTORS ACADEMY DAVANAGERE & SHIMOGA
(www.doctorsacademydvg.com)
39. THERE ARE DIFFERENT
CHECK POINTS IN CELL
GROWTH AND REGULATION.
WHICH ONE IS THE PRIMARY
POINT FOR REGULATION OF
CELL GROWTH?
A. END OF G1
B. START OF G2
C. END OF S1
D. END OF M
ANS: A. END OF G1
REF: ROBBIN’S 9/25; COMPLETE REVIEW
OF PATHOLOGY 1/E P197
DR MANJUNATH, DOCTORS
ACADEMY, DAVANAGEREDR. MANJUNATH, DOCTORS ACADEMY 83
AIIMS NOVEMBER 2015-QUESTIONS & ANSWERS WITH EXPLANATIONS-SOLVED BY
DR: MANJUNATH, MBBS, MD (MAMC NEW DELHI) DIRECTOR
DOCTORS ACADEMY DAVANAGERE & SHIMOGA
(www.doctorsacademydvg.com)
EXPLANATION
CELL CYCLE
Definition: Cell cycle is the sequence of events that results in cell division
DR MANJUNATH, DOCTORS
ACADEMY, DAVANAGEREDR. MANJUNATH, DOCTORS ACADEMY
 PHASES: G0→G1→S→G2→M
 G1: PRE-SYNTHETIC GROWTH
 S: DNA SYNTHESIS (MOST RADIO-RESISTANT PHASE)
 G2: PRE-MITOTIC GROWTH
 M: MITOTIC PHASE (MOST RADIO-SENSITIVE PHASE)
 G0: QUIESCENT CELLS THAT ARE NOT ACTIVELY CYCLING ARE SAID TO BE IN GO STATE
84
AIIMS NOVEMBER 2015-QUESTIONS & ANSWERS WITH EXPLANATIONS-SOLVED BY
DR: MANJUNATH, MBBS, MD (MAMC NEW DELHI) DIRECTOR
DOCTORS ACADEMY DAVANAGERE & SHIMOGA
(www.doctorsacademydvg.com)
40. IDENTIFY THE CRYSTALS IN THE URINE ANALYSIS?
A. OXALATE
B. URIC ACID
C. PHOSPHAHATES
D. CYSTEINE
ANS: A. OXALATE
DR MANJUNATH, DOCTORS
ACADEMY, DAVANAGEREDR. MANJUNATH, DOCTORS ACADEMY 85
AIIMS NOV 2015-QUESTIONS & ANSWERS WITH EXPLANATIONS-SOLVED BY
DR: MANJUNATH, MBBS, MD (MAMC NEW DELHI) DIRECTOR
DOCTORS ACADEMY DAVANAGERE & SHIMOGA
(www.doctorsacademydvg.com)
EXPLANATION
Crystals you can get in Urine
DR MANJUNATH, DOCTORS
ACADEMY, DAVANAGEREDR. MANJUNATH, DOCTORS ACADEMY
TRIPLE PHOSPHATE  SEEN IN ALKALINE URINE
 RECTANGULAR SHAPE
URIC ACID  SEEN IN ACIDIC URINE
 BROWN LEMONSHAPED
OR STAR SHAPED
 BIREFRINGENT WITH
POLARIZED LIGHT
CALCIUM OXALATE  ENVELOPE SHAPED
 SEEN IN ACIDIC URINE
86
AIIMS NOV 2015-QUESTIONS & ANSWERS WITH EXPLANATIONS-SOLVED BY
DR: MANJUNATH, MBBS, MD (MAMC NEW DELHI) DIRECTOR
DOCTORS ACADEMY DAVANAGERE & SHIMOGA
(www.doctorsacademydvg.com)
41. CAVITATORY LESION IN RIGHT
LOWER LUNG WITH DYSPONEA
WITH FOLLOWING
HISTOPATHOLOGICAL
APPEARANCE?
A. ECHINOCOCCUS WITH 2 LAYERS
B. STRONGYLOIDES WITH 2 LAYERS
C. PARAGONIMUS WITH 2 LAYERS
D. CYSTICERCOSIS WITH 3 LAYERS
ANS: C. PARAGONIMUS WITH 2 LAYERS
REF: HARRISON’S 19/E P1429; CDC WEBSITE
DR MANJUNATH, DOCTORS
ACADEMY, DAVANAGEREDR. MANJUNATH, DOCTORS ACADEMY 87
AIIMS NOV 2015-QUESTIONS & ANSWERS WITH EXPLANATIONS-SOLVED BY
DR: MANJUNATH, MBBS, MD (MAMC NEW DELHI) DIRECTOR
DOCTORS ACADEMY DAVANAGERE & SHIMOGA
(www.doctorsacademydvg.com)
EXPLANATION: THE FIGURE SHOWS: EGG OF PARAGONIMUS SP. TAKEN FROM A LUNG
BIOPSY STAINED WITH HEMATOXYLIN AND EOSIN
DR MANJUNATH, DOCTORS
ACADEMY, DAVANAGEREDR. MANJUNATH, DOCTORS ACADEMY
GEOGRAPHIC
DISTRIBUTION
 PARAGONIUM WESTERMANI IS DISTRIBUTED IN SOUTHEAST ASIA AND JAPAN.
CLINICAL PRE-
SENTATION
 ACUTE PHASE: DIARRHEA, ABDOMINAL PAIN, FEVER, COUGH, URTICARIA,
HEPATOSPLENOMEGALY & EOSINOPHILIA.
 DURING THE CHRONIC PHASE, PULMONARY MANIFESTATIONS INCLUDE COUGH,
EXPECTORATION OF DISCOLORED SPUTUM & HEMOPTYSIS
MODE OF
TRANSMISSION
 HUMANS ACQUIRE LUNG FLUKE INFECTION BY INGESTING INFECTIVE METACERCERCARIAE
ENCYSTED IN THE MUSCLES AND VISCERA OF CRAYFISH AND FRESHWATER CRABS.
 IN ENDEMIC AREAS, THESE CRUSTACEANS ARE CONSUMED RAW, MATINATED, OR PICKLED.
 ONCE THE ORGANISM REACH THE DUODENUM, THEY EXCYST, PENETRATE THE GUT WALL,
AND TRAVEL THROUGH THE PERITONEAL CAVITY, DIAPHRAGM, AND PLEURAL SPACE TO
REACH LUNGS
 MATURE FLUKES ARE FOUND IN THE BRONCHIOLES SURROUNDED BY CYSTIC LESIONS
 PARASITE EGGS ARE EITHER EXPECTORATED WITH SPUTUM OR SWALLOWED AND PASSED
TO THE OUTSIDE ENVIRONMENT WITH FACES
 THE LIFE CYCLE IS COMPLETED IN SNAILS AND FRESHWATER CRUSTACEANS.
88
AIIMS NOV 2015-QUESTIONS & ANSWERS WITH EXPLANATIONS-SOLVED BY
DR: MANJUNATH, MBBS, MD (MAMC NEW DELHI) DIRECTOR
DOCTORS ACADEMY DAVANAGERE & SHIMOGA
(www.doctorsacademydvg.com)
DR MANJUNATH, DOCTORS
ACADEMY, DAVANAGEREDR. MANJUNATH, DOCTORS ACADEMY
MORPHOLOGY  ADULT LUNG FLUKES, WHICH ARE 7-12 MM IN LENGTH, ARE
FOUND ENCAPSULATED IN THE LUNGS OF INFECTED PERSONS.
RARELY, FLUKES ARE FOUND ENCYSTED IN CNS (CEREBRAL
PARAGONIMIASIS) OR THE ABDOMINAL CAVITY.
 WHEN MATIRUG FLUKES LODGE IN LUNG TISSUES, THEY CAUSE
HAEMORRHAGE AND NECROSIS, RESULTING IN CYST
FORMATION
 THE ADJACENT LUNG PARENCHYMA SHOWS EVIDENCE OF
INFLAMMATORY INFILTRATION, PREDOMINANTLY BY
EOSINOPHILS.
 CYSTS USUALLY MEASURE 1-2 CM IN DIAMETER AND MAY
CONTAIN ONE OR TWO WORMS EACH
 OLDER CYSTS DEVELOP THICKENED WALLS, WHICH MAY
UNDER GO CALCIFICATION.
 DURING THE ACTIVE PHASE OF PARAGONIMIASIS, LUNG
TISSUES SURROUNDING PARASITE CYSTS MAY SHOW
EVIDENCE OF PNEUMONIA, BRONCHIECTASIS, AND FIBROSIS.
89
AIIMS NOV 2015-QUESTIONS & ANSWERS WITH EXPLANATIONS-SOLVED BY
DR: MANJUNATH, MBBS, MD (MAMC NEW DELHI) DIRECTOR
DOCTORS ACADEMY DAVANAGERE & SHIMOGA
(www.doctorsacademydvg.com)
DR MANJUNATH, DOCTORS
ACADEMY, DAVANAGEREDR. MANJUNATH, DOCTORS ACADEMY
DIAGNOSIS  PRODUCTIVE COUGH WITH BROWNISH SPUTUM OR FRANK
HEMOPTYSIS ASSOCIATED WITH PERIPHERAL-BLOOD
EOSINOPHILIA IS USUALLY THE PRESENTING FEATURE.
 CHEST EXAMINATION MAY SIGNS OF PLEURISY, BRONCHITIS OR
BRONCHIECTASIS MAY PREDOMINATE, RARELY LUNG ABSCESS.
 IMAGING OF LUNGS SHOWS PATCHY DENSITIES, CAVITIES,
PLEURAL EFFUSION, AND RING SHADOWS.
 PULMONARY PARAGONIMIASIS IS DIAGNOSED BY DETECTION
PARASITE OVA IN SPUTUM, LUNG BIOPSY AND/OR STOOLS.
 SEROLOGY IS OF CONSIDERABLE HELP IN EGG-NEGATIVE CASES
AND IN CEREBRAL PARAGONIMIASIS.
 THE DIFFERENTIAL DIAGNOSIS INCLUDES ACTIVE TUBERCULOSIS.
BACTERIAL LUNG ABSCESS, AND LUNG CARCINOMA.
TREATMENT THE DRUG OF CHOICE FOR TREATMENT IS PRAZIQUANTEL
90
AIIMS NOV 2015-QUESTIONS & ANSWERS WITH EXPLANATIONS-SOLVED BY
DR: MANJUNATH, MBBS, MD (MAMC NEW DELHI) DIRECTOR
DOCTORS ACADEMY DAVANAGERE & SHIMOGA
(www.doctorsacademydvg.com)
42.ABSENT UROBILINOGEN IN
URINE WITH ICTERUS
INDICATES?
A. PERIHEPATIC OBSTRUCTION
B. HEMOLYSIS
C. HEPATITIS
D. LIVER FAILURE
ANS: A. PERIHEPATIC
OBSTRUCTION
[REF: ROBBIN’S 9/830-840]
DR MANJUNATH, DOCTORS
ACADEMY, DAVANAGEREDR. MANJUNATH, DOCTORS ACADEMY 91
AIIMS NOV 2015-QUESTIONS & ANSWERS WITH EXPLANATIONS-SOLVED BY
DR: MANJUNATH, MBBS, MD (MAMC NEW DELHI) DIRECTOR
DOCTORS ACADEMY DAVANAGERE & SHIMOGA
(www.doctorsacademydvg.com)
EXPLANATION: UROBILIONGEN APPEARS IN URINE ONLY AFTER ENTROHEPATIC CIRCULATION, SO
ITS ABSECENCE WITH ICTERUS INDICATES POST HEPATIC JAUNDICE (PERIHEPATIC
OBSTRUCTION)
DR MANJUNATH, DOCTORS
ACADEMY, DAVANAGEREDR. MANJUNATH, DOCTORS ACADEMY
CONDITION SERUM BILIRUBIN URINE
UROBILINOGEN
URINE BILIRUBIN FECAL
UROBILINOGEN
NORMAL DIRECT 0.1-0.4
MG/DL
INDIRECT 0.2-07
0-4 MG/24H ABSENT 40-280 MG/24H
HEMOLYTIC
ANEMIA
↑ INDIRECT INCREASED ABSENT INCREASED
HEPATITIS ↑ DIRECT AND
INDIRET
DECREASED IF
MICROOBSTRUCTIO
N IS PRESENT
PRESENT IF
MICROOBSTRUCTIO
N
DECREASE
OBSTRUCTION
JAUNDICE
↑ DIRECT ABSENT PRESENT TRACE TO ABSENT
92
AIIMS NOV 2015-QUESTIONS & ANSWERS WITH EXPLANATIONS-SOLVED BY
DR: MANJUNATH, MBBS, MD (MAMC NEW DELHI) DIRECTOR
DOCTORS ACADEMY DAVANAGERE & SHIMOGA
(www.doctorsacademydvg.com)
43. FOR WHICH PROCEDURE IS THE FOLLOWING INSTRUMENT
USED?
A. BONE MARROW EXAMINATION
B. LIVER BIOPSY
C. PLEURAL BIOPSY
D. LUMBAR PUNCTURE
ANS: A. BONE MARROW EXAMINATION
[REF: DACIE PRACTICAL HEMATOLOGY]
DR MANJUNATH, DOCTORS
ACADEMY, DAVANAGEREDR. MANJUNATH, DOCTORS ACADEMY 93
AIIMS NOV 2015-QUESTIONS & ANSWERS WITH EXPLANATIONS-SOLVED BY
DR: MANJUNATH, MBBS, MD (MAMC NEW DELHI) DIRECTOR
DOCTORS ACADEMY DAVANAGERE & SHIMOGA
(www.doctorsacademydvg.com)
DR MANJUNATH, DOCTORS
ACADEMY, DAVANAGEREDR. MANJUNATH, DOCTORS ACADEMY 94
AIIMS NOV 2015-QUESTIONS & ANSWERS WITH EXPLANATIONS-SOLVED BY
DR: MANJUNATH, MBBS, MD (MAMC NEW DELHI) DIRECTOR
DOCTORS ACADEMY DAVANAGERE & SHIMOGA
(www.doctorsacademydvg.com)
DR MANJUNATH, DOCTORS
ACADEMY, DAVANAGEREDR. MANJUNATH, DOCTORS ACADEMY 95
AIIMS NOV 2015-QUESTIONS & ANSWERS WITH EXPLANATIONS-SOLVED BY
DR: MANJUNATH, MBBS, MD (MAMC NEW DELHI) DIRECTOR
DOCTORS ACADEMY DAVANAGERE & SHIMOGA
(www.doctorsacademydvg.com)
44. IN A PATIENT SUFFERING WITH SICKLE CELL ANEMIA,
ELETROPHORETIC MOBILITY OF HBS IN RELATION TO HBA
WILL BE?
A. RETARDED
B. ACCELERATED
C. SAME
D. WILL DEPEND UPON CONCENTRETATION OF HBS
ANS: A. RETARDED
[REF: COMPLETE REVIEW OF PATHOLOGY 1/343]
DR MANJUNATH, DOCTORS
ACADEMY, DAVANAGEREDR. MANJUNATH, DOCTORS ACADEMY 96
AIIMS NOV 2015-QUESTIONS & ANSWERS WITH EXPLANATIONS-SOLVED BY
DR: MANJUNATH, MBBS, MD (MAMC NEW DELHI) DIRECTOR
DOCTORS ACADEMY DAVANAGERE & SHIMOGA
(www.doctorsacademydvg.com)
45. WHICH OF THE FOLLOWING MECHANISM IS MAINLY INVOLVED IN
EPIGENETICS?
A. METHYLATION
B. ACETYLATION
C. DEAMINATION
D. PHOSPHORYLATION
ANS: A. METHYLATION
(The term epigenetics refers to heritable changes in gene expression
(active versus inactive genes) that does not involve changes to the
underlying DNA sequence; a change in phenotype without a change
in genotype.)
[REF: ROBBIN’S 9/ EP5]
DR MANJUNATH, DOCTORS
ACADEMY, DAVANAGEREDR. MANJUNATH, DOCTORS ACADEMY 97
AIIMS NOV 2015-QUESTIONS & ANSWERS WITH EXPLANATIONS-SOLVED BY
DR: MANJUNATH, MBBS, MD (MAMC NEW DELHI) DIRECTOR
DOCTORS ACADEMY DAVANAGERE & SHIMOGA
(www.doctorsacademydvg.com)
EXPLANATION
DR MANJUNATH, DOCTORS
ACADEMY, DAVANAGEREDR. MANJUNATH, DOCTORS ACADEMY
EPIGENETICS
DEFINITION HERITABLE CHANGES IN GENE EXPRESSION, NOT CAUSED BY ALTERATIONS
IN DNA SEQUENCE.
EPIGENETIC FACTORS HISTONES AND HISTONE MODIFYING FACTORS
o HISTONE METHYLATION
o HISTONE ACETLATION
o HISTONE PHOSPHORYLATION
98
AIIMS NOV 2015-QUESTIONS & ANSWERS WITH EXPLANATIONS-SOLVED BY
DR: MANJUNATH, MBBS, MD (MAMC NEW DELHI) DIRECTOR
DOCTORS ACADEMY DAVANAGERE & SHIMOGA
(www.doctorsacademydvg.com)
DR MANJUNATH, DOCTORS
ACADEMY, DAVANAGEREDR. MANJUNATH, DOCTORS ACADEMY
SIGNIFICANCE  EPIGENETIC DYSREGULATION→CENTRAL ROLE IN MALIGNANCY
 MANY OTHER DIEASES ARE ASSOCIATED WITH INHERITED OR ACQUIRED
EPIGENETIC ALTERATIONS. EG GENOMIC IMPRINTING IN QUIRED
EPIGENETIC ALTERATION. EG GENOMIC IMPRINTING IN PRADER WILLI
SYNDROME
 EPIGENETIC ALTERATIONS LIKE HISTONE ACETYLATION AND DNA
METHYLATION ARE REVERSIBLE AND ARE RESPONSIVE TO DRUGS; SO,
HDAC INHIBITORS AND DNA METHYLATION INHIBITORS ARE BEING
TESTED IN THE TREATMENT OF CANCER
DIAGNOSIS  SEQUENCING
 CHIP ON CHIP (MICROARRAY TECHNOLOGY)
 USING METHYLATION SPECIFIC PRIMERS IN POLYMERASE CHAIN
REACTION (PCR)
 BISULPHITE METHOD: BISULPHITE CONVERTS UNMETHYLATED CYTOSINE
TO URACIL, WHICH ACTS LIKE THYMINE IN DOWNSTREAM REACTIONS.
THE UNMETHYLATED (MODIFIED) DNA IS DETECTED BY SEQUENCE
ANALYSIS.
99
AIIMS NOVEMBER 2015-QUESTIONS & ANSWERS WITH EXPLANATIONS-SOLVED BY
DR: MANJUNATH, MBBS, MD (MAMC NEW DELHI) DIRECTOR
DOCTORS ACADEMY DAVANAGERE & SHIMOGA
(www.doctorsacademydvg.com)
PHARMACOLOGY
47. NEW ORAL DRUG APPROVED FOR
THE TREATMENT OF HEPATITIS C IS?
A. RIBAVIRINE
B. LEDIPASVIR
C. LAMIVUDINE
D. PEG INTERFERON
ANS: B. LEDIPASVIR (Ledipasvir (formerly GS-5885) is a drug for
the treatment of hepatitis C that was developed by Gilead
Sciences)
REF: HARRISON’S 19/E P2041-2047
DR MANJUNATH, DOCTORS
ACADEMY, DAVANAGEREDR. MANJUNATH, DOCTORS ACADEMY 100
AIIMS NOV 2015-QUESTIONS & ANSWERS WITH EXPLANATIONS-SOLVED BY
DR: MANJUNATH, MBBS, MD (MAMC NEW DELHI) DIRECTOR
DOCTORS ACADEMY DAVANAGERE & SHIMOGA
(www.doctorsacademydvg.com)
EXPLANATION:
DISCUSSING ANOUT THE PTIONS ONE BY ONE:
DR MANJUNATH, DOCTORS
ACADEMY, DAVANAGEREDR. MANJUNATH, DOCTORS ACADEMY
(A) RIBAVIRINE  IT IS A NUCLEOSIDE INHIBITOR
 USED FOR SEVERE RSV, CHRONIC HEPATITIS C INFECTION
 RIBAVIRIN SHOULD NERVE BE GIVEM AS MONOTHERAPY.
 SHOULD BE GIVEN IN COMBINATION WITH PEGINTERFERON ALFA-2A;
 DAILT DOSE IS 800-1200 MG ADMINISTERED ORALLY
 MOST IMPORTANT SIDE EFFECT IS HEMOLYSIS
 CONTRA-INDICATED IN PREGNANCY & HEMOGLOBINOPATHIES
(B) LEDIPASVIR  LEDIPASVIR (FORMERLY GS-5885) IS A RECENTLY DEVELOPED DRUG FOR THE
TREATMENT OF EPATITIS C
 IT IS A POTENT INHIBITOR OF HCV NS5A, A VIRAL PHOSPOHOPROTEIN THAT
PLAYS AN IMPORTANT ROLE IN VIRAL REPLICATION, ASSE,BLY, AND
SECRETION.
 SOFTOSBUVIR IS A NUCLEOTIDE ANALOG INHIBITOR OF HEPATITIS C VIRUS
NS5B POLYMERASE
 FIXED DOSE COMBINATION LADIPASVIR-SOFOSBUVIR IS FDA-APPROVED FOR
THE TREATMENT OF CHRONIC HEPATITIS GENOTYPE 1
 MOST COMMON ADVERSE EFFECTS ARE FATIGUE AND HEADACHE.
101
AIIMS NOV 2015-QUESTIONS & ANSWERS WITH EXPLANATIONS-SOLVED BY
DR: MANJUNATH, MBBS, MD (MAMC NEW DELHI) DIRECTOR
DOCTORS ACADEMY DAVANAGERE & SHIMOGA
(www.doctorsacademydvg.com): EXTRA EDGE NEW MCQS
DR MANJUNATH, DOCTORS
ACADEMY, DAVANAGEREDR. MANJUNATH, DOCTORS ACADEMY
© LAMIVUDINE  LAMIVUDINE, IS A NUCLEOSIDE ANALOG THAT INHIBITS HIV
REVERSE TRANSCRIPTASE AND HBC DNA POLYMERASE.
 FOLLOWING ORALADMINISTRATION. LAMIVUDINE IS
ABSORBED RAPIDLY WITH A BIOAVAILABILITY OF 80%
 LAMIVUDINE IS A WELL TOLERATED DRUG
 AMINOTRANFERASE RISES AFTER THERAPY MAY BE SEEN
(D) PEG INTERFERON  INTERFERONS (IFNS) ARE POTENT CYROKINES THAT POSSESS
ANTIVIRAL, IMMUNIMODULATING, AND ANTIPROLIFERATIVE
ACTIVITIES
 ORAL ADMINISTRATION DOES NOT RESULT IN DETECTABLE IFN
LEVEL IN SERUM, HENCE GIVEN SUBCUTANEOUS OR
INTRAMUSCULAR
 ATTACHMENT OF IFN PROTEINS TO LARGE, POLYETHYLENE
GLYCOL (PEG) MOLECULES SLOWS ABSORPTION, DECREASES
CLEARANCE, AND PROVIDES HIGHER & MORE PROLONGED SERUM
CONCENTRATIONS THAT ENABLE NCE-WEEKLY DOSING
 SIDE EFFECTS:ACUTE INFLUENZA B, DISCOMFORT AT THE
INJECTION SITE, AND LEUCOPENIA
 ONCE-A-WEEK ADMINISTERED LONG-ACTING PEG IFN IS MORE
EFFECTIVE THAN THE STANDARD IFN
102
AIIMS NOV 2015-QUESTIONS & ANSWERS WITH EXPLANATIONS-SOLVED BY
DR: MANJUNATH, MBBS, MD (MAMC NEW DELHI) DIRECTOR
DOCTORS ACADEMY DAVANAGERE & SHIMOGA
(www.doctorsacademydvg.com)
48. MECHANISM OF ACTION OF ASPIRIN FOR ANTIPLATELET
ACTION IS?
A. BLOCKS TXA2 SYNTHESIS
B. COX 2 INHIBITOR
C. LIPOXYGENASE INHIBITOR
D. PLATELET DYSFUNCTION
ANS: A. BLOCKS TXA2 SYNTHESIS
REF: H19 746-747
DR MANJUNATH, DOCTORS
ACADEMY, DAVANAGEREDR. MANJUNATH, DOCTORS ACADEMY 103
AIIMS NOV 2015-QUESTIONS & ANSWERS WITH EXPLANATIONS-SOLVED BY
DR: MANJUNATH, MBBS, MD (MAMC NEW DELHI) DIRECTOR
DOCTORS ACADEMY DAVANAGERE & SHIMOGA
(www.doctorsacademydvg.com)
EXPLANATION:
ASPIRIN
DR MANJUNATH, DOCTORS
ACADEMY, DAVANAGEREDR. MANJUNATH, DOCTORS ACADEMY
IMPORTANT INFO IT IS THE MOST WIDELY USED ANTIPLATELET AGENT
WORLDWIDE
MECHANISM OF ACTION  ASPIRIN PRODUCES ITS ANTITHROMBOTIC EFFECT BY
IRREVERSIBLY ACETYLATING & INHIBITING PLARELET
CYCLOOXY-GENASE (COX)-1, A CRITICAL ENZYME IN
THROMBOXANE A2 SYNTHESIS
 AT HIGH DOSES, ASPIRIN ALSO INHIBITS COX-2, AN
INDUCIBLE COX ISOFORM FOUND IN ENDOTHELIAL
CELLS AND INFLAMMATORY CELLS.
INDICATIONS  SECONDARY PREVENTION OF CARDIOVASCULAR EVENTS
IN PATIENTS WITH CORONARY ARTERY,
CEREBROVASCULAR OR PERIPHERAL VASCULAR DISEASE
 ASPIRIN PRODUCES A 25% REDUCTION IN THE RISK OF
CARDIOVASCULAR DEATH, MI, R STROKE
 ASPIRIN IS ALSO USED FOR PRIMARY PREVENTION IN
HIGH RISK PATIENTS
104
AIIMS NOV 2015-QUESTIONS & ANSWERS WITH EXPLANATIONS-SOLVED BY
DR: MANJUNATH, MBBS, MD (MAMC NEW DELHI) DIRECTOR
DOCTORS ACADEMY DAVANAGERE & SHIMOGA
(www.doctorsacademydvg.com)
DR MANJUNATH, DOCTORS
ACADEMY, DAVANAGEREDR. MANJUNATH, DOCTORS ACADEMY
DOSAGE 75-325 MG ONCE DAILY
ADVERSE EFFECTS SIDE EFFECTS OF ASPIRIN ARE DOSE-RELATED
MOST COMMON ARE GASTROINTESTINAL (DYSPEPSIA, EROSIVE GASTRITIS,
PEPIC ULCERS WITH BLLEDING AND PERFORATION)
105
AIIMS NOV 2015-QUESTIONS & ANSWERS WITH EXPLANATIONS-SOLVED BY
DR: MANJUNATH, MBBS, MD (MAMC NEW DELHI) DIRECTOR
DOCTORS ACADEMY DAVANAGERE & SHIMOGA
(www.doctorsacademydvg.com)
49. WHICH OF THE FOLLOWING IS
THE LATEST APPROVED DRUG
FOR TUBERCULOSIS?
A. ETHIONAMIDE
B. LEVOFLOXACIN
C. BEDAQUILINE
D. TIPRANAVIR
• ANS: C. BEDAQUILINE
• (Bedaquiline (trade name Sirturo,
code names TMC207 and R207910)
is a medication used to
treat tuberculosis)
DR MANJUNATH, DOCTORS
ACADEMY, DAVANAGEREDR. MANJUNATH, DOCTORS ACADEMY 106
AIIMS NOV 2015-QUESTIONS & ANSWERS WITH EXPLANATIONS-SOLVED BY
DR: MANJUNATH, MBBS, MD (MAMC NEW DELHI) DIRECTOR
DOCTORS ACADEMY DAVANAGERE & SHIMOGA
(www.doctorsacademydvg.com): EXTRA EDGE
EXPLANATION: SECOND-LINE DRUGS (HAVE LOWER EFFICACY & HIGHER TOXICITY, USED IN MDR &
XDR TB):
DR MANJUNATH, DOCTORS
ACADEMY, DAVANAGEREDR. MANJUNATH, DOCTORS ACADEMY
FLUOROQUINOLONES
(LEVOFLOXACIN & MOXIFLAXACIN
MOST EFFECTIVE)
INHIBIT MYCOBACTERIAL DNA GYRASE & TOPOISOMERASE IV,
PREVENTING CELL REPLICATION AND PROTEIN SYNTHESIS, AND ARE
BACTERICIDAL.
INJECTABLE AMINOGLYCONSIDES
(KANAMYCIN, AMIKACIN &
STREPTOMYCIN)
EXERT MYCOBACTERICIDAL ACTIVITY BY BINDING TO THE 16S
RIBOSOMAL SUBUNIT.
ADVERSE EFFECTS OTOTXICITY, NEPHROTOXICITY,A AND
NEUROTOXICITY.
CAPREOMYCIN USEFUL WHEN ADDITIONAL RESISTANCE TO AMINOGLYCOSIDES IS
DOCUMENTED. CAPREOMYCIN IS ADMINISTERED BY THE IM ROUTE;
ADVERSE EFFECTS; HYPOKALEMIA, HYPOMAGNESEMIA, OTO-AND
RENAL TOXICITY.
ETHIONAMIDE AND
PROTHIONAMIDE
(BACTERIOSTATIC)
INHIBITS INHA GENE PRODUCT ENOYL-ACYL CARRIER PROTEIN (ACP)
REDUCTASE, INVOLVED IN MYCOLIC ACID SYNTHESIS.
ADVERSE EFFECTS: SEVERE GI REACTIONS (INCLUDING ABDOMINAL
PAIN, NAUSEA, VOMITING), NEUROLOGIC SIDE EFFECTS, REVERSIBLE
HEPATITIS, HYPERSENSITIVITY REACTIONS, AND HYPOTHYROIDISM.
107
AIIMS NOV 2015-QUESTIONS & ANSWERS WITH EXPLANATIONS-SOLVED BY
DR: MANJUNATH, MBBS, MD (MAMC NEW DELHI) DIRECTOR
DOCTORS ACADEMY DAVANAGERE & SHIMOGA
(www.doctorsacademydvg.com)
DR MANJUNATH, DOCTORS
ACADEMY, DAVANAGEREDR. MANJUNATH, DOCTORS ACADEMY
CYCLOSERINE AND TERIZIDONE
(THERIZIDONE)
PREVENTS CELL WALL SYNTHESIS & INHIBITS THE ACTION OF
ALANINE RACEMASE, INVOLVED IN PRODUCTION OF
PEPTIDOGLYCANS
IT IS WELL ABSORBED AFTER ORAL ADMINISTRATION AND IS
WELL DISTRIBUTED AFTER ORAL ADMINISTRATION AND IS WELL
DISTRIBUTED THROUGHOUT BODY FLUID, INCL CSF
SIDE EFFECTS INCLUDE SEIZURES, PSYCHOSIS, PERIPHERAL
NEUROPATHY, HEADACHE, SOMNOLENCE & ALLERGIC
REACTIONS.
PAS (PARA-AMINOSALICYLIC ACID) IT IS AN ORAL AGENT USED IN TREATMENT OF MDR-& XDR-TB.
ITS BACTERIOSTATIC ACTIVITY IS DUE TO INHIBITION OF FOLATE
SYNTHESIS AND OF IRON UPTAKE.
ADVERSE EFFECTS INCLUDE NAUSEA, VOMITING,
DIARRHEA, HEMOLYSIS IN C6PD DEFICIENCY.
108
AIIMS NOV 2015-QUESTIONS & ANSWERS WITH EXPLANATIONS-SOLVED BY
DR: MANJUNATH, MBBS, MD (MAMC NEW DELHI) DIRECTOR
DOCTORS ACADEMY DAVANAGERE & SHIMOGA
(WWW.DOCTORSACADEMYDVG.COM)
50. DRUG USED FOR DETOXIFICATION IN CHRONIC
ALCOHOLISM ARE ALL EXCEPT?
A. NALTREXONE
B. FLUMAZENIL
C. DISULFIRAM
D. ACAMPROSATE
ANS: B. FLUMAZENIL
REF:HARRISON 19/E P2727
DR MANJUNATH, DOCTORS
ACADEMY, DAVANAGEREDR. MANJUNATH, DOCTORS ACADEMY 109
AIIMS NOV 2015-QUESTIONS & ANSWERS WITH EXPLANATIONS-SOLVED BY
DR: MANJUNATH, MBBS, MD (MAMC NEW DELHI) DIRECTOR
DOCTORS ACADEMY DAVANAGERE & SHIMOGA
(www.doctorsacademydvg.com)
EXPLANATION:
MEDICATIONS FOR REHABILITATION IN CHRONIC ALCOHOLISM
DR MANJUNATH, DOCTORS
ACADEMY, DAVANAGEREDR. MANJUNATH, DOCTORS ACADEMY
NALTREXONE  AN OPIOID ANTAGONIST
 MAY BE TAKEN DAILY ORALLY, OR AS MONTHLY INJECTION
 SHORTENS SUB-SEQUENT RELAPSES
 BY BLOCKING OPIOID RECEPTORS, NALTREXONE DECREASES ACTIVITY IN
THE DOPAMINE-RICH VENTRAL TEGMENTAL REWARD SYSTEM &
DECREASES THE FEELING OF PLEASURE IF ALCOHOL IS IMBIBED.
ACAMPROSATE  IT INHIBITS NMDA RECEPTORS, DECREASING MILD SYMPTOMS OF
PROTRACTED WITHDRAWAL.
 DOSE~2 G.D DIVIDED INTO THREE ORAL DOSES
DISULFIRAM  IT IS AN ALDH INHIBITOR, USED AT DOSES OF 250 MG/D
 IT PRODUCES VOMITING & AUTONOMIC INSTABILITY IN THE
PRESENCE OF ALCOHOL AS A RESULT OF RAPIDLY RISING BLOOD
LEVELS OF ACETALDEHYDE.
 THIS REACTION CAN BE DANGEROUS, ESPECIALLY FOR ATIENTS
WITH HEART DISEASE, STROKE, DIABETES MELLITUS, OR
HYPERTENSION
 IT ALSO CARRIES POTENTIAL RISK OF DEPRESSION, PSYCHOTIC
SYMPTOMS, PERIPHERAL NEUROPAHY, AND LIVER DAMAGE
110
AIIMS NOV 2015-QUESTIONS & ANSWERS WITH EXPLANATIONS-SOLVED BY
DR: MANJUNATH, MBBS, MD (MAMC NEW DELHI) DIRECTOR
DOCTORS ACADEMY DAVANAGERE & SHIMOGA
(www.doctorsacademydvg.com)
DR MANJUNATH, DOCTORS
ACADEMY, DAVANAGEREDR. MANJUNATH, DOCTORS ACADEMY
OTHER DRUGS UNDER INVESTIGATION  NALMEFENE (OPIOID ANTAGONIST)
 VARENICLINE (NICOTINIC RECEPTOR
AGONIST)
 ONDANSETRON (SEROTONIN ANTAGONIST),
 PRAZOSIN (Α-ADRENERGIC AGONIST)
 BACLOFEN (GABA B RECEPTOR AGONIST),
 TOPIRAMATE (ANTICONVULSANT)
111
AIIMS NOV 2015-QUESTIONS & ANSWERS WITH EXPLANATIONS-SOLVED BY
DR: MANJUNATH, MBBS, MD (MAMC NEW DELHI) DIRECTOR
DOCTORS ACADEMY DAVANAGERE & SHIMOGA
(WWW.DOCTORSACADEMYDVG.COM)
51. ETANERCEPT IS?
A. COX 2 INHIBITOR
B. TNF INHIBITOR
C. IL 6 INHIBITOR
D. IFN GAMMA INHIBITOR
ANS: B. TNF INHIBITOR
REF: HARRISON 19/E P272E30
DR MANJUNATH, DOCTORS
ACADEMY, DAVANAGEREDR. MANJUNATH, DOCTORS ACADEMY 112
AIIMS NOV 2015-QUESTIONS & ANSWERS WITH EXPLANATIONS-SOLVED BY
DR: MANJUNATH, MBBS, MD (MAMC NEW DELHI) DIRECTOR
DOCTORS ACADEMY DAVANAGERE & SHIMOGA
(www.doctorsacademydvg.com)
EXPLANATION
INDICATIONS OF ETANERCEPT:-
DR MANJUNATH, DOCTORS
ACADEMY, DAVANAGEREDR. MANJUNATH, DOCTORS ACADEMY
• ANKYLOSING SPONDYLITIS
• ADULT RHEUMATOID ARTHRITIS
• PSORIATIC ARTHRITIS
• PLAQUE PSORIASIS
113
AIIMS NOV 2015-QUESTIONS & ANSWERS WITH EXPLANATIONS-SOLVED BY
DR: MANJUNATH, MBBS, MD (MAMC NEW DELHI) DIRECTOR
DOCTORS ACADEMY DAVANAGERE & SHIMOGA
(www.doctorsacademydvg.com)
52. WHICH OF THE FOLLOWING
ADVICE WILL YOU GIVE TO A
PATIENT WHEN HE IS GIVEN
BISPHOSPHONATES?
A. TAKE TABLET BEFORE FOOD WITH FULL
GLASS OF WATER
B. DISCONTINUE IF GASTRITIS PERSISTS
C. TAKE TABLET AFTER FOOD WITH FULL
GLASS OF WATER
D. WITHDRAW THE DRUG WHEN BONE PAIN
OCCURS
ANS: A. TAKE TABLET BEFORE FOOD WITH FULL GLASS
OF WATER
REF: HARRISON 19/E P2499
DR MANJUNATH, DOCTORS
ACADEMY, DAVANAGEREDR. MANJUNATH, DOCTORS ACADEMY 114
AIIMS NOV 2015-QUESTIONS & ANSWERS WITH EXPLANATIONS-SOLVED BY
DR: MANJUNATH, MBBS, MD (MAMC NEW DELHI) DIRECTOR
DOCTORS ACADEMY DAVANAGERE & SHIMOGA
(www.doctorsacademydvg.com)
EXPLANATION: BISPHOSPHONATES
DR MANJUNATH, DOCTORS
ACADEMY, DAVANAGEREDR. MANJUNATH, DOCTORS ACADEMY
NAME OF DRUGS ALENDRONATE, RISEDRONATE, IBANDRONATE,
AND ZOLEDRONIC ACID
INDICATIONS  PREVENTION AND TREATMENT OF POST-
MENOPAUSAL OSTEOPOROSIS.
 TREATMENT OF STEROID-INDUCED
OSTEOPOROSIS,
 PREVENTION OF STEROID-INDUCED
OSTEOPOROSIS
 PAGET’S DISEASE
 HYPERCALCEMIA OF MALIGNANCY
MECHANISM OF ACTION  INHIBIT OSTEOCLAST MEDIATED BONE
RESORPTION
 INCREASES APOPTOSIS OF OSTEOCLASTS
 INHIBIT IL6, THEREBY SUPPRESSES
DIFFERENTIATION OF OSTEOCLAST PRECURSOR
TO MATURE OSTEOCLAST
115
AIIMS NOV 2015-QUESTIONS & ANSWERS WITH EXPLANATIONS-SOLVED BY
DR: MANJUNATH, MBBS, MD (MAMC NEW DELHI) DIRECTOR
DOCTORS ACADEMY DAVANAGERE & SHIMOGA
(www.doctorsacademydvg.com)
DR MANJUNATH, DOCTORS
ACADEMY, DAVANAGEREDR. MANJUNATH, DOCTORS ACADEMY
PROCESS OF ADMINISTRATION  ALENDRONATE SHOULD BE GIVEN WITH A FULL
GLASS OF WATER BEFORE BREAKFAST, BECAUSE
BISPHOSPHONATES ARE POORLY ABSORBED.
 IT IS RECOMMENDED THAT PATIENTS REMAIN
UPRIGHT FOR AT LEAST 30 MIN AFTER TAKING THE
MEDICATION TO AVOID ESOPHAGEAL IRRITATION.
CONTRAINDICATION  BECAUSE OF THE POTENTIAL FOR ESOPHAGEAL
IRRITATION, ALENDRONATE IS CONTRAINDICATED
IN PATIENTS WITH HAVE STRICTURE OR
INADEQUATE EMPTYING OF THE ESOPHAGUS.
116
AIIMS NOV 2015-QUESTIONS & ANSWERS WITH EXPLANATIONS-SOLVED BY
DR: MANJUNATH, MBBS, MD (MAMC NEW DELHI) DIRECTOR
DOCTORS ACADEMY DAVANAGERE & SHIMOGA
(www.doctorsacademydvg.com)
DR MANJUNATH, DOCTORS
ACADEMY, DAVANAGEREDR. MANJUNATH, DOCTORS ACADEMY 117
AIIMS NOV 2015-QUESTIONS & ANSWERS WITH EXPLANATIONS-SOLVED BY
DR: MANJUNATH, MBBS, MD (MAMC NEW DELHI) DIRECTOR
DOCTORS ACADEMY DAVANAGERE & SHIMOGA
(www.doctorsacademydvg.com)
53. DRUG OF CHOICE IN PREGNANCY
INDUCED HYPERTENSION?
A. ACE INHIBITOR
B. ATENOLOL
C. ALPHA METHYL DOPA
D. NITROPRUSSIDE
ANS: C. ALPHA METHYL DOPA
REF:HYPERTENSION IN PREGNANCY (REPORT OF THE ACOG TASK FORCE
ON HYPERTENSION IN PREGNANCY) OBSTETRICS & GYNECOLOGY
VOL. 122, NO. 5 NOVEMBER 2013
DR MANJUNATH, DOCTORS
ACADEMY, DAVANAGEREDR. MANJUNATH, DOCTORS ACADEMY 118
AIIMS NOV 2015-QUESTIONS & ANSWERS WITH EXPLANATIONS-SOLVED BY
DR: MANJUNATH, MBBS, MD (MAMC NEW DELHI) DIRECTOR
DOCTORS ACADEMY DAVANAGERE & SHIMOGA
(WWW.DOCTORSACADEMYDVG.COM)
55. WHICH OF THE FOLLOWING IS UREIDOPENICILLIN?
A. PENICILLIN V
B. METHICILLIN
C. PIPERCILLIN
D. CLAVULANIC ACID
ANS: C. PIPERCILLIN
DR MANJUNATH, DOCTORS
ACADEMY, DAVANAGEREDR. MANJUNATH, DOCTORS ACADEMY 119
AIIMS NOV 2015-QUESTIONS & ANSWERS WITH EXPLANATIONS-SOLVED BY
DR: MANJUNATH, MBBS, MD (MAMC NEW DELHI) DIRECTOR
DOCTORS ACADEMY DAVANAGERE & SHIMOGA
(WWW.DOCTORSACADEMYDVG.COM)
56. WHILE PILOCARPINE ACTS ON SPHINCTER PAPILLAE. WHICH
OF THE FOLLOWING HAS ANALOGOUS EFFECT ON DILATOR
PUPILLAE?
A. EPINEPHRINE
B. TIMOLOL
C. COCAINE
D. LATANOPROST
ANS: A. EPINEPHRINE
(REF: K.D. TRIPATHI)
DR MANJUNATH, DOCTORS
ACADEMY, DAVANAGEREDR. MANJUNATH, DOCTORS ACADEMY 120
AIIMS NOV 2015-QUESTIONS & ANSWERS WITH EXPLANATIONS-SOLVED BY
DR: MANJUNATH, MBBS, MD (MAMC NEW DELHI) DIRECTOR
DOCTORS ACADEMY DAVANAGERE & SHIMOGA
(www.doctorsacademydvg.com)
EXPLANATION:
PUPILLARY MYDRIASIS
DR MANJUNATH, DOCTORS
ACADEMY, DAVANAGEREDR. MANJUNATH, DOCTORS ACADEMY
THIS IS CAUSED BY DRUGS THAT ACTIVELY DILATE THE PUPIL (ACTING ON THE
DILATOR PAPILLA) ON PRESENT THE ACTIVE CONSTRICTION OF THE PPIL
(PREVENTING ACTION ON THE SPHINCTER PAPILLA).
THUS, DRUGS WHICH DILATE THE PUIL BY ACTIVELY ACTING ON THE SYMPATHETIC
INNERVATIONS OF THE DILATOR PAPILLA ARE CALLED SYMPATHOMIMETIC DRUGS
AND DRUGS WHICH RESULT IN DILATATION OF THE PUPIL BY PREVENTING THE
PARASYMPATHETIC ACTION OF THE SPHINCTER ARE CALLED PARASYMPATHOLYRIC
DRUGS.
121
AIIMS NOV 2015-QUESTIONS & ANSWERS WITH EXPLANATIONS-SOLVED BY
DR: MANJUNATH, MBBS, MD (MAMC NEW DELHI) DIRECTOR
DOCTORS ACADEMY DAVANAGERE & SHIMOGA
(WWW.DOCTORSACADEMYDVG.COM)
57. PHASE 1 TRIAL IS DONE FOR?
A. HUMAN PHARMACOLOGY & SAFETY
B. THERAPEUTIC EXPLORATORY
C. MICRODOSING
D. PHARMACOKINETICS
ANS: A
(REF: K. D TRIPATHI 7/E P63-64)
DR MANJUNATH, DOCTORS
ACADEMY, DAVANAGEREDR. MANJUNATH, DOCTORS ACADEMY 122
AIIMS NOV 2015-QUESTIONS & ANSWERS WITH EXPLANATIONS-SOLVED BY
DR: MANJUNATH, MBBS, MD (MAMC NEW DELHI) DIRECTOR
DOCTORS ACADEMY DAVANAGERE & SHIMOGA
(www.doctorsacademydvg.com)
EXPLANATION
DR MANJUNATH, DOCTORS
ACADEMY, DAVANAGEREDR. MANJUNATH, DOCTORS ACADEMY
PHASE NAME CONDUCRED ON PURPOSE
I HUMAN PHARMACOLOGY AND
SAFETY
HEALTH VOLUNTEERS
(20-100)
 SAFETY AND TOLERABILITY
 TO KNOW MAIMUM TOLERABLE
DOSES (MTD)
II THERAPEUTIC EXPLORATORY 100-150
PATIENTS
 TO ESTABLISH THERAPEUTIC
EFFICACY
 DOSE RANGING AND CEILING
EFFECT
III THERAPEUTIC CINFIRMATORY UPTO 5000 PATIENTS FROM
SEVERAL CENTRE
 TO CONFIRM THERAPEUTIC
EFFICACY
IV POST MARKETING SURVEILI
LUNCH
TREATED BY PRACTICING
PHYSICIANS
 TO KNOW RARE AND LONG
TERM ADVERSE EFFECTS
 SPECIAL GROUPS LIKE
CHILDREN, PREGNANCY ETC
CAN BE TESTED
0
(ZERO)
MICRODOSING STUDIES HEALTHY VOLUNTEERS
(SMALL NUMBER)
 VERY LOW DOSE 1/100 OF
HUMAN DOES; MAX 100 µG
123
AIIMS NOV 2015-QUESTIONS & ANSWERS WITH EXPLANATIONS-SOLVED BY
DR: MANJUNATH, MBBS, MD (MAMC NEW DELHI) DIRECTOR
DOCTORS ACADEMY DAVANAGERE & SHIMOGA
(WWW.DOCTORSACADEMYDVG.COM)
58. WHICH ANTIDOTE IS REQUIRED FOR FIBRINOLYTIC
THERAPY?
A. tPA
B. PROTAMINE SULFATE
C. STREPTOKINASE
D. E-AMINO CAPROIC ACID
ANS: D. E-AMINO CAPROIC ACID
REF: K. D TRIPATHI 6/E P608-610
DR MANJUNATH, DOCTORS
ACADEMY, DAVANAGEREDR. MANJUNATH, DOCTORS ACADEMY 124
AIIMS NOV 2015-QUESTIONS & ANSWERS WITH EXPLANATIONS-SOLVED BY
DR: MANJUNATH, MBBS, MD (MAMC NEW DELHI) DIRECTOR
DOCTORS ACADEMY DAVANAGERE & SHIMOGA
(www.doctorsacademydvg.com)
EXPLANATION
DISCUSSING THE OPTIONS ONE BY ONE,
DR MANJUNATH, DOCTORS
ACADEMY, DAVANAGEREDR. MANJUNATH, DOCTORS ACADEMY
(A) EACA EPSILON AMINO CAPROIC ACID (EACA) &
TRANEXAMIC ACID ARE SPECIFIC ANTIDOTES OF
FIBRINOLYTIC THERAPY
(B) PROTAMINE SULFATE IT IS AN ANTIDOTE OF HEPARIN
(C ) STREPTOKINASE IT FORMS A COMPLEX WITH PLASMINOGEN &
EXPOSES ITS ACTIVE SITE. THIS ALTERED
PLASMINOGEN ACTS LIKE TPA & ACTIVATES OTHER
PLASMINOGEN MOLECULES TO PLASMIN THUS,
STREPTOKINASE ACTS AS A FIBRINOLYTIC
(D) TPA TISSUE PLASMINOGEN ACTIVATOR SELECTIVELY
ACTIVATES PLASMINOGEN THAT IS BOUND TO
FIBRIN (IN THE THROMBUS)
125
AIIMS NOV 2015-QUESTIONS & ANSWERS WITH EXPLANATIONS-SOLVED BY
DR: MANJUNATH, MBBS, MD (MAMC NEW DELHI) DIRECTOR
DOCTORS ACADEMY DAVANAGERE & SHIMOGA
(WWW.DOCTORSACADEMYDVG.COM)
59. WHICH ANTICHOLINERGIC IS
EXCLUSIVELY USED IN PRE-
ANAESTHETIC MEDICATION?
A. ATROPINE
B. PROMETHAZINE
C. GLYCOPYRROLATE
D. SCOPOLAMINE
ANS: C. GLYCOPYRROLATE (TO DECREASE
SECRETIONS)
REF: K. D TRIPATHI 7/E P110-112
DR MANJUNATH, DOCTORS
ACADEMY, DAVANAGEREDR. MANJUNATH, DOCTORS ACADEMY 126
AIIMS NOV 2015-QUESTIONS & ANSWERS WITH EXPLANATIONS-SOLVED BY
DR: MANJUNATH, MBBS, MD (MAMC NEW DELHI) DIRECTOR
DOCTORS ACADEMY DAVANAGERE & SHIMOGA
(WWW.DOCTORSACADEMYDVG.COM)
60. DRUGS SOLD ONLY ON
PRESCRIPTION OF
REGISTERED MEDICAL
PRACTITIONER BE LONGS
TO WHICH SCHEDULE?
A) H
B) X
C) J
D) P
ANS: A) H
(REF: TEXTBOOK OF FORENSIC
MEDICINE AND TOXICOLOGY BY
N.G.RAO P356)
DR MANJUNATH, DOCTORS
ACADEMY, DAVANAGEREDR. MANJUNATH, DOCTORS ACADEMY 127
AIIMS NOV 2015-QUESTIONS & ANSWERS WITH EXPLANATIONS-SOLVED BY
DR: MANJUNATH, MBBS, MD (MAMC NEW DELHI) DIRECTOR
DOCTORS ACADEMY DAVANAGERE & SHIMOGA
(www.doctorsacademydvg.com)
EXPLANATION
THE DRUGS AND COSMETIC RULES 1945
DR MANJUNATH, DOCTORS
ACADEMY, DAVANAGEREDR. MANJUNATH, DOCTORS ACADEMY
SCHEDULE DESCRIPTION
C BIOLOGICAL AND SPECIAL PRODUCTS
E LIST OF POISONS
F VACCINES AND SERA
G HORMONE PREPARATION;
H DRUGS (POISON TO BE SOLD ONLY ON THE PRESCRIPTION OF A REGISTERED MEDICAL
PRACTITIONER);
J LIST OF DISEASES FOR THE CURE OF WHICH NO DRUG SHOULD BE ADVERTISED
L ANTIBIOTICS, ANTIHISTAMINICS AND OTHER RECENT CHEMOTHERAPEUTIC AGENTS
128
AIIMS NOV 2015-QUESTIONS & ANSWERS WITH EXPLANATIONS-SOLVED BY
DR: MANJUNATH, MBBS, MD (MAMC NEW DELHI) DIRECTOR
DOCTORS ACADEMY DAVANAGERE & SHIMOGA
(WWW.DOCTORSACADEMYDVG.COM)
61. A PATIENT WITH EPILEPTIC DISORDER PRESENTED WITH
CONCENTRIC VISUAL FIELD DEFECTS. WHICH DRUG WAS HE
TAKING ?
A. VALPROATE
B. ETHOSUXIMIDE
C. PHENOBARBITOL
D. VIGABATRIN
ANS: D. VIGABATRIN
(REF: NELSON 20/E P2846)
DR MANJUNATH, DOCTORS
ACADEMY, DAVANAGEREDR. MANJUNATH, DOCTORS ACADEMY 129
AIIMS NOV 2015-QUESTIONS & ANSWERS WITH EXPLANATIONS-SOLVED BY
DR: MANJUNATH, MBBS, MD (MAMC NEW DELHI) DIRECTOR
DOCTORS ACADEMY DAVANAGERE & SHIMOGA
(www.doctorsacademydvg.com)
EXPLANATION
DR MANJUNATH, DOCTORS
ACADEMY, DAVANAGEREDR. MANJUNATH, DOCTORS ACADEMY
ANTIEPILETIC DRUG SIDE EFFECTS
ACETAZOLAMIDE MINOR: DIZZINESS, POLYURIA, ELECTROLYTE IMBALANCE SERIOUS: STEVENTS-
JOHNSON SYNDRINE
BENZODIAZEPINES MINOR: DOES-RELATED NEUROTOXICITY (DROWSINESS, SEDATION, ATAXIA),
HYPERACTIVITY, DROOLING INCREASED SECRETIONS SERIOUS APNEA
CARBAMAZEPINE NUISANCE: TICS, TRANSIENT LEUCOPENIA, HYPONATREMIA, WEIGHT GAIN,
NAUSEA, DIZZINESS
SERIOUS: STEVENS-JOHNSON SYNDROME, AGRANULOCYTSIS, APLASTIC ANEMIA,
LIVER TOXICITY
GABAPENTIN IN CHILDREN: ACUTE ONSET OF AGGRESSION, HYPERACTIVITY IN ADULTS:
EUPHORIA AND BEHAVIOURAL DISINHIITION, WEIGHT GAIN
LAMOTRINGINE MINOR: HEADACHE, ATAXIA, DIZZINESS, TREMOR, SERIOUS, STEVENS-JOHNSON
SYNDROME, RARELY LIVE TOXICITY
LEVETIRACETAM CNS ADVERSE EVENTS: SOMNOLENCE ASTHENIA DIZZINESS, BUTUSUALLY LESS
THAN OTHER AEDS
IN ADULTS: BEHAVIOURAL SYMPTOMS ARE COMMON, DEPRESSION
130
AIIMS NOV 2015-QUESTIONS & ANSWERS WITH EXPLANATIONS-SOLVED BY
DR: MANJUNATH, MBBS, MD (MAMC NEW DELHI) DIRECTOR
DOCTORS ACADEMY DAVANAGERE & SHIMOGA
(www.doctorsacademydvg.com)
DR MANJUNATH, DOCTORS
ACADEMY, DAVANAGEREDR. MANJUNATH, DOCTORS ACADEMY
PHENOBARBITONE MINOR: NEUROTOXICITY INSOMNIA, HYPERACTIVITY, SIGNS OF DISTRACTIBILITY,
FLUCTUATION OF MOOD, AGGRESSIVE OUTBURSTS
SERIOS:LIVER TOXICITY, STEVENS-JOHNSON SYNDROME
PHENYTOIN AND OTHER
HYDANTOINS
MINOR: GINGIVAL HYPERPLASIA, COARSENING OF FACIES, HIRSUTISM,
CEREBELLOVESTIBULAR SYMPTOMS (NYSTAGMUS AND ATAXIA)
SERIOUS: STEVENS-JOHNSON SYNDROME LIVE TOXICITY
PREGABALIN MINOR: DIZZINESS, PERIPHERAL EDEMA, BLURED VISION WEIGHT GAIN,
THROMBOCYTOPENIA
SERIOUS: HYPERSENSITIVITY REACTIONS, RHABDOMYOLYSIS
TOPIRAMATE MINOR: CONGNITIVE DYSFUNCTION, WEIGHT LOSS, RENAL CALCULI, HYPOHIDROSIS,
FEVER
SERIOUS: PRECIPITATION OF GLAUCOMA
VALPROCIC ACID MINOR: WEIGHT GAIN; HYPERAMMONEMIA TREMOR, ALOPECIA, MENSTRUAL
IRREGULARITIES
SERIOUS: HEPATIC AND PANCREATIC TOXICITY
VIGABATRIN MINOR: HYPERACTIVITY
SERIOUS: IRREVERSIBLE VISUAL FIELD DEFICTS, RETINOPATHY THAT REQUIRES
FREQUENT OPHTHALMOGIC EVALUATIONS AND FOLLOW UP
131
AIIMS NOV 2015-QUESTIONS & ANSWERS WITH EXPLANATIONS-SOLVED BY
DR: MANJUNATH, MBBS, MD (MAMC NEW DELHI) DIRECTOR
DOCTORS ACADEMY DAVANAGERE & SHIMOGA
(WWW.DOCTORSACADEMYDVG.COM)
62. DRUG WHOSE CONCENTRATION INCREASES AFTER A FATTY
MEAL ARE ALL EXCEPT?
A. AMPHOTERICIN B
B. AMPICILLIN
C. PPI
D. NIMESULIDE
ANS: C. PPI
(REF: NUTRITION IN PEDIATRICS /E P196)
DR MANJUNATH, DOCTORS
ACADEMY, DAVANAGEREDR. MANJUNATH, DOCTORS ACADEMY 132
AIIMS NOV 2015-QUESTIONS & ANSWERS WITH EXPLANATIONS-SOLVED BY
DR: MANJUNATH, MBBS, MD (MAMC NEW DELHI) DIRECTOR
DOCTORS ACADEMY DAVANAGERE & SHIMOGA
(www.doctorsacademydvg.com)
EXPLANATION
DR MANJUNATH, DOCTORS
ACADEMY, DAVANAGEREDR. MANJUNATH, DOCTORS ACADEMY
DRUG ABRORPTION
REDUCED/DELAYED BY FOOD
DRUG ABSORPTION INCREASED BY FOOD
AMPICILLIN
ASPIRIN
ATENOLOL
AZITHROMYCIN
CAPTOPRIL
CEFACLOR
CEFIXIME
CEPHALEXIN
CIPROFLOXACIN
DIDANOSINE
ATOVAQUONE
CARBAMAZEPINE
CHLOROTHIAZIDE
CEFUROXIME
CLOFAZIMINE
DIAZEPAM
ERYTHROMYCIN ESTOLATE
GANCICLOVIR
HYDROCHLOROTHIAZIDE
ITRACONAZOLE
133
AIIMS NOV 2015-QUESTIONS & ANSWERS WITH EXPLANATIONS-SOLVED BY
DR: MANJUNATH, MBBS, MD (MAMC NEW DELHI) DIRECTOR
DOCTORS ACADEMY DAVANAGERE & SHIMOGA
(www.doctorsacademydvg.com)
DR MANJUNATH, DOCTORS
ACADEMY, DAVANAGEREDR. MANJUNATH, DOCTORS ACADEMY
INDINARIR
ISONIAZID
LORATIDINE
NAFICILLIN
PENICILLIN G OR V
PHENOBARBITAL
PHENYTOIN
RIFAMPIN
SUCRALFATE
TERACYCLINE
DOXYCYCLINE
KETOCONAZOLE
LITHIUM
LOVASTATIN
METHYLPHENIDATE
METOPROLOL
NELFINAVIR
NITROFURANTOIN
PROPRANOLOL
PROPOXYPHENE
RITONAVIR
SAQUINAVIR
SPIRONOLACTONE
HYDRALAZINE
134
AIIMS NOV 2015-QUESTIONS & ANSWERS WITH EXPLANATIONS-SOLVED BY
DR: MANJUNATH, MBBS, MD (MAMC NEW DELHI) DIRECTOR
DOCTORS ACADEMY DAVANAGERE & SHIMOGA
(WWW.DOCTORSACADEMYDVG.COM)
63. ALL OF THE FOLLOWING INHIBITS
NUCLEIC ACID SYNTHESIS EXCEPT?
A. QUINOLONES
B. NALIDIXIC ACID
C. LINEZOLID
D. RIFAMPICIN
ANS: C. LINEZOLID (LINEZOLID INHIBTS THE PROTEIN
SYNTHESIS)
(REF: HARRIOSN 19/E P931)
DR MANJUNATH, DOCTORS
ACADEMY, DAVANAGEREDR. MANJUNATH, DOCTORS ACADEMY 135
AIIMS NOV 2015-QUESTIONS & ANSWERS WITH EXPLANATIONS-SOLVED BY
DR: MANJUNATH, MBBS, MD (MAMC NEW DELHI) DIRECTOR
DOCTORS ACADEMY DAVANAGERE & SHIMOGA
(www.doctorsacademydvg.com)
EXPLANATION
DR MANJUNATH, DOCTORS
ACADEMY, DAVANAGEREDR. MANJUNATH, DOCTORS ACADEMY
ANTIBACTERIAL AGENT MAJOR CELLULAR TARGET MECHANISM OF ACTION
Β-LACTAMS (PENICILINS
CEPHALOSPORINS)
CELL WALL INHIBIT CELL-WALL CROSS-LINKING
VANCOMYCIN CELL WALL INTERTERES WITH ADDITION OF NEW
CELL WALL SUBUNITS
(MURAMY PENTAPEPTIDES)
BACITRACIN CELL WALL PREVENTS ADDITION OF CELL-WALL-
SUBUNITS BY INHIBITING RECYCLING OF
MEMBRANE LIPID CARRIER
MACROLIDES (ERYTHROMYCIN) PROTEIN SYNTHESIS BIND TO 50S RIBOSOMAL SUBUNIT
LINCOSAMIDES (CLINDAMYCIN) PROTEIN SYNTHESIS BIND TO 50S RIBOSOMAL SUBUNIT
BLOCK PEPTIDE CHAIN ELONGTATION
CHLORAMPHENICOL PROTEIN SYNTHESIS BIND TO 50S RIBOSOMAL SUBUNIT
BLANK AMINOACYL TRNA ATTAXHMENT
136
AIIMS NOV 2015-QUESTIONS & ANSWERS WITH EXPLANATIONS-SOLVED BY
DR: MANJUNATH, MBBS, MD (MAMC NEW DELHI) DIRECTOR
DOCTORS ACADEMY DAVANAGERE & SHIMOGA
(www.doctorsacademydvg.com)
DR MANJUNATH, DOCTORS
ACADEMY, DAVANAGEREDR. MANJUNATH, DOCTORS ACADEMY
TETRACYCLINE PROTEIN SYNTHESIS BIND TO 50S RIBOSOMAL SUBUNIT
BLOCK AMINOACYL TRNA
AMINOGLYCOSIDES
(GENTAMYCIN)
PROTEIN SYNTHESIS BIND TO 50S RIBOSOMAL SUBUNIT
INHIBIT TRANSLOCATION OF PEPTIDYL-
TRNA
LINEZOLID PROTEIN SYNTHESIS BIND TO 50S RIBOSOMAL SUBUNIT
INHIBITS INITIATION OF PROTEIN
SYNTHESIS
SULFONAMIDES AND TRIMETHOPRIM CELL METABOLISM COMPETITIVELY INHIBIT ENZYMES
INVOLVED IN TWO STEPS OF FOLIC ACID
BIOSYNTHESIS
RIFAMPICIN NUCLEIC ACID SYNTHESIS INHIBITS DNA-DEPENDENT-RNA
POLYMERASE
METRONIDAZOLE NUCLEIC ACID SYNTHESIS INTRACELLULARLY GENERATES SHORT-
LIVED RE-ACTIVE INTERMEDIATES THAT
DAMAGE DNA BY ELECTRON TRANSFER
SYSTEM
QUINOLONESS
(CIPROFLOXACIN)
DNA SYNTHESIS INHIBIT ACTIVITY OF DNA GYRASE (A
SUBUNIT) AND TOPOISOMERASE IV
137
AIIMS NOV 2015-QUESTIONS & ANSWERS WITH EXPLANATIONS-SOLVED BY
DR: MANJUNATH, MBBS, MD (MAMC NEW DELHI) DIRECTOR
DOCTORS ACADEMY DAVANAGERE & SHIMOGA
(www.doctorsacademydvg.com)
DR MANJUNATH, DOCTORS
ACADEMY, DAVANAGEREDR. MANJUNATH, DOCTORS ACADEMY
NOVOBIOCIN DNA SYNTHESIS INHIBIT ACTIVITY OF DNA GYRASE ( B
SUBUNIT)
POLYMYXINS CELL MEMBRANE DISRUPT MEMBRANE PERMEABILITY
BY CHARGE ALTERATION
GRAMICIDIN CELL MEMBRANE FORM PORES
CHLORAMPHENICOL PROTEIN SYNTHESIS BIND 50S RIBOSOMAL SUBUNIT
BLOCK AMINOACYL TRNA
POSITIONING
OXAZOLIDINOMES (LINEZOLID
TEDIZOLID)
PROTEIN SYNTHESIS BIND 50S RIBOSOMAL SUBUNIT
INHIBIT IN;TIATIO OF PEPTIDE
SYNTHESIS
MUPIROCIN PROTEIN SYNTHESIS BLOCK ISOLEUCYL TRNA SYNTHEASE
SULFONAMIODES (SULFADIAZINE
SULFLSOXAZOLE AND
SULFAMETHOXAZOLE)
FOLATE SYNTHESIS INHIBITS DIHYDROFOLATE SYNTHETASE
TRIMETHOPRIN FOLATE SYNTHESIS INHIBITS DIHYDROFOLATE REDUCTASE
QUINOLONERS (NORFLOXACIN,
CIPROFLOXACIN, OFLOXACIN
LEVOFLOXACIN MOXIFLOXACIN
GEMIFLOXACIN)
DNA SYNTHESIS INHIBIT DNA GYRASE AND DNA
TOPOISOMERASE IV ENZYME-DNA-
DRUG COMPLEX:BLOCK REPLICATION
APPARATUS
138
AIIMS NOV 2015-QUESTIONS & ANSWERS WITH EXPLANATIONS-SOLVED BY
DR: MANJUNATH, MBBS, MD (MAMC NEW DELHI) DIRECTOR
DOCTORS ACADEMY DAVANAGERE & SHIMOGA
(www.doctorsacademydvg.com)
DR MANJUNATH, DOCTORS
ACADEMY, DAVANAGEREDR. MANJUNATH, DOCTORS ACADEMY
RIFAMYCINS (RIFAMPIN
FIFABUTIN, RIFAPENTNE)
RNA SYNTHESIS INHIBIT RNA POLYMERASE
NITROFURANTOIN NUCLEIC ACID SYNTHESIS REDUCE REACTIVE DRUG
DERIVATIVES THAT DAMAGE DNA
METRONIDAZOLE NUCLEIC ACID SYNTHESIS BIND LPS AND MEMBRANE
CHANNEL AND MEMBRANE
LEAKAGE
POLYMYXIN E (CLISTIN) CELL MEMBRANE BIND LPS AND DISRUPT: BOTH
OUTER AND SYTOPLASMIC
MEMBRANE
DAPTOMYCIN CELL MEMBRANE PRODUCES MEMBRANES
CHANNEL AND MEMBRANE
LEAKAGE
139
AIIMS NOV 2015-QUESTIONS & ANSWERS WITH EXPLANATIONS-SOLVED BY
DR: MANJUNATH, MBBS, MD (MAMC NEW DELHI) DIRECTOR
DOCTORS ACADEMY DAVANAGERE & SHIMOGA
(www.doctorsacademydvg.com)
DR MANJUNATH, DOCTORS
ACADEMY, DAVANAGEREDR. MANJUNATH, DOCTORS ACADEMY 140
AIIMS NOV 2015-QUESTIONS & ANSWERS WITH EXPLANATIONS-SOLVED BY
DR: MANJUNATH, MBBS, MD (MAMC NEW DELHI) DIRECTOR
DOCTORS ACADEMY DAVANAGERE & SHIMOGA
(WWW.DOCTORSACADEMYDVG.COM)
64. GENDER SPECIFIC SIDE EFFECT OF VALPROATE IS?
A. WEIGHT GAIN
B. TREMORS
C. PCOD
D. ALOPECIA
ANS: C. PCOD
(REF: K.D TRIPATHY 7/E P405-409)
DR. MANJUNATH, DOCTORS ACADEMY 141
AIIMS NOV 2015-QUESTIONS & ANSWERS WITH EXPLANATIONS-SOLVED BY
DR: MANJUNATH, MBBS, MD (MAMC NEW DELHI) DIRECTOR
DOCTORS ACADEMY DAVANAGERE & SHIMOGA
(WWW.DOCTORSACADEMYDVG.COM)
65. GLUCOCORTICOID IS NOT USED IN WHICH OF THE
FOLLOWING CONDITIONS?
A. MULTIPLE MYELOMA
B. KAPOSI SARCOMA
C. HODGKIN LYMPHOMA
D. CLL
ANS: B. KAPOSI SARCOMA
(REF: HARRISON’S 19/E P716)
DR MANJUNATH, DOCTORS
ACADEMY, DAVANAGEREDR. MANJUNATH, DOCTORS ACADEMY 142
AIIMS NOV 2015-QUESTIONS & ANSWERS WITH EXPLANATIONS-SOLVED BY
DR: MANJUNATH, MBBS, MD (MAMC NEW DELHI) DIRECTOR
DOCTORS ACADEMY DAVANAGERE & SHIMOGA
(www.doctorsacademydvg.com)
EXPLANATION
DISCUSSING THE DRUGS USED IN THE TREATMENT OF ABOVE MALIGNANCIES
ONE NY ONE
DR MANJUNATH, DOCTORS
ACADEMY, DAVANAGEREDR. MANJUNATH, DOCTORS ACADEMY
(A) MULTIPLE MYELOMA THALIDOMIDE, DEXAMETHASONE, LENALIDOMIDE (A DERIVATIVE OF
THALIDOMIDE), BORTEZOMIB (A PROTEASOME INHIBITOR),
CYCLOPHOSPHAMIDE
(B) KAPOSI SARCOMA LIPOSOMAL DAUNORUBICIN, LIPOSOMAL DOXORUNICIN,
VINBLASTINE, AND PACLITAXEL-HAVE BEEN APPROVED BY FDA FOR
THIS INDICATION
(C) HODGKIN LYMPHOMA COMBINATION OF DOXOTUBICIN (ADRIAMYCIN), BLEOMYCIN
VINBLASTINE, AND DACARBAZINE (ABVD)
(D) CLL THE MOST COMMON TREATMENTS FOR PATIENTS WITH TYPICAL B-
CELL CLL/SMALL LYMPHOCYTIC LYMPHOMA HAVE BEEN
CHLORAMBUCIL OR FLUDARABINE, ALONE OR IN COMBINATION.
OTHER DRUGS USED: BENDAMUSTINE, CVP (CYCLOPHOSPHAMIDE,
VINCRISTINE, AND PREFNISONE) OR CHOP PLUS RITUXIMAB.
ALEMTUZUMAB (ANTI-CD52)
143
AIIMS NOV 2015-QUESTIONS & ANSWERS WITH EXPLANATIONS-SOLVED BY
DR: MANJUNATH, MBBS, MD (MAMC NEW DELHI) DIRECTOR
DOCTORS ACADEMY DAVANAGERE & SHIMOGA
(WWW.DOCTORSACADEMYDVG.COM)
66. WHICH OF THE FOLLOWING IS A CALCINEURIN INHIBITOR?
A. CYCLOSPORINE
B. METHOTREXATE
C. LEFLUNOMIDE
D. MYCOPHENOLATE MOFETIL
ANS: A. CYCLOSPORINE
REF: K.D TRIPATHY 7/E P838-839
DR MANJUNATH, DOCTORS
ACADEMY, DAVANAGEREDR. MANJUNATH, DOCTORS ACADEMY 144
AIIMS NOV 2015-QUESTIONS & ANSWERS WITH EXPLANATIONS-SOLVED BY
DR: MANJUNATH, MBBS, MD (MAMC NEW DELHI) DIRECTOR
DOCTORS ACADEMY DAVANAGERE & SHIMOGA
(www.doctorsacademydvg.com)
EXPLANATION:
DR MANJUNATH, DOCTORS
ACADEMY, DAVANAGEREDR. MANJUNATH, DOCTORS ACADEMY
BIND TO IMMUNIPHILLIN
↓
INHIBITS ACTIVATION OF NUCLEAR FACTOR OF ACTIVATED T CELLS (NFAT)
↓
INHIBITION OF TRANSCRIPTION FACTOR OF IL-2
↓
IMMUNISUPPRESSIVE ACTION
145
AIIMS NOV 2015-QUESTIONS & ANSWERS WITH EXPLANATIONS-SOLVED BY
DR: MANJUNATH, MBBS, MD (MAMC NEW DELHI) DIRECTOR
DOCTORS ACADEMY DAVANAGERE & SHIMOGA
(WWW.DOCTORSACADEMYDVG.COM)
67. DRUG OF CHOICE FOR BIRD FLU?
A. OSELTAMIVIR
B. RIBAVIRIN
C. ENTECAVIR
D. ACYCLOVIR
ANS: A. OSELTAMIVIR
REF: HARRISON’S 19/E P1209-1213
DR MANJUNATH, DOCTORS
ACADEMY, DAVANAGEREDR. MANJUNATH, DOCTORS ACADEMY 146
AIIMS NOV 2015-QUESTIONS & ANSWERS WITH EXPLANATIONS-SOLVED BY
DR: MANJUNATH, MBBS, MD (MAMC NEW DELHI) DIRECTOR
DOCTORS ACADEMY DAVANAGERE & SHIMOGA
(WWW.DOCTORSACADEMYDVG.COM)
68. AT A DOSE OF 3-
5MCG/KG/MIN, DOPAMINE
CAUSES?
A. β1 RECEPTOR STIMULATION
B. INCREASE IN RENAL FLOW
C. INCREASE BLOOD PRESSURE
D. VASOCONTRICTION
ANS: B. INCREASE IN RENAL
FLOW
(REF:GOODMAN & GILMAN
PHARMACOLOGY/CH 10)
DR MANJUNATH, DOCTORS
ACADEMY, DAVANAGEREDR. MANJUNATH, DOCTORS ACADEMY 147
AIIMS NOV 2015-QUESTIONS & ANSWERS WITH EXPLANATIONS-SOLVED BY
DR: MANJUNATH, MBBS, MD (MAMC NEW DELHI) DIRECTOR
DOCTORS ACADEMY DAVANAGERE & SHIMOGA
(WWW.DOCTORSACADEMYDVG.COM)
69. URIPRISTAL ACETATE IS A ?
A. SELECTIVE ESTROGEN
RECEPTOR MODULATOR
B. SELECTIVE PROGESTERONE
RECEPTOR MODULATOR
C. GNRH ANATAGONIST
D. GNRH AGONIST
ANS: B. SELETIVE PROGESTERONE
RECEPTOR MODULATOR (SPRM)
(REF: HARRISON 19/E P2391)
DR MANJUNATH, DOCTORS
ACADEMY, DAVANAGEREDR. MANJUNATH, DOCTORS ACADEMY 148
AIIMS NOV 2015-QUESTIONS & ANSWERS WITH EXPLANATIONS-SOLVED BY
DR: MANJUNATH, MBBS, MD (MAMC NEW DELHI) DIRECTOR
DOCTORS ACADEMY DAVANAGERE & SHIMOGA
(WWW.DOCTORSACADEMYDVG.COM)
MICROBIOLOGY
70. MOST COST EFFECTIVE WAY TO PREVENT INFECTION?
A. HANDWASHING
B. ANTIBIOTIC
C. CHEMOPROPHYLAXIS
D. CULTURE SENSITIVITY
ANS: A. HANDWASHING
(REF: HARRISON’S 19/E P152E9)
DR MANJUNATH, DOCTORS
ACADEMY, DAVANAGEREDR. MANJUNATH, DOCTORS ACADEMY 149
AIIMS NOV 2015-QUESTIONS & ANSWERS WITH EXPLANATIONS-SOLVED BY
DR: MANJUNATH, MBBS, MD (MAMC NEW DELHI) DIRECTOR
DOCTORS ACADEMY DAVANAGERE & SHIMOGA
(WWW.DOCTORSACADEMYDVG.COM)
71. IDENTIFY THE ORGANISM IN THE GIVEN FIGURE?
A. HISTOPLASMOSIS
B. CANDIDA
C. CRYPTOCOCCUS
D. COCCIDIODOMYCOSIS
ANS: C. CRYPTOCOCCUS
(REF: ANANTNARAYAN P611)
DR MANJUNATH, DOCTORS
ACADEMY, DAVANAGEREDR. MANJUNATH, DOCTORS ACADEMY 150
AIIMS NOV 2015-QUESTIONS & ANSWERS WITH EXPLANATIONS-SOLVED BY
DR: MANJUNATH, MBBS, MD (MAMC NEW DELHI) DIRECTOR
DOCTORS ACADEMY DAVANAGERE & SHIMOGA
(www.doctorsacademydvg.com)
EXPLANATION:
DIAGNOSIS OF CRYPTOCOCCUS INFECTION
DR MANJUNATH, DOCTORS
ACADEMY, DAVANAGEREDR. MANJUNATH, DOCTORS ACADEMY
INDIA INK STAINING  A DIAGNOSIS OF CRYPTOCOCCOSIS REQUIRES THE DEMONSTRATION OF
YEAST CELLS IN NORMALLY STERILE TISSUES.
 VISUALIZATION OF THE CAPSULE OF FUNGAL CELLS IN CEREBROSPINAL
FLUID (CSF) MIXED WITH INDIA INK IS A USEFUL RAPID DIAGNOSTIC
TECHNIQUE.
 CRYPTOCOCCAL CELLS IN INDIA INK HAVE A DISTINCTIVE APPEARANCE
BECAUSE THEIR CAPSULES EXCLUDE INK PARTICLES.
 HOWEVER, THE CSF INDIA INK EXAMINATION MAY YIELD NEGATIVE
RESULTS IN PATIENTS WITH A LOW FUNGAL BURDEN.
 THIS EXAMINATION SHOULD BE PERFORMED BY A TRAINED INDIVIDUAL,
SINCE LEUKOCYTES & FAT GLOBULES CAN SOMETIMES BE MISTAKEN FOR
FUNGAL CELLS.
151
AIIMS NOV 2015-QUESTIONS & ANSWERS WITH EXPLANATIONS-SOLVED BY
DR: MANJUNATH, MBBS, MD (MAMC NEW DELHI) DIRECTOR
DOCTORS ACADEMY DAVANAGERE & SHIMOGA
(www.doctorsacademydvg.com): CRYPTOCOCCUS
DR MANJUNATH, DOCTORS
ACADEMY, DAVANAGEREDR. MANJUNATH, DOCTORS ACADEMY 152
AIIMS NOV 2015-QUESTIONS & ANSWERS WITH EXPLANATIONS-SOLVED BY
DR: MANJUNATH, MBBS, MD (MAMC NEW DELHI) DIRECTOR
DOCTORS ACADEMY DAVANAGERE & SHIMOGA
(WWW.DOCTORSACADEMYDVG.COM)
72. STOOL SAMPLE FROM A PATIENT
SHOWS THE FOLLOWING. HOW DO
HUMANS GET THIS INFECTION?
A. INGESTION WITH FOOD
B. WORK IN DIRTY WATER
C. IMPROPERLY COOKED BEEF
D. DIRECT CONTACT
ANS: A. INGESTION WITH FOOD
(ENTAMOEBA CYST)
(REF: K. D CHATTERJEE COLOUR PLATES)
DR MANJUNATH, DOCTORS
ACADEMY, DAVANAGEREDR. MANJUNATH, DOCTORS ACADEMY 153
AIIMS NOV 2015-QUESTIONS & ANSWERS WITH EXPLANATIONS-SOLVED BY
DR: MANJUNATH, MBBS, MD (MAMC NEW DELHI) DIRECTOR
DOCTORS ACADEMY DAVANAGERE & SHIMOGA
(www.doctorsacademydvg.com)
EXPLANATION:
DR MANJUNATH, DOCTORS
ACADEMY, DAVANAGEREDR. MANJUNATH, DOCTORS ACADEMY
CHARACTERISTIC FEATURES DIAGNOSIS
 10-15 UM SIZED
 ROUND YELLOW COLORED STRUCTURE SEEN
 BROWN COLOURED 1-4 NUCLEI AND GLYCOGEN MASS
CYST OF ENTAMEOBA
 PEAR SHAPED YELLOW COLOURED SEEN WITH VISIBLE
AXOSTYLE
TROPHOZOITE OF GIARDIA
 6-10 UM SIZED DISTINCT CYST WALL SURROUND WITH
VISIBLE AXOSTYLE
CYST OF GIARDIA
154
AIIMS NOV 2015-QUESTIONS & ANSWERS WITH EXPLANATIONS-SOLVED BY
DR: MANJUNATH, MBBS, MD (MAMC NEW DELHI) DIRECTOR
DOCTORS ACADEMY DAVANAGERE & SHIMOGA
(www.doctorsacademydvg.com)
DR MANJUNATH, DOCTORS
ACADEMY, DAVANAGEREDR. MANJUNATH, DOCTORS ACADEMY
 40X60UM SIZED OVAL SHAPED YELLOW COLORED
STRUCTURE SEEN WITH LIGHTLY STAINED
BLASTOMERE
 SHELL MEMBRANE AND SEGMENTED OVUM WITH
LIGHT YELLOW STAINED BLASTOMERE
 CKEAR SHELL MEMBRANE AND BLASTOMERE
HOOKWORM EGG
 60X40 SIZED ROUND OVAL SHAPED YELLOW
COLOURED STRUCTURE SEEN WITH YELLOW STAINED
OUTER CORTICAL THICK CELL WALL
 UNSEGMENTED OVUM AND ALSO THE SPACE
BETWEEN THE SHELL AND OVUM AT EACH POLE ARE
STAINED YELLOW.
FERTILIZED EGG OF ASCARIS LUMBRICOIDS
 25X50 UM SIZED YELLOW COLORED,
 BARREL SHAPED STRUCTURE SEEN WITH LIGHTLY
STAINED MUCUS PLUGS AT EACH POLE
 EGG SHELL IS STAINED BROWN AND ENCLOSES THE
LIGHT YELLOW STAINED UNSEGMENTED OVUM
EGGS OF TRICHURIS TRICHURA
155
1. AIIMS NOVEMBER 2015 QUESTIONS & ANSWERS
1. AIIMS NOVEMBER 2015 QUESTIONS & ANSWERS
1. AIIMS NOVEMBER 2015 QUESTIONS & ANSWERS
1. AIIMS NOVEMBER 2015 QUESTIONS & ANSWERS
1. AIIMS NOVEMBER 2015 QUESTIONS & ANSWERS
1. AIIMS NOVEMBER 2015 QUESTIONS & ANSWERS
1. AIIMS NOVEMBER 2015 QUESTIONS & ANSWERS
1. AIIMS NOVEMBER 2015 QUESTIONS & ANSWERS
1. AIIMS NOVEMBER 2015 QUESTIONS & ANSWERS
1. AIIMS NOVEMBER 2015 QUESTIONS & ANSWERS
1. AIIMS NOVEMBER 2015 QUESTIONS & ANSWERS
1. AIIMS NOVEMBER 2015 QUESTIONS & ANSWERS
1. AIIMS NOVEMBER 2015 QUESTIONS & ANSWERS
1. AIIMS NOVEMBER 2015 QUESTIONS & ANSWERS
1. AIIMS NOVEMBER 2015 QUESTIONS & ANSWERS
1. AIIMS NOVEMBER 2015 QUESTIONS & ANSWERS
1. AIIMS NOVEMBER 2015 QUESTIONS & ANSWERS
1. AIIMS NOVEMBER 2015 QUESTIONS & ANSWERS
1. AIIMS NOVEMBER 2015 QUESTIONS & ANSWERS
1. AIIMS NOVEMBER 2015 QUESTIONS & ANSWERS
1. AIIMS NOVEMBER 2015 QUESTIONS & ANSWERS
1. AIIMS NOVEMBER 2015 QUESTIONS & ANSWERS
1. AIIMS NOVEMBER 2015 QUESTIONS & ANSWERS
1. AIIMS NOVEMBER 2015 QUESTIONS & ANSWERS
1. AIIMS NOVEMBER 2015 QUESTIONS & ANSWERS
1. AIIMS NOVEMBER 2015 QUESTIONS & ANSWERS
1. AIIMS NOVEMBER 2015 QUESTIONS & ANSWERS
1. AIIMS NOVEMBER 2015 QUESTIONS & ANSWERS
1. AIIMS NOVEMBER 2015 QUESTIONS & ANSWERS
1. AIIMS NOVEMBER 2015 QUESTIONS & ANSWERS
1. AIIMS NOVEMBER 2015 QUESTIONS & ANSWERS
1. AIIMS NOVEMBER 2015 QUESTIONS & ANSWERS
1. AIIMS NOVEMBER 2015 QUESTIONS & ANSWERS
1. AIIMS NOVEMBER 2015 QUESTIONS & ANSWERS
1. AIIMS NOVEMBER 2015 QUESTIONS & ANSWERS
1. AIIMS NOVEMBER 2015 QUESTIONS & ANSWERS
1. AIIMS NOVEMBER 2015 QUESTIONS & ANSWERS
1. AIIMS NOVEMBER 2015 QUESTIONS & ANSWERS
1. AIIMS NOVEMBER 2015 QUESTIONS & ANSWERS
1. AIIMS NOVEMBER 2015 QUESTIONS & ANSWERS
1. AIIMS NOVEMBER 2015 QUESTIONS & ANSWERS
1. AIIMS NOVEMBER 2015 QUESTIONS & ANSWERS
1. AIIMS NOVEMBER 2015 QUESTIONS & ANSWERS
1. AIIMS NOVEMBER 2015 QUESTIONS & ANSWERS
1. AIIMS NOVEMBER 2015 QUESTIONS & ANSWERS
1. AIIMS NOVEMBER 2015 QUESTIONS & ANSWERS
1. AIIMS NOVEMBER 2015 QUESTIONS & ANSWERS
1. AIIMS NOVEMBER 2015 QUESTIONS & ANSWERS
1. AIIMS NOVEMBER 2015 QUESTIONS & ANSWERS
1. AIIMS NOVEMBER 2015 QUESTIONS & ANSWERS
1. AIIMS NOVEMBER 2015 QUESTIONS & ANSWERS
1. AIIMS NOVEMBER 2015 QUESTIONS & ANSWERS
1. AIIMS NOVEMBER 2015 QUESTIONS & ANSWERS
1. AIIMS NOVEMBER 2015 QUESTIONS & ANSWERS
1. AIIMS NOVEMBER 2015 QUESTIONS & ANSWERS
1. AIIMS NOVEMBER 2015 QUESTIONS & ANSWERS
1. AIIMS NOVEMBER 2015 QUESTIONS & ANSWERS
1. AIIMS NOVEMBER 2015 QUESTIONS & ANSWERS
1. AIIMS NOVEMBER 2015 QUESTIONS & ANSWERS
1. AIIMS NOVEMBER 2015 QUESTIONS & ANSWERS
1. AIIMS NOVEMBER 2015 QUESTIONS & ANSWERS
1. AIIMS NOVEMBER 2015 QUESTIONS & ANSWERS
1. AIIMS NOVEMBER 2015 QUESTIONS & ANSWERS
1. AIIMS NOVEMBER 2015 QUESTIONS & ANSWERS
1. AIIMS NOVEMBER 2015 QUESTIONS & ANSWERS
1. AIIMS NOVEMBER 2015 QUESTIONS & ANSWERS
1. AIIMS NOVEMBER 2015 QUESTIONS & ANSWERS
1. AIIMS NOVEMBER 2015 QUESTIONS & ANSWERS
1. AIIMS NOVEMBER 2015 QUESTIONS & ANSWERS
1. AIIMS NOVEMBER 2015 QUESTIONS & ANSWERS
1. AIIMS NOVEMBER 2015 QUESTIONS & ANSWERS
1. AIIMS NOVEMBER 2015 QUESTIONS & ANSWERS
1. AIIMS NOVEMBER 2015 QUESTIONS & ANSWERS
1. AIIMS NOVEMBER 2015 QUESTIONS & ANSWERS
1. AIIMS NOVEMBER 2015 QUESTIONS & ANSWERS
1. AIIMS NOVEMBER 2015 QUESTIONS & ANSWERS
1. AIIMS NOVEMBER 2015 QUESTIONS & ANSWERS
1. AIIMS NOVEMBER 2015 QUESTIONS & ANSWERS
1. AIIMS NOVEMBER 2015 QUESTIONS & ANSWERS
1. AIIMS NOVEMBER 2015 QUESTIONS & ANSWERS
1. AIIMS NOVEMBER 2015 QUESTIONS & ANSWERS
1. AIIMS NOVEMBER 2015 QUESTIONS & ANSWERS
1. AIIMS NOVEMBER 2015 QUESTIONS & ANSWERS
1. AIIMS NOVEMBER 2015 QUESTIONS & ANSWERS
1. AIIMS NOVEMBER 2015 QUESTIONS & ANSWERS
1. AIIMS NOVEMBER 2015 QUESTIONS & ANSWERS
1. AIIMS NOVEMBER 2015 QUESTIONS & ANSWERS
1. AIIMS NOVEMBER 2015 QUESTIONS & ANSWERS
1. AIIMS NOVEMBER 2015 QUESTIONS & ANSWERS
1. AIIMS NOVEMBER 2015 QUESTIONS & ANSWERS
1. AIIMS NOVEMBER 2015 QUESTIONS & ANSWERS
1. AIIMS NOVEMBER 2015 QUESTIONS & ANSWERS
1. AIIMS NOVEMBER 2015 QUESTIONS & ANSWERS
1. AIIMS NOVEMBER 2015 QUESTIONS & ANSWERS
1. AIIMS NOVEMBER 2015 QUESTIONS & ANSWERS
1. AIIMS NOVEMBER 2015 QUESTIONS & ANSWERS
1. AIIMS NOVEMBER 2015 QUESTIONS & ANSWERS
1. AIIMS NOVEMBER 2015 QUESTIONS & ANSWERS
1. AIIMS NOVEMBER 2015 QUESTIONS & ANSWERS
1. AIIMS NOVEMBER 2015 QUESTIONS & ANSWERS
1. AIIMS NOVEMBER 2015 QUESTIONS & ANSWERS
1. AIIMS NOVEMBER 2015 QUESTIONS & ANSWERS
1. AIIMS NOVEMBER 2015 QUESTIONS & ANSWERS
1. AIIMS NOVEMBER 2015 QUESTIONS & ANSWERS
1. AIIMS NOVEMBER 2015 QUESTIONS & ANSWERS
1. AIIMS NOVEMBER 2015 QUESTIONS & ANSWERS
1. AIIMS NOVEMBER 2015 QUESTIONS & ANSWERS
1. AIIMS NOVEMBER 2015 QUESTIONS & ANSWERS
1. AIIMS NOVEMBER 2015 QUESTIONS & ANSWERS
1. AIIMS NOVEMBER 2015 QUESTIONS & ANSWERS
1. AIIMS NOVEMBER 2015 QUESTIONS & ANSWERS
1. AIIMS NOVEMBER 2015 QUESTIONS & ANSWERS
1. AIIMS NOVEMBER 2015 QUESTIONS & ANSWERS
1. AIIMS NOVEMBER 2015 QUESTIONS & ANSWERS
1. AIIMS NOVEMBER 2015 QUESTIONS & ANSWERS
1. AIIMS NOVEMBER 2015 QUESTIONS & ANSWERS
1. AIIMS NOVEMBER 2015 QUESTIONS & ANSWERS
1. AIIMS NOVEMBER 2015 QUESTIONS & ANSWERS
1. AIIMS NOVEMBER 2015 QUESTIONS & ANSWERS
1. AIIMS NOVEMBER 2015 QUESTIONS & ANSWERS
1. AIIMS NOVEMBER 2015 QUESTIONS & ANSWERS
1. AIIMS NOVEMBER 2015 QUESTIONS & ANSWERS
1. AIIMS NOVEMBER 2015 QUESTIONS & ANSWERS
1. AIIMS NOVEMBER 2015 QUESTIONS & ANSWERS
1. AIIMS NOVEMBER 2015 QUESTIONS & ANSWERS
1. AIIMS NOVEMBER 2015 QUESTIONS & ANSWERS
1. AIIMS NOVEMBER 2015 QUESTIONS & ANSWERS
1. AIIMS NOVEMBER 2015 QUESTIONS & ANSWERS
1. AIIMS NOVEMBER 2015 QUESTIONS & ANSWERS
1. AIIMS NOVEMBER 2015 QUESTIONS & ANSWERS
1. AIIMS NOVEMBER 2015 QUESTIONS & ANSWERS
1. AIIMS NOVEMBER 2015 QUESTIONS & ANSWERS
1. AIIMS NOVEMBER 2015 QUESTIONS & ANSWERS
1. AIIMS NOVEMBER 2015 QUESTIONS & ANSWERS
1. AIIMS NOVEMBER 2015 QUESTIONS & ANSWERS
1. AIIMS NOVEMBER 2015 QUESTIONS & ANSWERS
1. AIIMS NOVEMBER 2015 QUESTIONS & ANSWERS
1. AIIMS NOVEMBER 2015 QUESTIONS & ANSWERS
1. AIIMS NOVEMBER 2015 QUESTIONS & ANSWERS
1. AIIMS NOVEMBER 2015 QUESTIONS & ANSWERS
1. AIIMS NOVEMBER 2015 QUESTIONS & ANSWERS
1. AIIMS NOVEMBER 2015 QUESTIONS & ANSWERS
1. AIIMS NOVEMBER 2015 QUESTIONS & ANSWERS
1. AIIMS NOVEMBER 2015 QUESTIONS & ANSWERS
1. AIIMS NOVEMBER 2015 QUESTIONS & ANSWERS
1. AIIMS NOVEMBER 2015 QUESTIONS & ANSWERS
1. AIIMS NOVEMBER 2015 QUESTIONS & ANSWERS
1. AIIMS NOVEMBER 2015 QUESTIONS & ANSWERS
1. AIIMS NOVEMBER 2015 QUESTIONS & ANSWERS
1. AIIMS NOVEMBER 2015 QUESTIONS & ANSWERS
1. AIIMS NOVEMBER 2015 QUESTIONS & ANSWERS
1. AIIMS NOVEMBER 2015 QUESTIONS & ANSWERS
1. AIIMS NOVEMBER 2015 QUESTIONS & ANSWERS
1. AIIMS NOVEMBER 2015 QUESTIONS & ANSWERS
1. AIIMS NOVEMBER 2015 QUESTIONS & ANSWERS
1. AIIMS NOVEMBER 2015 QUESTIONS & ANSWERS
1. AIIMS NOVEMBER 2015 QUESTIONS & ANSWERS
1. AIIMS NOVEMBER 2015 QUESTIONS & ANSWERS
1. AIIMS NOVEMBER 2015 QUESTIONS & ANSWERS
1. AIIMS NOVEMBER 2015 QUESTIONS & ANSWERS
1. AIIMS NOVEMBER 2015 QUESTIONS & ANSWERS
1. AIIMS NOVEMBER 2015 QUESTIONS & ANSWERS
1. AIIMS NOVEMBER 2015 QUESTIONS & ANSWERS
1. AIIMS NOVEMBER 2015 QUESTIONS & ANSWERS
1. AIIMS NOVEMBER 2015 QUESTIONS & ANSWERS
1. AIIMS NOVEMBER 2015 QUESTIONS & ANSWERS
1. AIIMS NOVEMBER 2015 QUESTIONS & ANSWERS
1. AIIMS NOVEMBER 2015 QUESTIONS & ANSWERS
1. AIIMS NOVEMBER 2015 QUESTIONS & ANSWERS
1. AIIMS NOVEMBER 2015 QUESTIONS & ANSWERS
1. AIIMS NOVEMBER 2015 QUESTIONS & ANSWERS
1. AIIMS NOVEMBER 2015 QUESTIONS & ANSWERS
1. AIIMS NOVEMBER 2015 QUESTIONS & ANSWERS
1. AIIMS NOVEMBER 2015 QUESTIONS & ANSWERS
1. AIIMS NOVEMBER 2015 QUESTIONS & ANSWERS
1. AIIMS NOVEMBER 2015 QUESTIONS & ANSWERS
1. AIIMS NOVEMBER 2015 QUESTIONS & ANSWERS
1. AIIMS NOVEMBER 2015 QUESTIONS & ANSWERS
1. AIIMS NOVEMBER 2015 QUESTIONS & ANSWERS
1. AIIMS NOVEMBER 2015 QUESTIONS & ANSWERS
1. AIIMS NOVEMBER 2015 QUESTIONS & ANSWERS
1. AIIMS NOVEMBER 2015 QUESTIONS & ANSWERS
1. AIIMS NOVEMBER 2015 QUESTIONS & ANSWERS
1. AIIMS NOVEMBER 2015 QUESTIONS & ANSWERS
1. AIIMS NOVEMBER 2015 QUESTIONS & ANSWERS
1. AIIMS NOVEMBER 2015 QUESTIONS & ANSWERS
1. AIIMS NOVEMBER 2015 QUESTIONS & ANSWERS
1. AIIMS NOVEMBER 2015 QUESTIONS & ANSWERS
1. AIIMS NOVEMBER 2015 QUESTIONS & ANSWERS
1. AIIMS NOVEMBER 2015 QUESTIONS & ANSWERS
1. AIIMS NOVEMBER 2015 QUESTIONS & ANSWERS
1. AIIMS NOVEMBER 2015 QUESTIONS & ANSWERS
1. AIIMS NOVEMBER 2015 QUESTIONS & ANSWERS
1. AIIMS NOVEMBER 2015 QUESTIONS & ANSWERS
1. AIIMS NOVEMBER 2015 QUESTIONS & ANSWERS
1. AIIMS NOVEMBER 2015 QUESTIONS & ANSWERS
1. AIIMS NOVEMBER 2015 QUESTIONS & ANSWERS
1. AIIMS NOVEMBER 2015 QUESTIONS & ANSWERS
1. AIIMS NOVEMBER 2015 QUESTIONS & ANSWERS
1. AIIMS NOVEMBER 2015 QUESTIONS & ANSWERS
1. AIIMS NOVEMBER 2015 QUESTIONS & ANSWERS
1. AIIMS NOVEMBER 2015 QUESTIONS & ANSWERS
1. AIIMS NOVEMBER 2015 QUESTIONS & ANSWERS
1. AIIMS NOVEMBER 2015 QUESTIONS & ANSWERS
1. AIIMS NOVEMBER 2015 QUESTIONS & ANSWERS
1. AIIMS NOVEMBER 2015 QUESTIONS & ANSWERS
1. AIIMS NOVEMBER 2015 QUESTIONS & ANSWERS
1. AIIMS NOVEMBER 2015 QUESTIONS & ANSWERS
1. AIIMS NOVEMBER 2015 QUESTIONS & ANSWERS
1. AIIMS NOVEMBER 2015 QUESTIONS & ANSWERS
1. AIIMS NOVEMBER 2015 QUESTIONS & ANSWERS
1. AIIMS NOVEMBER 2015 QUESTIONS & ANSWERS
1. AIIMS NOVEMBER 2015 QUESTIONS & ANSWERS
1. AIIMS NOVEMBER 2015 QUESTIONS & ANSWERS
1. AIIMS NOVEMBER 2015 QUESTIONS & ANSWERS
1. AIIMS NOVEMBER 2015 QUESTIONS & ANSWERS
1. AIIMS NOVEMBER 2015 QUESTIONS & ANSWERS
1. AIIMS NOVEMBER 2015 QUESTIONS & ANSWERS
1. AIIMS NOVEMBER 2015 QUESTIONS & ANSWERS
1. AIIMS NOVEMBER 2015 QUESTIONS & ANSWERS
1. AIIMS NOVEMBER 2015 QUESTIONS & ANSWERS
1. AIIMS NOVEMBER 2015 QUESTIONS & ANSWERS
1. AIIMS NOVEMBER 2015 QUESTIONS & ANSWERS
1. AIIMS NOVEMBER 2015 QUESTIONS & ANSWERS
1. AIIMS NOVEMBER 2015 QUESTIONS & ANSWERS
1. AIIMS NOVEMBER 2015 QUESTIONS & ANSWERS
1. AIIMS NOVEMBER 2015 QUESTIONS & ANSWERS
1. AIIMS NOVEMBER 2015 QUESTIONS & ANSWERS
1. AIIMS NOVEMBER 2015 QUESTIONS & ANSWERS
1. AIIMS NOVEMBER 2015 QUESTIONS & ANSWERS
1. AIIMS NOVEMBER 2015 QUESTIONS & ANSWERS
1. AIIMS NOVEMBER 2015 QUESTIONS & ANSWERS
1. AIIMS NOVEMBER 2015 QUESTIONS & ANSWERS
1. AIIMS NOVEMBER 2015 QUESTIONS & ANSWERS
1. AIIMS NOVEMBER 2015 QUESTIONS & ANSWERS
1. AIIMS NOVEMBER 2015 QUESTIONS & ANSWERS
1. AIIMS NOVEMBER 2015 QUESTIONS & ANSWERS
1. AIIMS NOVEMBER 2015 QUESTIONS & ANSWERS
1. AIIMS NOVEMBER 2015 QUESTIONS & ANSWERS
1. AIIMS NOVEMBER 2015 QUESTIONS & ANSWERS
1. AIIMS NOVEMBER 2015 QUESTIONS & ANSWERS
1. AIIMS NOVEMBER 2015 QUESTIONS & ANSWERS
1. AIIMS NOVEMBER 2015 QUESTIONS & ANSWERS
1. AIIMS NOVEMBER 2015 QUESTIONS & ANSWERS
1. AIIMS NOVEMBER 2015 QUESTIONS & ANSWERS
1. AIIMS NOVEMBER 2015 QUESTIONS & ANSWERS
1. AIIMS NOVEMBER 2015 QUESTIONS & ANSWERS
1. AIIMS NOVEMBER 2015 QUESTIONS & ANSWERS
1. AIIMS NOVEMBER 2015 QUESTIONS & ANSWERS
1. AIIMS NOVEMBER 2015 QUESTIONS & ANSWERS
1. AIIMS NOVEMBER 2015 QUESTIONS & ANSWERS
1. AIIMS NOVEMBER 2015 QUESTIONS & ANSWERS
1. AIIMS NOVEMBER 2015 QUESTIONS & ANSWERS
1. AIIMS NOVEMBER 2015 QUESTIONS & ANSWERS
1. AIIMS NOVEMBER 2015 QUESTIONS & ANSWERS
1. AIIMS NOVEMBER 2015 QUESTIONS & ANSWERS
1. AIIMS NOVEMBER 2015 QUESTIONS & ANSWERS
1. AIIMS NOVEMBER 2015 QUESTIONS & ANSWERS
1. AIIMS NOVEMBER 2015 QUESTIONS & ANSWERS
1. AIIMS NOVEMBER 2015 QUESTIONS & ANSWERS
1. AIIMS NOVEMBER 2015 QUESTIONS & ANSWERS
1. AIIMS NOVEMBER 2015 QUESTIONS & ANSWERS
1. AIIMS NOVEMBER 2015 QUESTIONS & ANSWERS
1. AIIMS NOVEMBER 2015 QUESTIONS & ANSWERS
1. AIIMS NOVEMBER 2015 QUESTIONS & ANSWERS
1. AIIMS NOVEMBER 2015 QUESTIONS & ANSWERS
1. AIIMS NOVEMBER 2015 QUESTIONS & ANSWERS
1. AIIMS NOVEMBER 2015 QUESTIONS & ANSWERS
1. AIIMS NOVEMBER 2015 QUESTIONS & ANSWERS
1. AIIMS NOVEMBER 2015 QUESTIONS & ANSWERS
1. AIIMS NOVEMBER 2015 QUESTIONS & ANSWERS
1. AIIMS NOVEMBER 2015 QUESTIONS & ANSWERS
1. AIIMS NOVEMBER 2015 QUESTIONS & ANSWERS
1. AIIMS NOVEMBER 2015 QUESTIONS & ANSWERS
1. AIIMS NOVEMBER 2015 QUESTIONS & ANSWERS
1. AIIMS NOVEMBER 2015 QUESTIONS & ANSWERS
1. AIIMS NOVEMBER 2015 QUESTIONS & ANSWERS
1. AIIMS NOVEMBER 2015 QUESTIONS & ANSWERS
1. AIIMS NOVEMBER 2015 QUESTIONS & ANSWERS
1. AIIMS NOVEMBER 2015 QUESTIONS & ANSWERS
1. AIIMS NOVEMBER 2015 QUESTIONS & ANSWERS

More Related Content

What's hot

Retinal diagram dr sabin sahu
Retinal diagram dr sabin sahuRetinal diagram dr sabin sahu
Retinal diagram dr sabin sahuvoveran312
 
Central retinal artery occlusion
Central retinal artery occlusionCentral retinal artery occlusion
Central retinal artery occlusionSSSIHMS-PG
 
Anti vegf intravitreal injections
Anti vegf intravitreal injectionsAnti vegf intravitreal injections
Anti vegf intravitreal injectionsAlaa Farsakh
 
Anti VEGF in Ophthalmology
Anti VEGF  in OphthalmologyAnti VEGF  in Ophthalmology
Anti VEGF in OphthalmologySahil Thakur
 
Intravitreal injection
Intravitreal injectionIntravitreal injection
Intravitreal injectionmaheshwari s
 
Anterior and posterior segment of eye
Anterior and posterior segment of eyeAnterior and posterior segment of eye
Anterior and posterior segment of eyeVisheshSAXENA11
 
Chronic progressive external ophthalmoplegia
Chronic progressive external ophthalmoplegiaChronic progressive external ophthalmoplegia
Chronic progressive external ophthalmoplegiaPS Deb
 
Paediatric cataract -DR ARNAV SAROYA
Paediatric cataract   -DR ARNAV SAROYAPaediatric cataract   -DR ARNAV SAROYA
Paediatric cataract -DR ARNAV SAROYADrArnavSaroya
 
Vascular Lesions Of The Orbit
Vascular Lesions Of The OrbitVascular Lesions Of The Orbit
Vascular Lesions Of The OrbitXiu Srithammasit
 
Myopic CNVM -DIAGNOSIS AND MGT BY dr ajay dudani
Myopic CNVM -DIAGNOSIS AND MGT BY dr ajay dudaniMyopic CNVM -DIAGNOSIS AND MGT BY dr ajay dudani
Myopic CNVM -DIAGNOSIS AND MGT BY dr ajay dudaniAjayDudani1
 

What's hot (20)

Oculomotor palsy
Oculomotor palsyOculomotor palsy
Oculomotor palsy
 
Retinal diagram dr sabin sahu
Retinal diagram dr sabin sahuRetinal diagram dr sabin sahu
Retinal diagram dr sabin sahu
 
Central retinal artery occlusion
Central retinal artery occlusionCentral retinal artery occlusion
Central retinal artery occlusion
 
Anti vegf intravitreal injections
Anti vegf intravitreal injectionsAnti vegf intravitreal injections
Anti vegf intravitreal injections
 
Anatomy of retina
Anatomy of retinaAnatomy of retina
Anatomy of retina
 
Optic neuritis
Optic neuritisOptic neuritis
Optic neuritis
 
Eye lymphatics
Eye lymphaticsEye lymphatics
Eye lymphatics
 
Intravitreal injection
Intravitreal  injectionIntravitreal  injection
Intravitreal injection
 
Vitreous hemorrhage
Vitreous hemorrhageVitreous hemorrhage
Vitreous hemorrhage
 
Fundus examination
Fundus examinationFundus examination
Fundus examination
 
Anti VEGF in Ophthalmology
Anti VEGF  in OphthalmologyAnti VEGF  in Ophthalmology
Anti VEGF in Ophthalmology
 
Intravitreal injection
Intravitreal injectionIntravitreal injection
Intravitreal injection
 
Anti vegf' s in Ophthalmology
Anti vegf' s in OphthalmologyAnti vegf' s in Ophthalmology
Anti vegf' s in Ophthalmology
 
Macular star
Macular starMacular star
Macular star
 
Anterior and posterior segment of eye
Anterior and posterior segment of eyeAnterior and posterior segment of eye
Anterior and posterior segment of eye
 
Chronic progressive external ophthalmoplegia
Chronic progressive external ophthalmoplegiaChronic progressive external ophthalmoplegia
Chronic progressive external ophthalmoplegia
 
Paediatric cataract -DR ARNAV SAROYA
Paediatric cataract   -DR ARNAV SAROYAPaediatric cataract   -DR ARNAV SAROYA
Paediatric cataract -DR ARNAV SAROYA
 
Vascular Lesions Of The Orbit
Vascular Lesions Of The OrbitVascular Lesions Of The Orbit
Vascular Lesions Of The Orbit
 
Sixth nerve palsy
Sixth nerve palsySixth nerve palsy
Sixth nerve palsy
 
Myopic CNVM -DIAGNOSIS AND MGT BY dr ajay dudani
Myopic CNVM -DIAGNOSIS AND MGT BY dr ajay dudaniMyopic CNVM -DIAGNOSIS AND MGT BY dr ajay dudani
Myopic CNVM -DIAGNOSIS AND MGT BY dr ajay dudani
 

Viewers also liked

MRCPsych08 - How To Analyse Diagnostic Test Studies (June08)
MRCPsych08 - How To Analyse Diagnostic Test Studies (June08)MRCPsych08 - How To Analyse Diagnostic Test Studies (June08)
MRCPsych08 - How To Analyse Diagnostic Test Studies (June08)Alex J Mitchell
 
Mr. Ranganathan
Mr. RanganathanMr. Ranganathan
Mr. RanganathanSujay Iyer
 
Future of Hyertension
Future of HyertensionFuture of Hyertension
Future of HyertensionSujay Iyer
 
Cardiorenal Syndrome
Cardiorenal SyndromeCardiorenal Syndrome
Cardiorenal SyndromeSujay Iyer
 
Diastolic murmurs
Diastolic murmursDiastolic murmurs
Diastolic murmursSujay Iyer
 
AIIMS 2016 MAY PSYCHIATRY QUESTIONS AND ANSWERS
AIIMS 2016 MAY PSYCHIATRY QUESTIONS AND ANSWERSAIIMS 2016 MAY PSYCHIATRY QUESTIONS AND ANSWERS
AIIMS 2016 MAY PSYCHIATRY QUESTIONS AND ANSWERSManjunath Shetty
 
Cardiology Class
Cardiology ClassCardiology Class
Cardiology ClassSujay Iyer
 
Systolic murmur
Systolic murmurSystolic murmur
Systolic murmurSujay Iyer
 
Diabetic dyslipidemia
Diabetic dyslipidemiaDiabetic dyslipidemia
Diabetic dyslipidemiadocaneesh
 
Beta blockers: Role in Hypertension
Beta blockers: Role in HypertensionBeta blockers: Role in Hypertension
Beta blockers: Role in HypertensionSujay Iyer
 
Prevalence of noncommunicable diseases in india
Prevalence of noncommunicable diseases in indiaPrevalence of noncommunicable diseases in india
Prevalence of noncommunicable diseases in indiaSujay Iyer
 

Viewers also liked (20)

New way to prepare medical pg
New way to prepare medical pgNew way to prepare medical pg
New way to prepare medical pg
 
Mci booklet
Mci bookletMci booklet
Mci booklet
 
MRCPsych08 - How To Analyse Diagnostic Test Studies (June08)
MRCPsych08 - How To Analyse Diagnostic Test Studies (June08)MRCPsych08 - How To Analyse Diagnostic Test Studies (June08)
MRCPsych08 - How To Analyse Diagnostic Test Studies (June08)
 
Hypertension
HypertensionHypertension
Hypertension
 
Mr. Ranganathan
Mr. RanganathanMr. Ranganathan
Mr. Ranganathan
 
Mr. Kannan
Mr. KannanMr. Kannan
Mr. Kannan
 
Future of Hyertension
Future of HyertensionFuture of Hyertension
Future of Hyertension
 
S1
S1S1
S1
 
S3 and S4
S3 and S4S3 and S4
S3 and S4
 
S2
S2S2
S2
 
Cardiorenal Syndrome
Cardiorenal SyndromeCardiorenal Syndrome
Cardiorenal Syndrome
 
Diastolic murmurs
Diastolic murmursDiastolic murmurs
Diastolic murmurs
 
AIIMS 2016 MAY PSYCHIATRY QUESTIONS AND ANSWERS
AIIMS 2016 MAY PSYCHIATRY QUESTIONS AND ANSWERSAIIMS 2016 MAY PSYCHIATRY QUESTIONS AND ANSWERS
AIIMS 2016 MAY PSYCHIATRY QUESTIONS AND ANSWERS
 
Cardiology Class
Cardiology ClassCardiology Class
Cardiology Class
 
Systolic murmur
Systolic murmurSystolic murmur
Systolic murmur
 
Diabetic dyslipidemia
Diabetic dyslipidemiaDiabetic dyslipidemia
Diabetic dyslipidemia
 
Value of tmt
Value of tmtValue of tmt
Value of tmt
 
Beta blockers: Role in Hypertension
Beta blockers: Role in HypertensionBeta blockers: Role in Hypertension
Beta blockers: Role in Hypertension
 
Prevalence of noncommunicable diseases in india
Prevalence of noncommunicable diseases in indiaPrevalence of noncommunicable diseases in india
Prevalence of noncommunicable diseases in india
 
Hyperkalemia
HyperkalemiaHyperkalemia
Hyperkalemia
 

Similar to 1. AIIMS NOVEMBER 2015 QUESTIONS & ANSWERS

Protraction face mask /certified fixed orthodontic courses by Indian dental a...
Protraction face mask /certified fixed orthodontic courses by Indian dental a...Protraction face mask /certified fixed orthodontic courses by Indian dental a...
Protraction face mask /certified fixed orthodontic courses by Indian dental a...Indian dental academy
 
Protraction face mask /certified fixed orthodontic courses by Indian dental a...
Protraction face mask /certified fixed orthodontic courses by Indian dental a...Protraction face mask /certified fixed orthodontic courses by Indian dental a...
Protraction face mask /certified fixed orthodontic courses by Indian dental a...Indian dental academy
 
Canaloplasty The Keys to Optimizing Patients' Outcomes
Canaloplasty The Keys to Optimizing Patients' OutcomesCanaloplasty The Keys to Optimizing Patients' Outcomes
Canaloplasty The Keys to Optimizing Patients' OutcomesDr David Richardson
 
Clipping or Coiling for MCA Aneurysm
Clipping or Coiling for MCA AneurysmClipping or Coiling for MCA Aneurysm
Clipping or Coiling for MCA AneurysmDr Vipul Gupta
 
Implant occlusion and failures /certified fixed orthodontic courses by Indian...
Implant occlusion and failures /certified fixed orthodontic courses by Indian...Implant occlusion and failures /certified fixed orthodontic courses by Indian...
Implant occlusion and failures /certified fixed orthodontic courses by Indian...Indian dental academy
 
Implant and occlusion failures goli / dental implant courses by Indian dental...
Implant and occlusion failures goli / dental implant courses by Indian dental...Implant and occlusion failures goli / dental implant courses by Indian dental...
Implant and occlusion failures goli / dental implant courses by Indian dental...Indian dental academy
 
Arch expansion with fixed appliance technique
Arch expansion with fixed appliance techniqueArch expansion with fixed appliance technique
Arch expansion with fixed appliance techniqueRavikanth lakkakula
 
Implant occlusion and failures./ stomatology and dentistry
Implant occlusion and failures./ stomatology and dentistryImplant occlusion and failures./ stomatology and dentistry
Implant occlusion and failures./ stomatology and dentistryIndian dental academy
 
Implant occlusion and failures1/ dental crown & bridge courses
Implant occlusion and failures1/ dental crown & bridge coursesImplant occlusion and failures1/ dental crown & bridge courses
Implant occlusion and failures1/ dental crown & bridge coursesIndian dental academy
 
Dr Patrick Treacy shares some of his most challenging cases. This month he ta...
Dr Patrick Treacy shares some of his most challenging cases. This month he ta...Dr Patrick Treacy shares some of his most challenging cases. This month he ta...
Dr Patrick Treacy shares some of his most challenging cases. This month he ta...Dr. Patrick J. Treacy
 
Early vs late orthodontic treatment /certified fixed orthodontic courses by I...
Early vs late orthodontic treatment /certified fixed orthodontic courses by I...Early vs late orthodontic treatment /certified fixed orthodontic courses by I...
Early vs late orthodontic treatment /certified fixed orthodontic courses by I...Indian dental academy
 
Invisalign in pediatric dentistry
Invisalign in pediatric dentistryInvisalign in pediatric dentistry
Invisalign in pediatric dentistryDr Ramesh R
 
sutures and suturing techniques
sutures and suturing techniquessutures and suturing techniques
sutures and suturing techniquesvesta enid lydia
 

Similar to 1. AIIMS NOVEMBER 2015 QUESTIONS & ANSWERS (20)

Protraction face mask /certified fixed orthodontic courses by Indian dental a...
Protraction face mask /certified fixed orthodontic courses by Indian dental a...Protraction face mask /certified fixed orthodontic courses by Indian dental a...
Protraction face mask /certified fixed orthodontic courses by Indian dental a...
 
Protraction face mask /certified fixed orthodontic courses by Indian dental a...
Protraction face mask /certified fixed orthodontic courses by Indian dental a...Protraction face mask /certified fixed orthodontic courses by Indian dental a...
Protraction face mask /certified fixed orthodontic courses by Indian dental a...
 
Canaloplasty The Keys to Optimizing Patients' Outcomes
Canaloplasty The Keys to Optimizing Patients' OutcomesCanaloplasty The Keys to Optimizing Patients' Outcomes
Canaloplasty The Keys to Optimizing Patients' Outcomes
 
Clipping or Coiling for MCA Aneurysm
Clipping or Coiling for MCA AneurysmClipping or Coiling for MCA Aneurysm
Clipping or Coiling for MCA Aneurysm
 
Implant occlusion and failures /certified fixed orthodontic courses by Indian...
Implant occlusion and failures /certified fixed orthodontic courses by Indian...Implant occlusion and failures /certified fixed orthodontic courses by Indian...
Implant occlusion and failures /certified fixed orthodontic courses by Indian...
 
Goli implant/ dental courses
Goli implant/ dental coursesGoli implant/ dental courses
Goli implant/ dental courses
 
Implant occlusion and failures
Implant occlusion and failuresImplant occlusion and failures
Implant occlusion and failures
 
Implant and occlusion failures goli / dental implant courses by Indian dental...
Implant and occlusion failures goli / dental implant courses by Indian dental...Implant and occlusion failures goli / dental implant courses by Indian dental...
Implant and occlusion failures goli / dental implant courses by Indian dental...
 
Arch expansion with fixed appliance technique
Arch expansion with fixed appliance techniqueArch expansion with fixed appliance technique
Arch expansion with fixed appliance technique
 
Distraction osteogenesis (4)
Distraction osteogenesis (4)Distraction osteogenesis (4)
Distraction osteogenesis (4)
 
Implant occlusion and failures./ stomatology and dentistry
Implant occlusion and failures./ stomatology and dentistryImplant occlusion and failures./ stomatology and dentistry
Implant occlusion and failures./ stomatology and dentistry
 
Implant occlusion and failures1/ dental crown & bridge courses
Implant occlusion and failures1/ dental crown & bridge coursesImplant occlusion and failures1/ dental crown & bridge courses
Implant occlusion and failures1/ dental crown & bridge courses
 
JADA_News_Oct_2014
JADA_News_Oct_2014JADA_News_Oct_2014
JADA_News_Oct_2014
 
How did it all start
How did it all startHow did it all start
How did it all start
 
Dr Patrick Treacy shares some of his most challenging cases. This month he ta...
Dr Patrick Treacy shares some of his most challenging cases. This month he ta...Dr Patrick Treacy shares some of his most challenging cases. This month he ta...
Dr Patrick Treacy shares some of his most challenging cases. This month he ta...
 
Early vs late orthodontic treatment /certified fixed orthodontic courses by I...
Early vs late orthodontic treatment /certified fixed orthodontic courses by I...Early vs late orthodontic treatment /certified fixed orthodontic courses by I...
Early vs late orthodontic treatment /certified fixed orthodontic courses by I...
 
Invisalign in pediatric dentistry
Invisalign in pediatric dentistryInvisalign in pediatric dentistry
Invisalign in pediatric dentistry
 
sutures and suturing techniques
sutures and suturing techniquessutures and suturing techniques
sutures and suturing techniques
 
Non Extraction treatment2
Non Extraction treatment2Non Extraction treatment2
Non Extraction treatment2
 
Non extraction treatment modality
Non extraction treatment modalityNon extraction treatment modality
Non extraction treatment modality
 

Recently uploaded

Top Rated Hyderabad Call Girls Erragadda ⟟ 6297143586 ⟟ Call Me For Genuine ...
Top Rated  Hyderabad Call Girls Erragadda ⟟ 6297143586 ⟟ Call Me For Genuine ...Top Rated  Hyderabad Call Girls Erragadda ⟟ 6297143586 ⟟ Call Me For Genuine ...
Top Rated Hyderabad Call Girls Erragadda ⟟ 6297143586 ⟟ Call Me For Genuine ...chandars293
 
VIP Hyderabad Call Girls Bahadurpally 7877925207 ₹5000 To 25K With AC Room 💚😋
VIP Hyderabad Call Girls Bahadurpally 7877925207 ₹5000 To 25K With AC Room 💚😋VIP Hyderabad Call Girls Bahadurpally 7877925207 ₹5000 To 25K With AC Room 💚😋
VIP Hyderabad Call Girls Bahadurpally 7877925207 ₹5000 To 25K With AC Room 💚😋TANUJA PANDEY
 
Bangalore Call Girls Nelamangala Number 7001035870 Meetin With Bangalore Esc...
Bangalore Call Girls Nelamangala Number 7001035870  Meetin With Bangalore Esc...Bangalore Call Girls Nelamangala Number 7001035870  Meetin With Bangalore Esc...
Bangalore Call Girls Nelamangala Number 7001035870 Meetin With Bangalore Esc...narwatsonia7
 
VIP Service Call Girls Sindhi Colony 📳 7877925207 For 18+ VIP Call Girl At Th...
VIP Service Call Girls Sindhi Colony 📳 7877925207 For 18+ VIP Call Girl At Th...VIP Service Call Girls Sindhi Colony 📳 7877925207 For 18+ VIP Call Girl At Th...
VIP Service Call Girls Sindhi Colony 📳 7877925207 For 18+ VIP Call Girl At Th...jageshsingh5554
 
Call Girls Jabalpur Just Call 9907093804 Top Class Call Girl Service Available
Call Girls Jabalpur Just Call 9907093804 Top Class Call Girl Service AvailableCall Girls Jabalpur Just Call 9907093804 Top Class Call Girl Service Available
Call Girls Jabalpur Just Call 9907093804 Top Class Call Girl Service AvailableDipal Arora
 
Russian Call Girls in Jaipur Riya WhatsApp ❤8445551418 VIP Call Girls Jaipur
Russian Call Girls in Jaipur Riya WhatsApp ❤8445551418 VIP Call Girls JaipurRussian Call Girls in Jaipur Riya WhatsApp ❤8445551418 VIP Call Girls Jaipur
Russian Call Girls in Jaipur Riya WhatsApp ❤8445551418 VIP Call Girls Jaipurparulsinha
 
Call Girls Nagpur Just Call 9907093804 Top Class Call Girl Service Available
Call Girls Nagpur Just Call 9907093804 Top Class Call Girl Service AvailableCall Girls Nagpur Just Call 9907093804 Top Class Call Girl Service Available
Call Girls Nagpur Just Call 9907093804 Top Class Call Girl Service AvailableDipal Arora
 
Call Girls Mumbai Just Call 9907093804 Top Class Call Girl Service Available
Call Girls Mumbai Just Call 9907093804 Top Class Call Girl Service AvailableCall Girls Mumbai Just Call 9907093804 Top Class Call Girl Service Available
Call Girls Mumbai Just Call 9907093804 Top Class Call Girl Service AvailableDipal Arora
 
Call Girls Gwalior Just Call 8617370543 Top Class Call Girl Service Available
Call Girls Gwalior Just Call 8617370543 Top Class Call Girl Service AvailableCall Girls Gwalior Just Call 8617370543 Top Class Call Girl Service Available
Call Girls Gwalior Just Call 8617370543 Top Class Call Girl Service AvailableDipal Arora
 
Call Girls Service Jaipur Grishma WhatsApp ❤8445551418 VIP Call Girls Jaipur
Call Girls Service Jaipur Grishma WhatsApp ❤8445551418 VIP Call Girls JaipurCall Girls Service Jaipur Grishma WhatsApp ❤8445551418 VIP Call Girls Jaipur
Call Girls Service Jaipur Grishma WhatsApp ❤8445551418 VIP Call Girls Jaipurparulsinha
 
The Most Attractive Hyderabad Call Girls Kothapet 𖠋 6297143586 𖠋 Will You Mis...
The Most Attractive Hyderabad Call Girls Kothapet 𖠋 6297143586 𖠋 Will You Mis...The Most Attractive Hyderabad Call Girls Kothapet 𖠋 6297143586 𖠋 Will You Mis...
The Most Attractive Hyderabad Call Girls Kothapet 𖠋 6297143586 𖠋 Will You Mis...chandars293
 
Call Girls Aurangabad Just Call 9907093804 Top Class Call Girl Service Available
Call Girls Aurangabad Just Call 9907093804 Top Class Call Girl Service AvailableCall Girls Aurangabad Just Call 9907093804 Top Class Call Girl Service Available
Call Girls Aurangabad Just Call 9907093804 Top Class Call Girl Service AvailableDipal Arora
 
Top Rated Bangalore Call Girls Richmond Circle ⟟ 8250192130 ⟟ Call Me For Gen...
Top Rated Bangalore Call Girls Richmond Circle ⟟ 8250192130 ⟟ Call Me For Gen...Top Rated Bangalore Call Girls Richmond Circle ⟟ 8250192130 ⟟ Call Me For Gen...
Top Rated Bangalore Call Girls Richmond Circle ⟟ 8250192130 ⟟ Call Me For Gen...narwatsonia7
 
Call Girls Bhubaneswar Just Call 9907093804 Top Class Call Girl Service Avail...
Call Girls Bhubaneswar Just Call 9907093804 Top Class Call Girl Service Avail...Call Girls Bhubaneswar Just Call 9907093804 Top Class Call Girl Service Avail...
Call Girls Bhubaneswar Just Call 9907093804 Top Class Call Girl Service Avail...Dipal Arora
 
♛VVIP Hyderabad Call Girls Chintalkunta🖕7001035870🖕Riya Kappor Top Call Girl ...
♛VVIP Hyderabad Call Girls Chintalkunta🖕7001035870🖕Riya Kappor Top Call Girl ...♛VVIP Hyderabad Call Girls Chintalkunta🖕7001035870🖕Riya Kappor Top Call Girl ...
♛VVIP Hyderabad Call Girls Chintalkunta🖕7001035870🖕Riya Kappor Top Call Girl ...astropune
 
Best Rate (Hyderabad) Call Girls Jahanuma ⟟ 8250192130 ⟟ High Class Call Girl...
Best Rate (Hyderabad) Call Girls Jahanuma ⟟ 8250192130 ⟟ High Class Call Girl...Best Rate (Hyderabad) Call Girls Jahanuma ⟟ 8250192130 ⟟ High Class Call Girl...
Best Rate (Hyderabad) Call Girls Jahanuma ⟟ 8250192130 ⟟ High Class Call Girl...astropune
 
(👑VVIP ISHAAN ) Russian Call Girls Service Navi Mumbai🖕9920874524🖕Independent...
(👑VVIP ISHAAN ) Russian Call Girls Service Navi Mumbai🖕9920874524🖕Independent...(👑VVIP ISHAAN ) Russian Call Girls Service Navi Mumbai🖕9920874524🖕Independent...
(👑VVIP ISHAAN ) Russian Call Girls Service Navi Mumbai🖕9920874524🖕Independent...Taniya Sharma
 
Call Girls Varanasi Just Call 9907093804 Top Class Call Girl Service Available
Call Girls Varanasi Just Call 9907093804 Top Class Call Girl Service AvailableCall Girls Varanasi Just Call 9907093804 Top Class Call Girl Service Available
Call Girls Varanasi Just Call 9907093804 Top Class Call Girl Service AvailableDipal Arora
 
(Rocky) Jaipur Call Girl - 09521753030 Escorts Service 50% Off with Cash ON D...
(Rocky) Jaipur Call Girl - 09521753030 Escorts Service 50% Off with Cash ON D...(Rocky) Jaipur Call Girl - 09521753030 Escorts Service 50% Off with Cash ON D...
(Rocky) Jaipur Call Girl - 09521753030 Escorts Service 50% Off with Cash ON D...indiancallgirl4rent
 
Call Girls Kochi Just Call 9907093804 Top Class Call Girl Service Available
Call Girls Kochi Just Call 9907093804 Top Class Call Girl Service AvailableCall Girls Kochi Just Call 9907093804 Top Class Call Girl Service Available
Call Girls Kochi Just Call 9907093804 Top Class Call Girl Service AvailableDipal Arora
 

Recently uploaded (20)

Top Rated Hyderabad Call Girls Erragadda ⟟ 6297143586 ⟟ Call Me For Genuine ...
Top Rated  Hyderabad Call Girls Erragadda ⟟ 6297143586 ⟟ Call Me For Genuine ...Top Rated  Hyderabad Call Girls Erragadda ⟟ 6297143586 ⟟ Call Me For Genuine ...
Top Rated Hyderabad Call Girls Erragadda ⟟ 6297143586 ⟟ Call Me For Genuine ...
 
VIP Hyderabad Call Girls Bahadurpally 7877925207 ₹5000 To 25K With AC Room 💚😋
VIP Hyderabad Call Girls Bahadurpally 7877925207 ₹5000 To 25K With AC Room 💚😋VIP Hyderabad Call Girls Bahadurpally 7877925207 ₹5000 To 25K With AC Room 💚😋
VIP Hyderabad Call Girls Bahadurpally 7877925207 ₹5000 To 25K With AC Room 💚😋
 
Bangalore Call Girls Nelamangala Number 7001035870 Meetin With Bangalore Esc...
Bangalore Call Girls Nelamangala Number 7001035870  Meetin With Bangalore Esc...Bangalore Call Girls Nelamangala Number 7001035870  Meetin With Bangalore Esc...
Bangalore Call Girls Nelamangala Number 7001035870 Meetin With Bangalore Esc...
 
VIP Service Call Girls Sindhi Colony 📳 7877925207 For 18+ VIP Call Girl At Th...
VIP Service Call Girls Sindhi Colony 📳 7877925207 For 18+ VIP Call Girl At Th...VIP Service Call Girls Sindhi Colony 📳 7877925207 For 18+ VIP Call Girl At Th...
VIP Service Call Girls Sindhi Colony 📳 7877925207 For 18+ VIP Call Girl At Th...
 
Call Girls Jabalpur Just Call 9907093804 Top Class Call Girl Service Available
Call Girls Jabalpur Just Call 9907093804 Top Class Call Girl Service AvailableCall Girls Jabalpur Just Call 9907093804 Top Class Call Girl Service Available
Call Girls Jabalpur Just Call 9907093804 Top Class Call Girl Service Available
 
Russian Call Girls in Jaipur Riya WhatsApp ❤8445551418 VIP Call Girls Jaipur
Russian Call Girls in Jaipur Riya WhatsApp ❤8445551418 VIP Call Girls JaipurRussian Call Girls in Jaipur Riya WhatsApp ❤8445551418 VIP Call Girls Jaipur
Russian Call Girls in Jaipur Riya WhatsApp ❤8445551418 VIP Call Girls Jaipur
 
Call Girls Nagpur Just Call 9907093804 Top Class Call Girl Service Available
Call Girls Nagpur Just Call 9907093804 Top Class Call Girl Service AvailableCall Girls Nagpur Just Call 9907093804 Top Class Call Girl Service Available
Call Girls Nagpur Just Call 9907093804 Top Class Call Girl Service Available
 
Call Girls Mumbai Just Call 9907093804 Top Class Call Girl Service Available
Call Girls Mumbai Just Call 9907093804 Top Class Call Girl Service AvailableCall Girls Mumbai Just Call 9907093804 Top Class Call Girl Service Available
Call Girls Mumbai Just Call 9907093804 Top Class Call Girl Service Available
 
Call Girls Gwalior Just Call 8617370543 Top Class Call Girl Service Available
Call Girls Gwalior Just Call 8617370543 Top Class Call Girl Service AvailableCall Girls Gwalior Just Call 8617370543 Top Class Call Girl Service Available
Call Girls Gwalior Just Call 8617370543 Top Class Call Girl Service Available
 
Call Girls Service Jaipur Grishma WhatsApp ❤8445551418 VIP Call Girls Jaipur
Call Girls Service Jaipur Grishma WhatsApp ❤8445551418 VIP Call Girls JaipurCall Girls Service Jaipur Grishma WhatsApp ❤8445551418 VIP Call Girls Jaipur
Call Girls Service Jaipur Grishma WhatsApp ❤8445551418 VIP Call Girls Jaipur
 
The Most Attractive Hyderabad Call Girls Kothapet 𖠋 6297143586 𖠋 Will You Mis...
The Most Attractive Hyderabad Call Girls Kothapet 𖠋 6297143586 𖠋 Will You Mis...The Most Attractive Hyderabad Call Girls Kothapet 𖠋 6297143586 𖠋 Will You Mis...
The Most Attractive Hyderabad Call Girls Kothapet 𖠋 6297143586 𖠋 Will You Mis...
 
Call Girls Aurangabad Just Call 9907093804 Top Class Call Girl Service Available
Call Girls Aurangabad Just Call 9907093804 Top Class Call Girl Service AvailableCall Girls Aurangabad Just Call 9907093804 Top Class Call Girl Service Available
Call Girls Aurangabad Just Call 9907093804 Top Class Call Girl Service Available
 
Top Rated Bangalore Call Girls Richmond Circle ⟟ 8250192130 ⟟ Call Me For Gen...
Top Rated Bangalore Call Girls Richmond Circle ⟟ 8250192130 ⟟ Call Me For Gen...Top Rated Bangalore Call Girls Richmond Circle ⟟ 8250192130 ⟟ Call Me For Gen...
Top Rated Bangalore Call Girls Richmond Circle ⟟ 8250192130 ⟟ Call Me For Gen...
 
Call Girls Bhubaneswar Just Call 9907093804 Top Class Call Girl Service Avail...
Call Girls Bhubaneswar Just Call 9907093804 Top Class Call Girl Service Avail...Call Girls Bhubaneswar Just Call 9907093804 Top Class Call Girl Service Avail...
Call Girls Bhubaneswar Just Call 9907093804 Top Class Call Girl Service Avail...
 
♛VVIP Hyderabad Call Girls Chintalkunta🖕7001035870🖕Riya Kappor Top Call Girl ...
♛VVIP Hyderabad Call Girls Chintalkunta🖕7001035870🖕Riya Kappor Top Call Girl ...♛VVIP Hyderabad Call Girls Chintalkunta🖕7001035870🖕Riya Kappor Top Call Girl ...
♛VVIP Hyderabad Call Girls Chintalkunta🖕7001035870🖕Riya Kappor Top Call Girl ...
 
Best Rate (Hyderabad) Call Girls Jahanuma ⟟ 8250192130 ⟟ High Class Call Girl...
Best Rate (Hyderabad) Call Girls Jahanuma ⟟ 8250192130 ⟟ High Class Call Girl...Best Rate (Hyderabad) Call Girls Jahanuma ⟟ 8250192130 ⟟ High Class Call Girl...
Best Rate (Hyderabad) Call Girls Jahanuma ⟟ 8250192130 ⟟ High Class Call Girl...
 
(👑VVIP ISHAAN ) Russian Call Girls Service Navi Mumbai🖕9920874524🖕Independent...
(👑VVIP ISHAAN ) Russian Call Girls Service Navi Mumbai🖕9920874524🖕Independent...(👑VVIP ISHAAN ) Russian Call Girls Service Navi Mumbai🖕9920874524🖕Independent...
(👑VVIP ISHAAN ) Russian Call Girls Service Navi Mumbai🖕9920874524🖕Independent...
 
Call Girls Varanasi Just Call 9907093804 Top Class Call Girl Service Available
Call Girls Varanasi Just Call 9907093804 Top Class Call Girl Service AvailableCall Girls Varanasi Just Call 9907093804 Top Class Call Girl Service Available
Call Girls Varanasi Just Call 9907093804 Top Class Call Girl Service Available
 
(Rocky) Jaipur Call Girl - 09521753030 Escorts Service 50% Off with Cash ON D...
(Rocky) Jaipur Call Girl - 09521753030 Escorts Service 50% Off with Cash ON D...(Rocky) Jaipur Call Girl - 09521753030 Escorts Service 50% Off with Cash ON D...
(Rocky) Jaipur Call Girl - 09521753030 Escorts Service 50% Off with Cash ON D...
 
Call Girls Kochi Just Call 9907093804 Top Class Call Girl Service Available
Call Girls Kochi Just Call 9907093804 Top Class Call Girl Service AvailableCall Girls Kochi Just Call 9907093804 Top Class Call Girl Service Available
Call Girls Kochi Just Call 9907093804 Top Class Call Girl Service Available
 

1. AIIMS NOVEMBER 2015 QUESTIONS & ANSWERS

  • 1. AIIMS NOVEMBER – 2015 QUESTIONS & EXPLANATORY ANSWERS (INCLUDING IMAGE BASED QUESTIONS) BY:DR MANJUNATH, MBBS MD (MAULANAAZAD MEDICAL COLLEGE), ALL INDIAFACULTY DIRECTOR, DOCTORSACADEMY, DAVANAGERE ANDSHIVAMOGGA . REGULAR COURSE/FOUNDATION COURSE, CONTACT: 9686252725, 9632241911 .WWW.DOCTORSACADEMYDVG.COM DR MANJUNATH, DOCTORS ACADEMY, DAVANAGERE 1
  • 2. AIIMS NOV 2015-QUESTIONS & ANSWERS WITH EXPLANATIONS-SOLVED BY DR: MANJUNATH, MBBS, MD (MAMC NEW DELHI) DIRECTOR DOCTORS ACADEMY DAVANAGERE & SHIMOGA (www.doctorsacademydvg.com) ANATOMY 1. WHICH OF THE FOLLOWING MUSLE HELPS IN PROTRUSION OF MANDIBLE? A) TEMPORALIS B)LATERAL PTERYGOID C) MEDIAL PTERYGOID D)BUCCINATOR • ANSWER: B) LATERAL PTERIGOID DR MANJUNATH, DOCTORS ACADEMY, DAVANAGERE 2
  • 3. AIIMS MAY 2015-QUESTIONS & ANSWERS WITH EXPLANATIONS-SOLVED BY DR: MANJUNATH, MBBS, MD (MAMC NEW DELHI) DIRECTOR DOCTORS ACADEMY DAVANAGERE & SHIMOGA (www.doctorsacademydvg.com)- • THE LATERAL PTERYGOID MUSCLE IS ALSO KNOWN AS PTERYGOIDEUS EXTERNUS OR EXTERNAL PTERYGOID MUSCLE. • • GROSS ANATOMY • THE LATERAL PTERYGOID IS A SHORT, THICK MUSCLE, SOMEWHAT CONICAL IN FORM, WHICH EXTENDS ALMOST HORIZONTALLY, POSTERIORLY AND LATERALLY BETWEEN THE INFRATEMPORAL FOSSA AND THE CONDYLE OF THE MANDIBLE . IT ARISES BY TWO HEADS: AN UPPER (SUPERIOR) AND A LOWER (INFERIOR). • THE SUPERIOR PART ARISES FROM THE LOWER PART OF THE LATERAL SURFACE OF THE GREATER WING OF THE SPHENOID AND FROM THE INFRATEMPORAL CREST. IT INSERTS IN THE TMJ CAPSULE AND TMJ DISC . • THE INFERIOR PART ARISES FROM THE LATERAL SURFACE OF THE LATERAL PTERYGOID PLATE AND INSERTS INTO A DEPRESSION IN FRONT OF THE NECK OF THE CONDYLE OF THE MANDIBLE; THE PTERYGOID FOVEA. DR MANJUNATH, DOCTORS ACADEMY, DAVANAGERE 3
  • 4. AIIMS MAY 2015-QUESTIONS & ANSWERS WITH EXPLANATIONS-SOLVED BY DR: MANJUNATH, MBBS, MD (MAMC NEW DELHI) DIRECTOR DOCTORS ACADEMY DAVANAGERE & SHIMOGA (www.doctorsacademydvg.com)- • ACTIONS OF LATERAL PTERIGOID • THE SUPERIOR PART IS ACTIVE DURING RETRUSION (OPPOSITE OF PROTRUSION) AND IPSILATERAL JAW MOVEMENT. IT IS ALSO ESSENTIAL IN PULLING THE CAPSULE AND DISC FORWARD DURING MOUTH OPENING, THEREBY MAINTAINING NORMAL RELATIONSHIP BETWEEN THE CONDYLE OF THE MANDIBLE AND THE TMJ DISC. • THE INFERIOR PART IS RESPONSIBLE FOR OPENING OF THE MOUTH, PROTRUSION AND CONTRALATERAL JAW MOVEMENT. • HYPERACTIVITY OF THE LPM MUSCLE HAS BEEN DESCRIBED IN TMJ INTERNAL DERANGEMENT , ESPECIALLY WITH LONGSTANDING ANTERIOR DISPLACEMENT OF THE DISC WITHOUT RECAPTURE. THICKENING OF THE TENDON (INFERIOR PART) CAN GIVE RISE TO THE "DOUBLE DISC SIGN". • NERVE SUPPLY: MANDIBULAR NERVE DR MANJUNATH, DOCTORS ACADEMY, DAVANAGERE 4
  • 5. AIIMS MAY 2015-QUESTIONS & ANSWERS WITH EXPLANATIONS-SOLVED BY DR: MANJUNATH, MBBS, MD (MAMC NEW DELHI) DIRECTOR DOCTORS ACADEMY DAVANAGERE & SHIMOGA (www.doctorsacademydvg.com) DR MANJUNATH, DOCTORS ACADEMY, DAVANAGERE MUSCLES NERVE SUPPLY ACTION SPECIAL REMARKS MASSETER ANTERIOR DIVISION OF MANDIBULAR NERVE ELEVATES THE MANDIBLE RETRACTS THE MANDIBLE SIDE MOVEMENT EFFECTOR OF JAW JERK IT IS HYPERTROPHIED IN BRUXISM TEMPORALIS ANTERIOR DIVISION OF MANDIBULAR NERVE ELEVATES MANDIBLE PROTRACTION & SIDE TO SIDE MOVEMENT. IT IS A FAN SHAPED MUSCLE LATERAL PTERYGOID ANTRIOR DIVISION OF MANDIBULAR NERVE MOST IMPORTANT MUSCLE FOR OPENING OF MOUTH ALONG WITH SUPRAHYOID MUSCLES LEFT & RIGHT MUSCLES CONTRACT TOGETHER TO CAUSE PROTRUSION OF JAW IF ONLY ONE LATERAL PTERYGOID CONTRACTS, JAW IS PULLED MEDIALLY TOWARD THE OPPOSITE SIDE. MEDIAL PTERYGOID BRANCH FROM TRUNK OF MANDIBULAR NERVE CONTRACTION OF LATERLA & MEDIAL PTERIGOID TOGETHER PROVIDE HELP IN GRINDING FOOD BETWEEN TEETH OF THE SAME SIDE. NERVE TO MEIDAL PTERIGOID SUPPLIES TENSOR VELI PALATINI & TENSOR TYMPANI THROUGH OTIC GANGLION 5
  • 6. AIIMS MAY 2015-QUESTIONS & ANSWERS WITH EXPLANATIONS-SOLVED BY DR: MANJUNATH, MBBS, MD (MAMC NEW DELHI) DIRECTOR DOCTORS ACADEMY DAVANAGERE & SHIMOGA (www.doctorsacademydvg.com) 2. AXILLARY NERVE IS NOT INJURED IN WHICH OF THE FOLLOWING CONDITIONS? A. FRACTURE OF SURGICAL NECK HUMERUS B. INTRAMUSCULAR INJECTION C. IMPROPER USE OF CRUTCHES D. SHOULDER DISLOCATION • ANS: C. IMPROPER USE OF CRUTCHES • REF: SNELL’S 9ED/354 DR MANJUNATH, DOCTORS ACADEMY, DAVANAGERE 6
  • 7. AIIMS MAY 2015-QUESTIONS & ANSWERS WITH EXPLANATIONS-SOLVED BY DR: MANJUNATH, MBBS, MD (MAMC NEW DELHI) DIRECTOR DOCTORS ACADEMY DAVANAGERE & SHIMOGA (www.doctorsacademydvg.com): AXILLARY NERVE • THE AXILLARY NERVE IS ONE OF FIVE TERMINAL BRANCHES OF THE BRACHIAL PLEXUS, SUPPLYING MOTOR AND SENSORY BRANCHES TO THE SHOULDER. • • SUMMARY • ORIGIN: POSTERIOR CORD OF THE BRACHIAL PLEXUS • COURSE: PASSES OUT OF AXILLA THROUGH THE QUADRANGULAR SPACE TO THE UPPER ARM • MAJOR BRANCHES: SUPERIOR LATERAL CUTANEOUS NERVE OF THE ARM • MOTOR SUPPLY: DELTOID AND TERES MINOR MUSCLES • SENSORY SUPPLY: SKIN OVERLYING DELTOID MUSCLE DR MANJUNATH, DOCTORS ACADEMY, DAVANAGERE 7
  • 8. AIIMS MAY 2015-QUESTIONS & ANSWERS WITH EXPLANATIONS-SOLVED BY DR: MANJUNATH, MBBS, MD (MAMC NEW DELHI) DIRECTOR DOCTORS ACADEMY DAVANAGERE & SHIMOGA (www.doctorsacademydvg.com) DR MANJUNATH, DOCTORS ACADEMY, DAVANAGERE 8
  • 9. AIIMS MAY 2015-QUESTIONS & ANSWERS WITH EXPLANATIONS-SOLVED BY DR: MANJUNATH, MBBS, MD (MAMC NEW DELHI) DIRECTOR DOCTORS ACADEMY DAVANAGERE & SHIMOGA (www.doctorsacademydvg.com) DR MANJUNATH, DOCTORS ACADEMY, DAVANAGERE HIGH YIELDING INFORMATION: NERVE DAMAGED EFFECT OF LESION UPPER TRUNK OF BRACHIAL PLEXUS (ERB’S PALSY) POLICEMAN’S TIP HAND OR PORTER’S TIP HAND LOWER TRUNK OF BRACHIAL PEXUS (KLUMPKE’S PALSY) CLAW HAND, HORNER’S SYNDROME LONG THORACIC NERVE WINGED SCAPULA MEDIAN POINTING INDEX, APE THUMB UNLAR CLAW HAND (ULNAR CLAW HAND) MEIDAN + ULNAR CLAW HAND (COMPLETE CLAW HAND) RADIAL WRIST DROP 9
  • 10. AIIMS MAY 2015-QUESTIONS & ANSWERS WITH EXPLANATIONS-SOLVED BY DR: MANJUNATH, MBBS, MD (MAMC NEW DELHI) DIRECTOR DOCTORS ACADEMY DAVANAGERE & SHIMOGA (www.doctorsacademydvg.com) 3. BUCCINATOR IS PIERCED BY ALL OF THE FOLLOWING EXCEPT ? A. BUCCAL BRANCH OF MANDIBULAR B. LABIAL BRANCH OF FACIAL ARTERY C. PAROTID DUCT D. MUCUS GLAND OF MOLARS ANS: B. LABIAL BRANCH OF FACIAL ARTERY (Ref: BDC vol3, 6ed/p63) DR MANJUNATH, DOCTORS ACADEMY, DAVANAGERE 10
  • 11. AIIMS MAY 2015-QUESTIONS & ANSWERS WITH EXPLANATIONS-SOLVED BY DR: MANJUNATH, MBBS, MD (MAMC NEW DELHI) DIRECTOR DOCTORS ACADEMY DAVANAGERE & SHIMOGA (www.doctorsacademydvg.com) DR MANJUNATH, DOCTORS ACADEMY, DAVANAGERE STRUCTURES PIERCING BUCCINATOR ARE: PAROTID DUCT: PAROTID DUCT BUCCAL BRANCH OF MANDIBULAR NERVE 4-5 MOLAR MUCUS GLANDS LYING ON BUCCOPHARYNGEAL FASCIA AROUND PAROTID DUCT EMERGES AT ANTERIOR BORDER OF GLAND RUNS OVER MASSETER PIERCES BUCCINATOR TO ENTER MOUTH OPPOSITE UPPER SECOND MOLAR 11
  • 12. AIIMS MAY 2015-QUESTIONS & ANSWERS WITH EXPLANATIONS-SOLVED BY DR: MANJUNATH, MBBS, MD (MAMC NEW DELHI) DIRECTOR DOCTORS ACADEMY DAVANAGERE & SHIMOGA (www.doctorsacademydvg.com) 4. WHICH OF THE FOLLOWING ARTERY DOES NOT SUPPLY RECTUM? A. SUPERIOR RECTAL ARTERY B. INFERIOR RECTAL ARTERY C. MIDDLE RECTAL ARTERY D. MIDDLE COLIC ARTERY ANS: D. MIDDLE COLIC ARTERY Ref: BDC vol2, 6th ed/267-269 DR MANJUNATH, DOCTORS ACADEMY, DAVANAGERE 12
  • 13. AIIMS MAY 2015-QUESTIONS & ANSWERS WITH EXPLANATIONS-SOLVED BY DR: MANJUNATH, MBBS, MD (MAMC NEW DELHI) DIRECTOR DOCTORS ACADEMY DAVANAGERE & SHIMOGA (www.doctorsacademydvg.com) DR MANJUNATH, DOCTORS ACADEMY, DAVANAGERE BLOOD SUPPLY OF RECTUM: NAME OF ARTERY ARISES FROM SUPPLIES SUPERIOR-RECTAL A INFERIOR MESENTERIC ARTRY MAIN SUPPLY FOR UPPER 2/3 OF THE RECTUM MIDDLE RECTAL A ANT DIV OF INTERNAL ILIAC ARTERY MIDDLE THRID OF RECTUM INFERIOR RECTAL A AORTA DISTAL THIRD OF RECTUM MEDIAN SACRAL A TERMINAL MIDLINE BRANCH OF THE AORTA POSTERIOR WALL OF ANORECTAL JUNCTION ON SACRORECTAL FASCIA 13
  • 14. AIIMS MAY 2015-QUESTIONS & ANSWERS WITH EXPLANATIONS-SOLVED BY DR: MANJUNATH, MBBS, MD (MAMC NEW DELHI) DIRECTOR DOCTORS ACADEMY DAVANAGERE & SHIMOGA (www.doctorsacademydvg.com) 5. IDENTIFY VAGUS NERVE IN THE GIVEN DIAGRAM OF TRANSVERSE SECTION OF THORAX? A. VAGUS NERVE B. AORTA C. THORACIC DUCT D. IVC Ans: A VAGUS NERVE Ref: Netter’s Anatomy Atlas 5th/27 DR MANJUNATH, DOCTORS ACADEMY, DAVANAGERE 14
  • 15. AIIMS MAY 2015-QUESTIONS & ANSWERS WITH EXPLANATIONS-SOLVED BY DR: MANJUNATH, MBBS, MD (MAMC NEW DELHI) DIRECTOR DOCTORS ACADEMY DAVANAGERE & SHIMOGA (www.doctorsacademydvg.com): HIGH YIELDING POINTS DR MANJUNATH, DOCTORS ACADEMY, DAVANAGERE LONGEST INTRACRANIAL COURSE → TROCHLEAR NERVE (ALL INDIA REPEAT) LONGEST COURSE OVERALL AND MOST WIDELY DISTRIBUTED → VAGUS NERVE SMALLEST (THINNEST) CRANIAL NERVE → TROCHLEAR NERVE LARGEST (THICKEST) CRANIAL NERVE → TRIGEMINAL NERVE CRANIAL NERVES CARRYING PARASYMPATHETIC FIBERS → 3, 7, 9, 10 THE ONLY CRANIAL NERVE DECUSSATING COMPLETELY BEFORE EMERGING → TROCHLEAR NERVE COMMONEST CRANIAL NERVE AFFECTED IN SPINAL ANESTHESIA → ABDUCENT NERVE CRANIAL NERVE MOST COMMONLY INVOLVED IN INTRACRANIAL IN INTRACRANIAL ANEURYSM → OCULOMOTOR NERVE CRANIAL NERVE MOST COMMONLY INVOLVED IN BASED SKULL FRACTURE → FACIAL NERVE CRANIAL NERVE MOST COMMONLY INVOLVED IN RAISED INTRACRANIAL TESNION → ABDUCENT NERVE 15
  • 16. 6. WHICH OF THE FOLLOWING IS NOT INVOLVED IN FORMATION OF THORACIC DIAPHRAGM? A. CENTRAL TENDON B. BODY WALL C. PERICARDIOPERITONEAL FOLD D. SOMATIC PLEURIPERITONEUM ANS: C. PERICARDIOPERITONEAL FOLD REF: BDC 6TH ED, VOL1/193-194 DR MANJUNATH, DOCTORS ACADEMY, DAVANAGERE AIIMS MAY 2015-QUESTIONS & ANSWERS WITH EXPLANATIONS-SOLVED BY DR: MANJUNATH, MBBS, MD (MAMC NEW DELHI) DIRECTOR DOCTORS ACADEMY DAVANAGERE & SHIMOGA (www.doctorsacademydvg.com) 16
  • 17. AIIMS MAY 2015-QUESTIONS & ANSWERS WITH EXPLANATIONS-SOLVED BY DR: MANJUNATH, MBBS, MD (MAMC NEW DELHI) DIRECTOR DOCTORS ACADEMY DAVANAGERE & SHIMOGA (www.doctorsacademydvg.com): EXTRA EDGE DR MANJUNATH, DOCTORS ACADEMY, DAVANAGERE EMBRYONIC STRUCTURE PART OF DIAPHRAGM SEPTUM TRANSVERSUM STERNAL PART & CENTRAL TENDINOUS REGION PLEUROPERITONEAL MEMBRANES DORSAL PAIRED PORTION DORSAL MESENTARY OF ESOPHAGUS UNPAIRED CRURAL PART LATERAL BODY WALLS PERIPHERAL COSTAL PORTION 17
  • 18. AIIMS MAY 2015-QUESTIONS & ANSWERS WITH EXPLANATIONS-SOLVED BY DR: MANJUNATH, MBBS, MD (MAMC NEW DELHI) DIRECTOR DOCTORS ACADEMY DAVANAGERE & SHIMOGA (www.doctorsacademydvg.com) DR MANJUNATH, DOCTORS ACADEMY, DAVANAGERE MAJOR OPENINGS IN DIAPHRAGM: SIM: SAVING INTO MEMEORY OPENING PART OF DIAPHRAGM SHAPE STRUCTURES PASSING VENAE CAVAL T8 CENTRAL TENDON QUADRILATE RAL 1.INFERIOR VENA CAVA 2. BRANCHES OF RIGHT PHRENIC NERVE OESOPHAGEAL T8 MUSCULAR PORTION DERIVED FROM RIGHT CRUS ELLIPTICAL 1.OESOPHAGUS 2.OESOPHAGUS BRANCH OF LEFT GASTRIC VESSELS 3. VAGUS/GASTRIC NERVES AORTIC (OSSEOAPANEUROTIC) T12 BETWEEN RIGHT AND LEFT CRUS (POSTERIOR TO DIAPHRAGM) ROUND 1. AORTA 2. THORACIC DUCT 3. AZYGOUS VEIN 18
  • 19. 7. 8 YEAR OLD BOY PRESENTED TO OPD WITH HYPEREXTENSION OF METACARPO PHALANGEAL JOINT AND FLEXION OF DISTAL PHALAGEAL JOINT OF LITTLE AND RING FINGER. WHICH NERVE IS INVOLVED? A. ULNAR NERVE B. RADIAL NERVE C. ANTERIOR INTEROSSEOUS NERVE D. POSTERIOR INTEROSSEOUS NERVE ANS- A. ULNAR NERVE (DIAGNOSIS-ULNAR CLAW HAND) Ref: BDC 5th/e vol 1, p110-111 DR MANJUNATH, DOCTORS ACADEMY, DAVANAGERE AIIMS MAY 2015-QUESTIONS & ANSWERS WITH EXPLANATIONS-SOLVED BY DR: MANJUNATH, MBBS, MD (MAMC NEW DELHI) DIRECTOR DOCTORS ACADEMY DAVANAGERE & SHIMOGA (www.doctorsacademydvg.com) 19
  • 20. AIIMS MAY 2015-QUESTIONS & ANSWERS WITH EXPLANATIONS-SOLVED BY DR: MANJUNATH, MBBS, MD (MAMC NEW DELHI) DIRECTOR DOCTORS ACADEMY DAVANAGERE & SHIMOGA (www.doctorsacademydvg.com): ULNAR NERVE-EVERY THING YOU SHOULD KNOW DR MANJUNATH, DOCTORS ACADEMY, DAVANAGERE THE ULNAR NERVE IS ONE OF THE TERMINAL BRANCHES OF THE BRACHIAL PLEXUS AND HAS MOTOR AND SENSORY SUPPLY TO THE FOREARM AND HAND. • GROSS ANATOMY • ORIGIN • THE ULNAR NERVE ORIGINATES AS A TERMINAL BRANCH OF THE MEDIAL CORD OF THE BRACHIAL PLEXUS WITH NERVE ROOT FIBRES FROM C8-T1. • COURSE • ARM • IN THE ARM, THE ULNAR NERVE RUNS MEDIAL TO THE AXILLARY ARTERY AND SUBSEQUENTLY THE BRACHIAL ARTERY ON THE CORACOBRACHIALIS MUSCLE IN THE ANTERIOR COMPARTMENT. THE NERVE PASSES TO THE POSTERIOR COMPARTMENT THROUGH THE MEDIAL INTERMUSCULAR SEPTUM DISTALLY RUNNING WITH THE SUPERIOR ULNAR COLLATERAL ARTERY. FURTHER ON, IT RUNS BETWEEN MEDIAL HEAD OF TRICEPS BRACHII MUSCLE AND THE MEDIAL INTERMUSCULAR SEPTUM TO PASS POSTERIOR TO THE MEDIAL HUMERAL EPICONDYLE IN THE SUPERFICIAL CONDYLAR GROOVE (CUBITAL TUNNEL). 20
  • 21. AIIMS MAY 2015-QUESTIONS & ANSWERS WITH EXPLANATIONS-SOLVED BY DR: MANJUNATH, MBBS, MD (MAMC NEW DELHI) DIRECTOR DOCTORS ACADEMY DAVANAGERE & SHIMOGA (www.doctorsacademydvg.com): ULNAR NERVE DR MANJUNATH, DOCTORS ACADEMY, DAVANAGERE • FOREARM • THE ULNAR NERVE ENTERS THE FOREARM FROM THE ARM VIA THE TWO HEADS OF FLEXOR CARPI ULNARIS (FCU) MUSCLE. IT SUBSEQUENTLY LIES SUPERFICIAL TO FLEXOR DIGITORUM PROFUNDUS AND DEEP TO FCU AND MEDIAL TO THE ULNAR ARTERY. AT THE WRIST, THE ULNAR NERVE RUNS LATERAL TO THE TENDON OF FCU. • HAND • THE ULNAR NERVE ENTERS THE HAND SUPERFICIAL TO THE FLEXOR RETINACULUM AND INSIDE GUYON'S CANAL. THEN IT DIVIDES INTO ITS TERMINAL BRANCHES AT THE LEVEL OF THE PISIFORM BONE. • PRIOR TO PASSING THE FLEXOR RETINACULUM AT THE WRIST, THE ULNAR NERVE GIVES OFF THE DORSAL CUTANEOUS BRANCH. 21
  • 22. AIIMS MAY 2015-QUESTIONS & ANSWERS WITH EXPLANATIONS-SOLVED BY DR: MANJUNATH, MBBS, MD (MAMC NEW DELHI) DIRECTOR DOCTORS ACADEMY DAVANAGERE & SHIMOGA (www.doctorsacademydvg.com) DR MANJUNATH, DOCTORS ACADEMY, DAVANAGERE CUBITAL TUNNEL SYNDROME IS A TYPE OF ULNAR NERVE COMPRESSION NEUROPATHY WHICH CAN OCCUR FROM A PATHOLOGICAL COMPRESSION OF THE ULNAR NERVE AT THE CUBITAL TUNNEL. • EPIDEMIOLOGY • IT IS THE SECOND MOST COMMON PERIPHERAL NEUROPATHY OF THE UPPER EXTREMITY. • CLINICAL PRESENTATION • ULNAR NERVE COMPRESSION CAN RESULT IN ALTERED SENSATION IN THE LITTLE AND RING FINGERS. IN MANY PATIENTS, SENSORY LOSS IS OFTEN THE FIRST SYMPTOM TO BE REPORTED. AS THE CONDITION PROGRESSES, THEY MAY BE CLUMSINESS IN THE HAND, AS THE ULNAR NERVE IS THE PRINCIPAL MOTOR SUPPLY TO THE INTRINSIC MUSCLES OF THE HAND. IN WELL‐ESTABLISHED CASES, THERE MAY BE MARKED WASTING OF THE SMALL MUSCLES OF THE HAND AND THE ULNAR‐SIDED MUSCLES OF THE FOREARM. 22
  • 23. AIIMS MAY 2015-QUESTIONS & ANSWERS WITH EXPLANATIONS-SOLVED BY DR: MANJUNATH, MBBS, MD (MAMC NEW DELHI) DIRECTOR DOCTORS ACADEMY DAVANAGERE & SHIMOGA (www.doctorsacademydvg.com) DR MANJUNATH, DOCTORS ACADEMY, DAVANAGERE 23
  • 24. AIIMS MAY 2015-QUESTIONS & ANSWERS WITH EXPLANATIONS-SOLVED BY DR: MANJUNATH, MBBS, MD (MAMC NEW DELHI) DIRECTOR DOCTORS ACADEMY DAVANAGERE & SHIMOGA (www.doctorsacademydvg.com): EXTRA EDGE DR MANJUNATH, DOCTORS ACADEMY, DAVANAGERE 24
  • 25. AIIMS MAY 2015-QUESTIONS & ANSWERS WITH EXPLANATIONS-SOLVED BY DR: MANJUNATH, MBBS, MD (MAMC NEW DELHI) DIRECTOR DOCTORS ACADEMY DAVANAGERE & SHIMOGA (www.doctorsacademydvg.com) DR MANJUNATH, DOCTORS ACADEMY, DAVANAGERE 25
  • 26. 8. INCORRECT REGARDING LOCATION OF OTIC GANGLION IS? A. ANTERIOR TO MIDDLE MENINGEAL ARTERY B. LATERAL TO MANDIBULAR NERVE C. INFERIOR TO FORAMEN OVALE D. LATERAL TO TENSOR VELI PALATINE • ANS: B. LATERAL TO MANDIBULAR NERVE ( OTIC GANGLION IS MEDIAL TO MANDIBULAR NERVE) • Ref: Gray’s 3rd/988-989 DR MANJUNATH, DOCTORS ACADEMY, DAVANAGERE AIIMS MAY 2015-QUESTIONS & ANSWERS WITH EXPLANATIONS-SOLVED BY DR: MANJUNATH, MBBS, MD (MAMC NEW DELHI) DIRECTOR DOCTORS ACADEMY DAVANAGERE & SHIMOGA (www.doctorsacademydvg.com) 26
  • 27. AIIMS MAY 2015-QUESTIONS & ANSWERS WITH EXPLANATIONS-SOLVED BY DR: MANJUNATH, MBBS, MD (MAMC NEW DELHI) DIRECTOR DOCTORS ACADEMY DAVANAGERE & SHIMOGA (www.doctorsacademydvg.com) DR MANJUNATH, DOCTORS ACADEMY, DAVANAGERE OTIC GANGLION : IT IS A SMALL, OVAL, REDDISH-GREY GANGLION SITUATED JUST BELOW FORAMEN OVALE. IT IS A PERIPHERAL PARASYMPATHETIC GANGLION RELATED TOPOGRAPHICALLY TO MANDIBULAR NERV, BUT FUNCTIONALLY WITH GLSSOPHARYNGEAL NERVE. NEAR ITS JUNCTION WITH TRIGEMINAL MOTOR ROOT, MANDIBULAR NERVE LIES LATERAL TO IT. TENSOR VELI PALATINI LIES MEDIALLY, SEPARATING THE GANGLION FROM CARTILAGINOUS PART OF THE PHARYNGOTYMPANIC TUBE MIDDLE MENINGEAL ARTERY IS POSTERIOR TO THE GANGLION. OTIC GANGLION USUALLY SURROUNDS THE ORIGIN OF THE NERVE TO MEDIAL PTERYGOID. 27
  • 28. AIIMS MAY 2015-QUESTIONS & ANSWERS WITH EXPLANATIONS-SOLVED BY DR: MANJUNATH, MBBS, MD (MAMC NEW DELHI) DIRECTOR DOCTORS ACADEMY DAVANAGERE & SHIMOGA (www.doctorsacademydvg.com) DR MANJUNATH, DOCTORS ACADEMY, DAVANAGERE 28
  • 29. 9. TRUE STATEMENT ABOUT OSTEOBLASTS ARE ALL EXCEPT? A. DERIVED FROM OSTEOPROGENITOR B. EXPRESS NEUROPEPTIDE RECEPTORS C. HAVE SMOOTH PLASMA MEMBRANE OUTLINE D. REGULATED BY BONE MORPHOGENIC FACTOR ANS: C. HAVE SMOOTH PLASMA MEMBRANE OUTLINE Ref: I.B Singh Histology 7/ed/pg 89-90 DR MANJUNATH, DOCTORS ACADEMY, DAVANAGERE AIIMS MAY 2015-QUESTIONS & ANSWERS WITH EXPLANATIONS-SOLVED BY DR: MANJUNATH, MBBS, MD (MAMC NEW DELHI) DIRECTOR DOCTORS ACADEMY DAVANAGERE & SHIMOGA (www.doctorsacademydvg.com) 29
  • 30. 10. ALL OF THE FOLLOWING ARTICULATED ARTICULATE WITH MAXILLA EXCEPT? A. LACRIMAL B. NASAL C. ZYGOMATIC D. SPHENOID ANS: D. SPHENOID (SPHENOID HAS NO ARTICULATION) Ref: Gray’s Anantomy 3rd e/p780-786 DR MANJUNATH, DOCTORS ACADEMY, DAVANAGERE AIIMS MAY 2015-QUESTIONS & ANSWERS WITH EXPLANATIONS-SOLVED BY DR: MANJUNATH, MBBS, MD (MAMC NEW DELHI) DIRECTOR DOCTORS ACADEMY DAVANAGERE & SHIMOGA (www.doctorsacademydvg.com) 30
  • 31. • BIOCHEMISTRY 12. FOR THE TRANSCRIPTION OF SELENO-CYSTEINE, CODON IS ? A. UGA B. UAA C. UAG D. GUA ANS: A. UGA (SELENOCYSTEINE IS 21 ST AMINOACID HELPS IN DELAYING AGEING AND CANCER AND HEART DISEASE) REF: HARPER 29/e p270 DR MANJUNATH, DOCTORS ACADEMY, DAVANAGERE AIIMS NOVEMBER 2015-QUESTIONS & ANSWERS WITH EXPLANATIONS-SOLVED BY DR: MANJUNATH, MBBS, MD (MAMC NEW DELHI) DIRECTOR DOCTORS ACADEMY DAVANAGERE & SHIMOGA (www.doctorsacademydvg.com) 31
  • 32. AIIMS MAY 2015-QUESTIONS & ANSWERS WITH EXPLANATIONS-SOLVED BY DR: MANJUNATH, MBBS, MD (MAMC NEW DELHI) DIRECTOR DOCTORS ACADEMY DAVANAGERE & SHIMOGA (www.doctorsacademydvg.com) DR MANJUNATH, DOCTORS ACADEMY, DAVANAGERE 32
  • 33. DR MANJUNATH, DOCTORS ACADEMY, DAVANAGERE WHAT IS IT? IT IS AN L-AMINO ACID FOUND IN SOME PEROXIDASES & REDUCTASES WHERE IT PARTICIPATES IN CATALYSIS OF ELECTRON TRANSFER REACTIONS. STRUCTURE AS ITS NAME IMPLIES, A SELENIUM ATOM REPLACES THE SULFUR OF ITS STRUCTUAL ANALOG, CYSTEINE. SINCE SELENOCYSTEINE IS INSERTED INTO POLYPEPTIDES DURING TRANSLATION, IT IS COMMONLY REFERRED TO AS “21ST AMINO ACID.” PK Pk3 OF SELENOCYSTEINE, 5.2, IS 3 UNITS LOWER THAN THAT OF CYSTEINE UNIQUE FEATURE UNLIKE THE OTHER 20 GENETICALLY ENCODED AMINO ACIDS, SELENOCYSTEINE IS NOT SPECIFIED BY A SIMPLE THREE-LETTER CODON CLINICAL SIGNIFICANCE: SELENOCYSTEINE IS PRESENT AT THE ACTIVE SITE OF SEVERAL HUMAN ENZYMES THAT CATALYZE REDOX REACTIONS. EXAMPLES: THIOREDOXIN REDUCTASE, GLUTATHIONE PEROXIDASE & DEIODINASE THAT CONVERTS THYROXINE TO THIIODOTHYRONINE. SIGNIFICANTLY , REPLACEMENTOF SELENOCYSTEINE BY CYSTEINE CAN SIGNIFICANTLY DECREASE CATALYTIC ACTIVITY. IMPAIRMENTS IN HUMAN SELENOPROTEINS HAVE BEEN IMPLICATED IN TUMORIGENESIS, ATHEROSCLEROSIS & SELENIUM DEFICIENCY CARDIOMYOPATHY (KESHAN DISEASE). AIIMS NOV 2015-QUESTIONS & ANSWERS WITH EXPLANATIONS-SOLVED BY DR: MANJUNATH, MBBS, MD (MAMC NEW DELHI) DIRECTOR DOCTORS ACADEMY DAVANAGERE & SHIMOGA (www.doctorsacademydvg.com): SELENOCYSTEINE 33
  • 34. 13. WHICH ONE OF THE REACTION DOES NOT REQUIRE BIOTIN? A. ACETYL-COA TO MALONY-COA B. PYRUVATE TO OXALOACETATE C. GLUTAMINE TO GAMMA CARBOXY GLUTAMATE D. PROPIONYL COA TO METHYL MALONYL COA ANS- C. GLUTAMINE TO GAMMA CARBOXY GLUTAMATE Ref: Harper’s 26/e p153-156 DR MANJUNATH, DOCTORS ACADEMY, DAVANAGERE AIIMS MAY 2015-QUESTIONS & ANSWERS WITH EXPLANATIONS-SOLVED BY DR: MANJUNATH, MBBS, MD (MAMC NEW DELHI) DIRECTOR DOCTORS ACADEMY DAVANAGERE & SHIMOGA (www.doctorsacademydvg.com) BIOTIN DEFICIENCY 34
  • 35. DR MANJUNATH, DOCTORS ACADEMY, DAVANAGERE AIIMS MAY 2015-QUESTIONS & ANSWERS WITH EXPLANATIONS-SOLVED BY DR: MANJUNATH, MBBS, MD (MAMC NEW DELHI) DIRECTOR DOCTORS ACADEMY DAVANAGERE & SHIMOGA (www.doctorsacademydvg.com) REACTIONS REQUIRING BIOTIN AS CO-ENZYME: BIOTIN ENZYME BIOCHEMICAL FUNCTION ACETYL-COA CARBOXYLASE LIPID SYNTHESIS FROM ACETATE METHYLMALONYL-COA CARBOXYLTRANSFERASE PROPINONIC ACID SYNTHESIS BY RUMEN BACTERIA 3-METHYLCROTONYL-COA CARBOXYLASE LEUCINE CATABOLISM PROPINYL-COA CARBOXYLASE CONVERSION OF AMINO ACIDS AND PROPIONATE TO GLUCOSE IN THE LIVER PYRUVATE CARBOXYLASE LIVER GLUCONEOGENESIS 35
  • 36. AIIMS NOV 2015-QUESTIONS & ANSWERS WITH EXPLANATIONS-SOLVED BY DR: MANJUNATH, MBBS, MD (MAMC NEW DELHI) DIRECTOR DOCTORS ACADEMY DAVANAGERE & SHIMOGA (www.doctorsacademydvg.com) DR MANJUNATH, DOCTORS ACADEMY, DAVANAGERE 36
  • 37. AIIMS MAY 2015-QUESTIONS & ANSWERS WITH EXPLANATIONS-SOLVED BY DR: MANJUNATH, MBBS, MD (MAMC NEW DELHI) DIRECTOR DOCTORS ACADEMY DAVANAGERE & SHIMOGA (www.doctorsacademydvg.com) DR MANJUNATH, DOCTORS ACADEMY, DAVANAGERE 37
  • 38. AIIMS MAY 2015-QUESTIONS & ANSWERS WITH EXPLANATIONS-SOLVED BY DR: MANJUNATH, MBBS, MD (MAMC NEW DELHI) DIRECTOR DOCTORS ACADEMY DAVANAGERE & SHIMOGA (www.doctorsacademydvg.com) DR MANJUNATH, DOCTORS ACADEMY, DAVANAGERE 38
  • 39. AIIMS NOVEMBER 2015-QUESTIONS & ANSWERS WITH EXPLANATIONS-SOLVED BY DR: MANJUNATH, MBBS, MD (MAMC NEW DELHI) DIRECTOR DOCTORS ACADEMY DAVANAGERE & SHIMOGA (www.doctorsacademydvg.com): BIOTIN FOOD SOURCES DR MANJUNATH, DOCTORS ACADEMY, DAVANAGERE 39
  • 40. 14. HEME IS NOT SYNTHESIZED IN? A. RBC B. OSTEOCYTE C. LIVER PARENCHYMA D. ERYTHROPOETIC PRECURSOR CELLS IN BONE MARROW ANS: A. RBC Ref: Lippincott’s Illustrated Reviews: Biochemistry 4/e p21 DR MANJUNATH, DOCTORS ACADEMY, DAVANAGERE AIIMS MAY 2015-QUESTIONS & ANSWERS WITH EXPLANATIONS-SOLVED BY DR: MANJUNATH, MBBS, MD (MAMC NEW DELHI) DIRECTOR DOCTORS ACADEMY DAVANAGERE & SHIMOGA (www.doctorsacademydvg.com) 40
  • 41. AIIMS NOV 2015-QUESTIONS & ANSWERS WITH EXPLANATIONS-SOLVED BY DR: MANJUNATH, MBBS, MD (MAMC NEW DELHI) DIRECTOR DOCTORS ACADEMY DAVANAGERE & SHIMOGA (www.doctorsacademydvg.com) 15. GLYCOGEN PHOSPHORYLASE IS REGULATED BY ALL EXCEPT? A. PROTEIN KINASE B. CALMODULIN C. C-AMP D. GLYCOGENIN ANS- D. GLYCOGENIN Ref: Harper’s 26/e p145-147 DR MANJUNATH, DOCTORS ACADEMY, DAVANAGERE 41
  • 42. AIIMS MAY 2015-QUESTIONS & ANSWERS WITH EXPLANATIONS-SOLVED BY DR: MANJUNATH, MBBS, MD (MAMC NEW DELHI) DIRECTOR DOCTORS ACADEMY DAVANAGERE & SHIMOGA (www.doctorsacademydvg.com) EXPLANATION: CYCLIC AMP INTEGRATES THE REGULATION OF GLYCOGENOLYSIS & GLCOGENESIS THE PRINCIPAL ENZYMES CONTROLLING GLYCOGEN METABOLISM-GLYCOGEN PHOSPHORYLASE AND GLYCOGEN SYNTHASE-ARE REGULATED BY ALLOSTERIC MECHANISMS AND COVALENT MODIFICATION BY REVERSIBLE PHOSPHORYLATION AND DEPHOSPHOSPHORYLATION IS INCREASED IN RECPONSE TO:  CYCLIC AMP (cAMP) FORMED FROM ATP BY ADENYLYL CYCLASE AT THE INNER SURFACE OF CELL MEMBRANES IN RESPONSE TO HORMONES SUCH AS EPINEPHRINE, NOREPINEPHRINE, AND GLUCAGEN.  cAMP IS HYPDRLYZED BY PHOSPHODIESTERASE, SO TERMINATING HORMONE ACTION; IN LIVER INSULINE INCREASES THE ACTIVITY OF PHOSPHODIESTERASE. DR MANJUNATH, DOCTORS ACADEMY, DAVANAGERE 42
  • 43. AIIMS NOV 2015-QUESTIONS & ANSWERS WITH EXPLANATIONS-SOLVED BY DR: MANJUNATH, MBBS, MD (MAMC NEW DELHI) DIRECTOR DOCTORS ACADEMY DAVANAGERE & SHIMOGA (www.doctorsacademydvg.com) 16. IN A PATIENT WITH LEAD TOXICITY, WHICH HEME PATHWAY INTERMEDIATE WILL INCREASE IN URINE? A. ALA B. PORPHOBILINOGEN C. UROPORPHOBILINOGEN D. COPROPORPHYRIN ANS: A. ALA (Ref: Harper 29/e p309-311) DR MANJUNATH, DOCTORS ACADEMY, DAVANAGERE CHILD WITH LEAD POISONING 43
  • 44. AIIMS NOVEMBER 2015-QUESTIONS & ANSWERS WITH EXPLANATIONS-SOLVED BY DR: MANJUNATH, MBBS, MD (MAMC NEW DELHI) DIRECTOR DOCTORS ACADEMY DAVANAGERE & SHIMOGA (www.doctorsacademydvg.com) EXPLANATION: HEME SYNTHESIS: STARTS WITH SUCCINYL-CoA, DERIVED FROM THE CITRIC ACID CYCLE IN MITOCHONDRIA, AND THE AMINO ACID GLYCINE. PYRIDOXAL PHOSPHATE IS ALSO NECESSARY IN THIS REACTION TO “ACTIVE” GLYCIN. THE PRODUCT OF THE CONDENSATION REACTIOB BETWEEN SUCCINYL-CoA AND GLYCINE IS α-AMINO-β-KETOADIPIC ACID, WHICH IS RAPIDLY DECARBOXYLATED TO FORM α AMINOLEVULINATE (ALA). DR MANJUNATH, DOCTORS ACADEMY, DAVANAGERE • THIS REACTION REQUENCE IS CATALYZED BY ALA SYNTHASE, THE RATE-CONTROLLING ENZYME IN PORPHYRIN BIOSYNTHESIS IN MAMMALIN LIVER. SYNTHESIS OF ALA OCCURS IN MITOCHONDRIA. •IN THE CYTOSOL, TWO MOLECULES OF ALA ARE CONDENSED BY THE ENZYME ALA DEHYDRATASE TO FORM TWO MOLECULES OF WATER AND ONE OF PORPHOBILINOGEN (PBG). •ALA DEHYDRATASE IS A ZINC-CONTAINING ENZYME AND IS SENSITIVE TO INHIBITION BY LEAD, AS CAN OCCUR IN KEAD POISONING. •IT IS FOR THIS REASON THAT ALA GETS INCREASED IN LEAD POISONING. 44
  • 45. AIIMS NOVEMBER 2015-QUESTIONS & ANSWERS WITH EXPLANATIONS-SOLVED BY DR: MANJUNATH, MBBS, MD (MAMC NEW DELHI) DIRECTOR DOCTORS ACADEMY DAVANAGERE & SHIMOGA (www.doctorsacademydvg.com) 17. GLUCOSE 6 PHOSPHATE PRODUCED IN CYTOPLASM OF HEPATOCYTE DOES NOT GET ACTED UPON BY GLUCOSE 6 PHOSPHATASE ENZYME BECAUSE? A. IT IS THEROMODYNAMICALLY VIABLE ONLY WHEN GLUCONEOGENESIS HAS STARTED B. IT REQUIRES ACTIVATION BY PYRUVATE KINASE C. GLUCOSE 6 PHOSPHATASE IS PRESENT IN ENDOPLASMIC RETICULUM D. STERIC HINDERANCE BY ALBUMIN ANS: C. GLUCOSE 6 PHOSPHATASE IS PRESENT IN ENDOPLASMIC RETICULUM (Ref: Chatterjee 5/e p488; Harper 29/e p19-21; 26/e p147) DR MANJUNATH, DOCTORS ACADEMY, DAVANAGERE 45
  • 46. AIIMS MAY 2015-QUESTIONS & ANSWERS WITH EXPLANATIONS-SOLVED BY DR: MANJUNATH, MBBS, MD (MAMC NEW DELHI) DIRECTOR DOCTORS ACADEMY DAVANAGERE & SHIMOGA (www.doctorsacademydvg.com) EXPLANATION: DR MANJUNATH, DOCTORS ACADEMY, DAVANAGERE •THE CONVERSION GLUCOSE 6-PHOSPHATE TO GLUCOSE IS CATALYZED BY GLUCOSE-6-PHOSPHATASE. •IT IS PRESENT IN LIVER AND KIDNEY BUT ABSENT FROM MUSCLE AND ADIPOSE TISSUE, WHICH, THEREFORE, CANNOT EXPORT GLUCOSE INTO THE BLOODSTREAM. •GLUCOSE-6-PHOSPHATASE CONSISTS OF 357 AMINO ACIDS, AND IS ANCHORED TO THE ENENDOPLASMIC RETICULUM (ER) BY NINE TRANSMEMBRANE HELICIES. •ITS N-TERMINAL AND ACTIVE SITE ARE FOUND ON THE LUMEN SIDE OF THE ER AND ITS C-TERMINUS PROJECTS INTO THE CYTOPLASM. •THE TRANSFER OF THE GLUCOSE 6-PHOSPHATE IS CARRIED OUT BY A TRANSPORTER PROTEIN (T1) AND THE ENDOPLASMIC RETICULUM (ER) CONTAINS STRUCTURES ALLOWING THE EXIT OF THE PHOSPHATE GROUP (T2) AND GLUCOSE (T3). 46
  • 47. AIIMS MAY 2015-QUESTIONS & ANSWERS WITH EXPLANATIONS-SOLVED BY DR: MANJUNATH, MBBS, MD (MAMC NEW DELHI) DIRECTOR DOCTORS ACADEMY DAVANAGERE & SHIMOGA (www.doctorsacademydvg.com) 18. WHICH IS NOT GLUCOGENIC? A. ACETYL COA B. PYRUCATE C. LACTATE D. OXALOACETETE ANS- A. ACETYL COA (ACETYL COA ENTERS KREB S CYCLE FOR ATP PRODUCTION) (Ref: Chatterjee 5/e p448: Harper 29/e p19-20; 26/e p153-155) DR MANJUNATH, DOCTORS ACADEMY, DAVANAGERE 47
  • 48. AIIMS NOVEMBER 2015-QUESTIONS & ANSWERS WITH EXPLANATIONS-SOLVED BY DR: MANJUNATH, MBBS, MD (MAMC NEW DELHI) DIRECTOR DOCTORS ACADEMY DAVANAGERE & SHIMOGA (www.doctorsacademydvg.com) EXPLANATION: GLUCONEOGENESIS DR MANJUNATH, DOCTORS ACADEMY, DAVANAGERE WHAT IS IT? IT IS THE TERM USED TO INCLUDE ALL PATHWAYS RESPONSIBLE FOR CONVERTING NON-CARBOHYDRATE PRECUSORS TO GLUCOSE OR GLYCOGEN RESERVES. MAJOR SUB-STRATES GLUCOGENIC AMINO ACIDS, LACTATE, GLYCEROL & PROPIONATE SITE LIVER AND KIDNEY ARE THE MAJOR GLUCONEOGENIC TISSUES. PURPOSE • IT MEETS THE NEEDS OF BODY FOR GLUCOSE WHEN CARBOHYDRATE IS NOT AVAILABLE IN SUFFICIENT AMOUNT FROM DIET OR FROM GLYCOGEN. A SUPPLY OF GLUCOSE IS NECESSARY ESPECIALLY FOR CNS & RBCs. HYPOGLYCEMIA CAUSES BRAIN DYSFUNCTION, GLUCOSE IS ALSO IMPORTANT IN MAINTAINING LEVEL OF INTERMEDIATES OF CITRIC ACID CYCLE GLUCONEOGENESIS ALSO CLEARS LACTATE PRODUCED BY MUSCLE & RBCs AND GLYCEROL PRODUCED BY ADIPOSE TISSUE. 48
  • 49. AIIMS MAY 2015-QUESTIONS & ANSWERS WITH EXPLANATIONS-SOLVED BY DR: MANJUNATH, MBBS, MD (MAMC NEW DELHI) DIRECTOR DOCTORS ACADEMY DAVANAGERE & SHIMOGA (www.doctorsacademydvg.com) 19. WHICH IS NOT A METHOS OF PROTEIN PRECIPITATION? A. SALTING OUT WITH METALS B. ACETONE & ALCOHOL C. CHANGING PH OTHER THAN ISOELECTRIC PH D. TRICHLORO ACETIC ACID ANS: C. CHANGING PH OTHER THAN ISOELECTRIC PH (Ref: Chatterjee 5/e p448: Harper 29/e p19-21) DR MANJUNATH, DOCTORS ACADEMY, DAVANAGERE 49
  • 50. AIIMS MAY 2015-QUESTIONS & ANSWERS WITH EXPLANATIONS-SOLVED BY DR: MANJUNATH, MBBS, MD (MAMC NEW DELHI) DIRECTOR DOCTORS ACADEMY DAVANAGERE & SHIMOGA (www.doctorsacademydvg.com) EXPLANATION: METHODS OF PROTEIN PRECIPITATION: DR MANJUNATH, DOCTORS ACADEMY, DAVANAGERE •SALTING OUT: •EXCESS SALT PRECIPITATES PROTEINS BECAUSE MOST OF WATER MOLECULES BECOME TIED UP IN FORMING HYDRATION SHELLS AROUND SALT IONS. •SO, LESS WATER IS AVAILABLE TO DISSOLVE PROTEINS •AMMONIUM SULFATE IS MOST COMMONLY USED REAGENT FOR SALTING OUT PROTEIN •THE SALT CONCETRATION AT WHICH PROTEIN PRECIPITATES DIFFERS FROM ONE PROTEIN TO ANOTHER. EG GLOBULIN PRECIPITATE AT HALF SATURATION WHILE ALBUMIN PRECIPITATES AT FULL SATURATION OF AMMONIUM SULPHATE. •BY ORGANC SOLVENTS(PROTEIN IS LEAST SOLUBLE AT ISOELECTRIC PH) •BY HEAVY METALS (LEAD, CADMIUM, MERCURY) 50
  • 51. AIIMS NOVEMBER 2015-QUESTIONS & ANSWERS WITH EXPLANATIONS-SOLVED BY DR: MANJUNATH, MBBS, MD (MAMC NEW DELHI) DIRECTOR DOCTORS ACADEMY DAVANAGERE & SHIMOGA (www.doctorsacademydvg.com) PHYSIOLOGY 21. Na-IODIDE TRANSPORTER IS PRESENT IN ALL EXCEPT? A. PITUITARY B. SALIVARY C. PLACENTA D. THYROID ANS- A. PITUITARY (Ref: Ganong 23/e p303) DR MANJUNATH, DOCTORS ACADEMY, DAVANAGERE 51
  • 52. AIIMS NOVEMBER 2015-QUESTIONS & ANSWERS WITH EXPLANATIONS-SOLVED BY DR: MANJUNATH, MBBS, MD (MAMC NEW DELHI) DIRECTOR DOCTORS ACADEMY, DAVANAGERE & SHIMOGA. (www.doctorsacademydvg.com) EXPLANATION: DR MANJUNATH, DOCTORS ACADEMY, DAVANAGERE “THE SALIVARY GLAND, THE GASTRIC MUCOSA, THE PLACENTA, THE CILIARY BODY OF THE EYE, THE CHOROID PLEXUS, THE MAMMARY GLANDS, AND CERTAIN CANCERS DERIVED FROM THESE TISSUES ALSO EXPRESS NA IODIDE TRANSPOTER, WHICH CAN TRANSPORT IODIDE AGAINST A CONCENTRATION GRADIENT.” BUT THE TRANSPORTER IN THESE TISSUES IS NOT AFFECTED BY TSH. THE PHYSIOLOGIC SIGNIFICANCE OF ALL THESE EXTRATHYROIDAL IODIDE- CONCENTRATING MECHANISM IS OBSCURE. THEY MAY PROVIDE PATHWAY FOR RADIOABLATION OF NA IODIDE TRANSPORTER- EXPRESSING CANCER CELLS USING IODIDE RADIOIDOTOPES. THIS APPROACH IS ALSO USEFUL FOR THE ABLATION OF THYROID CANCERS. 52
  • 53. AIIMS NOVEMBER 2015-QUESTIONS & ANSWERS WITH EXPLANATIONS-SOLVED BY DR: MANJUNATH, MBBS, MD (MAMC NEW DELHI) DIRECTOR DOCTORS ACADEMY DAVANAGERE & SHIMOGA (www.doctorsacademydvg.com) 22. ATRIAL NATRIURETIC PEPTIDE EXERTS ITS ACTION ON ALL EXCEPT? A. INHIBITION OF Na+ REABSORPTION IN PCT B. DILATE AFFERENT ARTERIOLE C. MESANGIAL CELL CONTRACTION D. INHIBITION OF Na+ REABSORPTION IN MEDULARRY COLLECTING DUCT ANS: C. MESANGIAL CELL CONTRACTION (ANP is a powerful vasodilator, and a protein (polypeptide) hormone secreted by heart muscle cells. It is involved in the homeostatic control of body water, sodium, potassium and fat (adipose tissue). It is released by muscle cells in the upper chambers (atria) of the heart (atrial myocytes) in response to high blood volume. ANP acts to reduce the water, sodium and adipose loads on the circulatory system). (Ref: Ganong 23/e p675) DR MANJUNATH, DOCTORS ACADEMY, DAVANAGERE 53
  • 54. AIIMS NOVEMBER 2015-QUESTIONS & ANSWERS WITH EXPLANATIONS-SOLVED BY DR: MANJUNATH, MBBS, MD (MAMC NEW DELHI) DIRECTOR DOCTORS ACADEMY DAVANAGERE & SHIMOGA (www.doctorsacademydvg.com) EXPLANATION: ACTIONS OF ATRIAL NATRIURETIC PEPTIDE (ANP) & BRAIN NATRIURETIC PEPTIDE (BNP) DR MANJUNATH, DOCTORS ACADEMY, DAVANAGERE • THEY ACT ON THE KIDNEY TO INCREASE NA+ EXCRETION • THEY APPEAR TO PRODUCE THIS EFFECT BY DILATING AFFERENT ARTERIOLES AND RELAXING MESANGIAL CELLS. BOTH OF THESE ACTIONS INCREASE GLOMERULAR FILTRATION •THEY ACT ON THE RENAL TUBULES TO NHIBIT NA+ REABSORPTION. •OTHER ACTIONS INCLUDE AN INCREASE IN CAPILLARY PERMEABILITY, LEADING TO EXTRAVASATION OF FLUID AND A DECLINE IN BLOOD PRESSURE. •THEY RELAX VASCULAR SMOOTH MUSCLE IN ARTERIOLES AND VENULES 54
  • 55. AIIMS NOV 2015-QUESTIONS & ANSWERS WITH EXPLANATIONS-SOLVED BY DR: MANJUNATH, MBBS, MD (MAMC NEW DELHI) DIRECTOR DOCTORS ACADEMY DAVANAGERE & SHIMOGA (www.doctorsacademydvg.com) 23. INTERSTITIAL FLUID VOLUME CAN BE DETERMINED BY ? A. RADIOACTIVE SODIUM AND RADIOIODINE LABELLED ALBUMIN B. RADIOACTIVE WATER AND RADIOLABELLED ALBUMIN C. RADIOACTIVE SODIUM AND RADIOACTIVE WATER D. DIIODOTHALMIUM IODIONE ANS- A. RADIOACTIVE SODIUM AND RADIOIODINE LABELLED ALBUMIN (Ref: Ganong 24e p3-5, Guyton 12/534-536, 541) DR MANJUNATH, DOCTORS ACADEMY, DAVANAGERE 55
  • 56. AIIMS NOV 2015-QUESTIONS & ANSWERS WITH EXPLANATIONS-SOLVED BY DR: MANJUNATH, MBBS, MD (MAMC NEW DELHI) DIRECTOR DOCTORS ACADEMY DAVANAGERE & SHIMOGA (www.doctorsacademydvg.com) EXPLANATION: MEASUREMENT OF BODY FLUID VOLUME DR MANJUNATH, DOCTORS ACADEMY, DAVANAGERE VOLUME INDICATORS TOTAL BODY WATER 3H2O, 2H2O, ANTIPYRINE EXTRACELLULAR FLUID 22NA, 125I-OTHAMAMATE, THIOSULFATE, INULIN EXTRACELLULAR FLUID (CALCULATED AS TOTAL BODY WATER-EXTRACELLULAR FLUID VOLUME) PLASMA VOLUME 125I-ALBUMIN, EVANS BLUE DYE (T-1824) BLOOD VOLUME 51CR-LABELED RED BLOOD CELLS, OR CALCULATED AS BLOOD VOLUME= PLASMA VOLUME/(1-HEMATOCRIT) INTERSTITIAL FLUID (CALCULATE AS EXTRACELLULAR FLUID VOLUME-PLASM VOLUME) 56
  • 57. AIIMS NOV 2015-QUESTIONS & ANSWERS WITH EXPLANATIONS-SOLVED BY DR: MANJUNATH, MBBS, MD (MAMC NEW DELHI) DIRECTOR DOCTORS ACADEMY DAVANAGERE & SHIMOGA (www.doctorsacademydvg.com) 24. THE CLOT FORMED IN NOT STABLE UNLESS EXTENSIVE CROSS LINKING OCCURS. THIS IS DONE BY? A. PLASMIN B. THROMBIN C. FACTOR XIII D. HMWK ANSWER: C. FACTOR XIII REF: GANONG 23/E P531 57DR MANJUNATH, DOCTORS ACADEMY, DAVANAGERE
  • 58. AIIMS NOV 2015-QUESTIONS & ANSWERS WITH EXPLANATIONS-SOLVED BY DR: MANJUNATH, MBBS, MD (MAMC NEW DELHI) DIRECTOR DOCTORS ACADEMY DAVANAGERE & SHIMOGA (www.doctorsacademydvg.com) 58DR MANJUNATH, DOCTORS ACADEMY, DAVANAGERE
  • 59. AIIMS NOV 2015-QUESTIONS & ANSWERS WITH EXPLANATIONS-SOLVED BY DR: MANJUNATH, MBBS, MD (MAMC NEW DELHI) DIRECTOR DOCTORS ACADEMY DAVANAGERE & SHIMOGA (www.doctorsacademydvg.com) 25. PHYSIOLOGICAL CHANGES IN LAPAROSCOPY INCLUDE ALL EXCEPT? A. INCREASED PCWP B. INCREASED ICP C. DECREASED FRC D. INCREASED PH ANSWER: D. INCREASED PH (THERE IS DECREASE IN PH UE TO METABOLIC ACIDOSIS) REF: WYLIE CBURCBILL-DAVIDSON’S A PRACTICE OF ANAESTHESIA 7/E P896 59DR MANJUNATH, DOCTORS ACADEMY, DAVANAGERE
  • 60. AIIMS NOV 2015-QUESTIONS & ANSWERS WITH EXPLANATIONS-SOLVED BY DR: MANJUNATH, MBBS, MD (MAMC NEW DELHI) DIRECTOR DOCTORS ACADEMY DAVANAGERE & SHIMOGA (www.doctorsacademydvg.com) 26. ALL OF THE STATEMENTS ARE TRUE ABOUT SMOOTH MUSLE CONTRACTION EXCEPT ? A. TENSION DEVELOPED IS PROPORTIONAL TO OR AFFECTED BY FREQUENCY OF SPIKE POTENTIALS B. MUSCLE CONTRACTION IS RELATED TO THE AMPLITUDE OF BASAL ELECTRICAL RHYTHUM (BER) C. FREQUENCY IS AROUND 6/MIN D. THRESHOULD IS -50mV ANS- B. MUSCLE CONTRACTION IS RELATED TO THE AMPLITUDE OF BER (BER ITSELF RARELY CAUSES MUSLE CONTRACTION) (Ref: Ganong 24/e p498-499, 23/e p471) DR MANJUNATH, DOCTORS ACADEMY, DAVANAGERE 60
  • 61. AIIMS NOV 2015-QUESTIONS & ANSWERS WITH EXPLANATIONS-SOLVED BY DR: MANJUNATH, MBBS, MD (MAMC NEW DELHI) DIRECTOR DOCTORS ACADEMY DAVANAGERE & SHIMOGA (www.doctorsacademydvg.com) 29. IN THE FORMULA, CLEARANCE C=U x V/P,U STANDS FOR? A. URINARY CONCENTRATION IN GM/24 HRS B. URINARY CONCENTRATION IN MG/100ML C. MICROALBIMINURIA D. URINE OSMOLARITY ANS- D. URINE OSMOLARITY (REF: GUYTON’S 12/E .663) Ref: Guyton’s 12/e p663 DR MANJUNATH, DOCTORS ACADEMY, DAVANAGERE 61
  • 62. AIIMS NOVEMBER 2015-QUESTIONS & ANSWERS WITH EXPLANATIONS-SOLVED BY DR: MANJUNATH, MBBS, MD (MAMC NEW DELHI) DIRECTOR DOCTORS ACADEMY DAVANAGERE & SHIMOGA (www.doctorsacademydvg.com) 30. ANATOMICAL DEAD SPACE IS MEASURED WITH ? A. SINGLE BREATH NITROGEN TEST OR NITROGEN WASH OUT B. BOHLERSETHOD C. SPIROMETER D. XENON ISOTOPES ANS: A. SINGLE BREATH NITROGEN TEST REF: GUYTON’S 12/E P883-885 (Nitrogen washout (or Fowler's method) is a test for measuring anatomic dead space in the lung during a respiratory cycle , as well as some parameters related to the closure of airways.) DR MANJUNATH, DOCTORS ACADEMY, DAVANAGERE 62
  • 63. AIIMS NOVEMBER 2015-QUESTIONS & ANSWERS WITH EXPLANATIONS-SOLVED BY DR: MANJUNATH, MBBS, MD (MAMC NEW DELHI) DIRECTOR DOCTORS ACADEMY DAVANAGERE & SHIMOGA (www.doctorsacademydvg.com) DR MANJUNATH, DOCTORS ACADEMY, DAVANAGERE 63
  • 64. AIIMS NOVEMBER 2015-QUESTIONS & ANSWERS WITH EXPLANATIONS-SOLVED BY DR: MANJUNATH, MBBS, MD (MAMC NEW DELHI) DIRECTOR DOCTORS ACADEMY DAVANAGERE & SHIMOGA (www.doctorsacademydvg.com) PATHOLOGY 31. HISTOPATHOLOGY OF SCHWANNOMA,FOLLOWING IS SEEN? A. ANTOMY A WITH VEROCAY BODY B. ROSETTES C. ANTOMY B WITH VEROCAY BODY D. PALLISADING ANS: A. ANTOMY A WITH VEROCAY BODY REF:ROBBIN’S 9/E P1314; COMPLETE REVIEW OF PATHOLOGY 1/E P710 DR MANJUNATH, DOCTORS ACADEMY, DAVANAGERE 64
  • 65. AIIMS NOVEMBER 2015-QUESTIONS & ANSWERS WITH EXPLANATIONS-SOLVED BY DR: MANJUNATH, MBBS, MD (MAMC NEW DELHI) DIRECTOR DOCTORS ACADEMY DAVANAGERE & SHIMOGA (www.doctorsacademydvg.com) DR MANJUNATH, DOCTORS ACADEMY, DAVANAGERE FEATURES SCHWANNOMAS NEUROFIBROMAS GENERAL CHARACTERISTICS  BENIGN TUMOR  ARISE DIRECTLY FROM PERIPHERAL NERVES.  BENIGN TUMORS, HETEROGEOUS IN COMPOSITION  NEOPLASTIC SCHWANN CELLS MIXED WITH PERINURIAL LIKE CELLS, FIBROBLASTS, MAST CELLS, AND CD34+ SPINCLE CELLS GROSS  WELL-CIRCUMSCRIBED &ENCAPSULATED  BUT ASSOCIATED NERVE WITHOUT INVADING  NON-ENCAPSULATED MASS MORPHOLOGY CONSISTS OF:  ANTONI A: CELLULAR AREAS  ANTONI B: LOOSE EDEMATOUS AREAS 3 TYPES:  LOCALIZED CUTANEOUS NEURROFIBROMA  DIFFUSE NEUROFIBROMA  PLEXIFORMNEUROFIBROMA 65
  • 66. AIIMS NOVEMBER 2015-QUESTIONS & ANSWERS WITH EXPLANATIONS-SOLVED BY DR: MANJUNATH, MBBS, MD (MAMC NEW DELHI) DIRECTOR DOCTORS ACADEMY DAVANAGERE & SHIMOGA (www.doctorsacademydvg.com): PATHOLOGY OF SCHWANNOMAS • Schwannomas are benign encapsulated neoplasms of schwann cells (WHO grade I). They arise eccentrically from their parent nerve. They are composed of two cell types: Antoni A and Antoni B. • The Antoni A cells are densely packed and arranged in fascicles; acellular areas lie between opposing rows of parallel nuclei (Verocay bodies). • The Antoni B cells are less compact and are prone to cystic degeneration. • PATHOLOGIC VARIANTS INCLUDE : • conventional (most common type): fibrous capsule, hyaline vessels, Antoni A and loose textured Antoni B areas (Verocay bodies) • cellular schwannomas (predominantly Antoni A tissue without Verocay bodies) • melanotic schwannomas (dense melanin pigment) • plexiform schwannomas (do not undergo malignant change, unlike plexiform neurofibromas) DR MANJUNATH, DOCTORS ACADEMY, DAVANAGERE 66
  • 67. AIIMS NOVEMBER 2015-QUESTIONS & ANSWERS WITH EXPLANATIONS-SOLVED BY DR: MANJUNATH, MBBS, MD (MAMC NEW DELHI) DIRECTOR DOCTORS ACADEMY DAVANAGERE & SHIMOGA (www.doctorsacademydvg.com): VESTIBULAR SCHWANNOMA DR MANJUNATH, DOCTORS ACADEMY, DAVANAGERE 67
  • 68. AIIMS NOVEMBER 2015-QUESTIONS & ANSWERS WITH EXPLANATIONS-SOLVED BY DR: MANJUNATH, MBBS, MD (MAMC NEW DELHI) DIRECTOR DOCTORS ACADEMY DAVANAGERE & SHIMOGA (www.doctorsacademydvg.com) 32. A 30 YEAR OLD SOFTWARE ENGINEER CAME TO OPD WITH CHIEF COMPLAINS OF HEARTBURN. ON ENDOSCOPIC BIOPSY, THE LESION SHOWS THE FOLLOWING (FIGURE BELOW). IDENTIFY THE LESION, STAIN HAS DONE FOR THE WHAT AND WHAT ADDITIONAL FEATURES SHOULD BE LOOKED FOR? A. ADENOCACINOMA; PAS; MALIGNANCY B. BARRETTS OESOPHAGUS; MUCIN STAIN; DYSPLASIA C. SQUAMOUS CELL CARCINOMA; CYTOKERATIN, SQUAMOUS PEARLE D. INFECTION; FUNGAL STAIN; INCLUSIO BODY ANS: B. BARRETTS OESOPHAGUS; MUCIN STAIN; DYSPLASIA REF: ROBBIN’S 9/757; COMPLETE REVIEW OF PATHOLOGY 1/E P482 DR MANJUNATH, DOCTORS ACADEMY, DAVANAGEREDR. MANJUNATH, DOCTORS ACADEMY 68
  • 69. AIIMS NOVEMBER 2015-QUESTIONS & ANSWERS WITH EXPLANATIONS-SOLVED BY DR: MANJUNATH, MBBS, MD (MAMC NEW DELHI) DIRECTOR DOCTORS ACADEMY DAVANAGERE & SHIMOGA (www.doctorsacademydvg.com) EXPLANATION: DR MANJUNATH, DOCTORS ACADEMY, DAVANAGERE Barrett esophagus represents progressive metaplasia of oesophageal stratified squamous cell epithelium to columnar epithelium. Although the exact number varies, 90-100% of oesophgeal adenocarcinoma is thought to arise from this metaplasia. Although patients with Barrett oesophagus have a 30x risk of developing oesophageal adenocarcinoma, the annual risk of developing adenocarcinoma depends on the degree of histological dysplasia, but may be ~1% (range 0.1-2%), and the absolute risk is low. 69
  • 70. AIIMS NOVEMBER 2015-QUESTIONS & ANSWERS WITH EXPLANATIONS-SOLVED BY DR: MANJUNATH, MBBS, MD (MAMC NEW DELHI) DIRECTOR DOCTORS ACADEMY DAVANAGERE & SHIMOGA (www.doctorsacademydvg.com) 33. THE GIVEN FIGURE SHOWS WHICH OF THE FOLLOWING? A. AMYLOIDOSIS-GREY; VIABLE WHITE NECROTIC B. NUTMEG LIVER-RED AREAS ARE VIABLE PERICENTRAL AREAS; WHITE AREAS ARE PERIPORTAL NECROTIC AREAS C. RED AREAS ARE NECROTIC AREAS NEAR CENTRAL VEIN, WHITE AREAS ARE VIABLE, FIBROTIC PERIPORTAL AREA D. AMYLOIDOSIS-NECROTIC WHITE PERIPORTAL VIABLE GREY PERICENTRAL AREAS ANS: C. RED AREAS ARE NECROTIC AREAS NEAR CENTRAL VEIN, WHITE AREAS ARE VIABLE, FIBROTIC PERIPORTAL AREA (CHRONIC HEAPTIC CONGESTION) REF: ROBBIN’S 9/129; COMPLETE REVIEW OF PATHOLOGY 1/E P86 DR MANJUNATH, DOCTORS ACADEMY, DAVANAGEREDR. MANJUNATH, DOCTORS ACADEMY 70
  • 71. AIIMS NOVEMBER 2015-QUESTIONS & ANSWERS WITH EXPLANATIONS-SOLVED BY DR: MANJUNATH, MBBS, MD (MAMC NEW DELHI) DIRECTOR DOCTORS ACADEMY DAVANAGERE & SHIMOGA (www.doctorsacademydvg.com) EXPLANATION: DR MANJUNATH, DOCTORS ACADEMY, DAVANAGEREDR. MANJUNATH, DOCTORS ACADEMY ACUTE PULMONARY CONGESTION:  ENGORGED ALVEOLAR CAPILLARIES  ALVEOLAR SEPTALEDEMA  FOCAL INTRA ALVEOLAR HEMORRHAGE CHRONIC PULMONARY CONGESTION  SEPTA ARE THICKENED AND FIBROTIC  HEMOSIDERIN-LADEN MACROPHAGES(HEART FAILURE CELLS) ACUTE HEPATIC CONGESTION  CENTRILOBULAR HEPATOCYTES: ISCHEMIC NECROSIS  PERIPORTAL HEPATOCYTES-FATTY CHANGE CHRONIC PASSIVE HEPATIC CONGESTION  NUTMEG LIVER: CENTRILOBULAR REGIONS ARERED-BROEN AGAINST SURROUNDING ZONES OF UNCONGESTED TAN LIVER  INITIALLY CENTRILOBULAR NECROSIS & HEMOSIDERIN LADEN MACROPHAGES.  LATER: HEPATIC FIBROSIS CALLED CARDIAC CIRRHOSIS. 71
  • 72. AIIMS NOVEMBER 2015-QUESTIONS & ANSWERS WITH EXPLANATIONS-SOLVED BY DR: MANJUNATH, MBBS, MD (MAMC NEW DELHI) DIRECTOR DOCTORS ACADEMY DAVANAGERE & SHIMOGA (www.doctorsacademydvg.com) 34. WHICH OF THE FOLLOWING IS A IMMUNE PRIVILEGED REGION? A. AREA POSTREMA B. SEMINIFEROUS TUBULES C. KIDNEY D. OPTIC NERVE ANS: B. SEMINIFEROUS TUBULES [REF: ROBBIN’S 9/215-216] DR MANJUNATH, DOCTORS ACADEMY, DAVANAGEREDR. MANJUNATH, DOCTORS ACADEMY 72
  • 73. AIIMS NOVEMBER 2015-QUESTIONS & ANSWERS WITH EXPLANATIONS-SOLVED BY DR: MANJUNATH, MBBS, MD (MAMC NEW DELHI) DIRECTOR DOCTORS ACADEMY DAVANAGERE & SHIMOGA (www.doctorsacademydvg.com) EXPLANATION: DR MANJUNATH, DOCTORS ACADEMY, DAVANAGEREDR. MANJUNATH, DOCTORS ACADEMY IMMUNE-PRIVILEGED SITES:  TESTIS, EYE AND BRAIN  TISSUES IN WHICH THESE ANTIGENS ARE LOCATED DO NOT COMMUNICATE WITH THE BLOOD AND LYMPH  DIFFICULT TO INDUCE IMMUNE RESPONSES TO ANTIGEN INTRODUCED INTO THESE SITES  PROLONGED TISSUE INFLAMMATION ON INJURY & RELEASE OF ANTIGEN FROM THESE SITES; POST-TRAUMATIC ORCHITIS & UVEIRIS 73
  • 74. AIIMS NOVEMBER 2015-QUESTIONS & ANSWERS WITH EXPLANATIONS-SOLVED BY DR: MANJUNATH, MBBS, MD (MAMC NEW DELHI) DIRECTOR DOCTORS ACADEMY DAVANAGERE & SHIMOGA (www.doctorsacademydvg.com) 35. PATHOLOGIST EXAMINES BIOPSY FROM A PATIENT PRESENTING WITH BLEEDING PER RECTUM WITH A PAST HISTORY OF INTUSSUPTION FOR THE PAST 6MONTHS. HIS OBTAINED FINDING HAS BEEN SHOWN BELOW. IDENTIFY THE PATHOLOGY? A. TUBULE VILLOUS ADENOMA B. ADENO CARCINOMA Ç. HAMARTOMA D. JUVENILE POLYPOSIS SYNDROME ANS: A. TUBULE VILLOUS ADENOMA REF: ROBBIN’S 9/808-809; COMPLETE REVIEW OF PATHOLOGY 1/E P517 DR MANJUNATH, DOCTORS ACADEMY, DAVANAGEREDR. MANJUNATH, DOCTORS ACADEMY 74
  • 75. AIIMS NOVEMBER 2015-QUESTIONS & ANSWERS WITH EXPLANATIONS-SOLVED BY DR: MANJUNATH, MBBS, MD (MAMC NEW DELHI) DIRECTOR DOCTORS ACADEMY DAVANAGERE & SHIMOGA (www.doctorsacademydvg.com) EXPLANATION: ADENOMAS CAN BE CLASSIFIED AS: (BASED ON THEIR ARCHITECTURE) DR MANJUNATH, DOCTORS ACADEMY, DAVANAGEREDR. MANJUNATH, DOCTORS ACADEMY TUBULAR SMALL, PEDUNCULATED POLYPS COMPOSED OF ROUNDED, OR TUBULAR, GLANDS TUBULO-VILLOUS (PICTURE) MIXTURE OF TUBULAR AND VILLOUS ELEMENTS VILLOUS LARGER AND SESSILE, ARE COVERED BY SLENDER VILLI 75
  • 76. AIIMS NOVEMBER 2015-QUESTIONS & ANSWERS WITH EXPLANATIONS-SOLVED BY DR: MANJUNATH, MBBS, MD (MAMC NEW DELHI) DIRECTOR DOCTORS ACADEMY DAVANAGERE & SHIMOGA (www.doctorsacademydvg.com) 36. WHICH ONE IS BEST PROGNOSTIC FACTOR FOR ALL? A. HYPERPLOIDY B. ORGANOMEGALY C. TLC MORE THAN 50,000/UL D. ENVIROMENTAL FACTORS ANS: A [REF: ROBBIN’S 9/590-592] DR MANJUNATH, DOCTORS ACADEMY, DAVANAGEREDR. MANJUNATH, DOCTORS ACADEMY 76
  • 77. AIIMS NOVEMBER 2015-QUESTIONS & ANSWERS WITH EXPLANATIONS-SOLVED BY DR: MANJUNATH, MBBS, MD (MAMC NEW DELHI) DIRECTOR DOCTORS ACADEMY DAVANAGERE & SHIMOGA (www.doctorsacademydvg.com) EXPLANATION: PROGNOSTIC FACTORS IN ACUTE LYMPHOBLASTIC LEUKEMIA DR MANJUNATH, DOCTORS ACADEMY, DAVANAGEREDR. MANJUNATH, DOCTORS ACADEMY DETERMINANTS FAVORABLE UNFAVORABLE WBC/UL <10,000 >2,00,000 AGE 2-9 YR <1 Y,>10 Y GENDER FEMALE MALE ETHNICITY WHITE BLACK L.NODE, LIVER. SPLEEN ENLARGEMENT ABSENT MASSIVE TESTICULAR ENLARGEMENT ABSENT PRESENT CENTRAL NERVOUS SYSTEM LEUKEMIA ABSENT PRESENT 77
  • 78. AIIMS NOVEMBER 2015-QUESTIONS & ANSWERS WITH EXPLANATIONS-SOLVED BY DR: MANJUNATH, MBBS, MD (MAMC NEW DELHI) DIRECTOR DOCTORS ACADEMY DAVANAGERE & SHIMOGA (www.doctorsacademydvg.com) 37. 10 YEAR CHILD PRESENTED WITH BILATERAL CERVICAL LYMPHADENOPATHY. LYMPHNODE BIOPSY WAS PERFORMED, WHICH SHOWED CELLS AS GIVEN IN THE FIGURE. WHICH OF THE FOLLOWING IS TRUE REGARDING THIS CONDITION? A. HODGKIN LYMPHOMA;EBV AND EMBRYO CELL B. NON HODGKIN LYMPHOMA; HIV AND GIANT B CELL C. TB, MYCOBACTERIA AND TINY GRANULOMA D. HODGKIN LYMPHOMA; EBV AND REED STERNBERG CELL ANS: D. HODGKIN LYMPHOMA; EBV AND REED STERNBERG CELL REF: ROBBIN’S 9/608-611; COMPLETE REVIEW OF PATHOLOGY 1/E P285, 761 DR MANJUNATH, DOCTORS ACADEMY, DAVANAGEREDR. MANJUNATH, DOCTORS ACADEMY 78
  • 79. AIIMS NOVEMBER 2015-QUESTIONS & ANSWERS WITH EXPLANATIONS-SOLVED BY DR: MANJUNATH, MBBS, MD (MAMC NEW DELHI) DIRECTOR DOCTORS ACADEMY DAVANAGERE & SHIMOGA (www.doctorsacademydvg.com) EXPLANATION: Hodgkin’s Lymphoma: DR MANJUNATH, DOCTORS ACADEMY, DAVANAGEREDR. MANJUNATH, DOCTORS ACADEMY • USUALLY PRESENTS AS SOLITARY OR GENERALIZED LYMPHADENOPATHY AND MOST COMMONLY OCCURS IN YOUNG ADULTS, ALTHOUGH ANY AGE GROUP MAY BE AFFECT ED. • THE DISEASE APPEARS TO SPREAD IN A CONTIGUOUS FASHION, AND MOST PATIENT PRESENT WITH DISEASE LIMITED TO THE LYMPH NODES OR TO THE LYMPH NODES AND SPLEEN • THE FIGURE GIVEN IN THIS EQUESTION IS “CLASSIC”HODGKIN LYMPHOMA. • THE BACKGROUND IS PRIMARY LYMPHOCYTES, AND THE REED-STERNBERG CELLS 79
  • 80. AIIMS NOVEMBER 2015-QUESTIONS & ANSWERS WITH EXPLANATIONS-SOLVED BY DR: MANJUNATH, MBBS, MD (MAMC NEW DELHI) DIRECTOR DOCTORS ACADEMY DAVANAGERE & SHIMOGA (www.doctorsacademydvg.com)- HODKINS LYMPHOMA DR MANJUNATH, DOCTORS ACADEMY, DAVANAGEREDR. MANJUNATH, DOCTORS ACADEMY 80
  • 81. AIIMS NOVEMBER 2015-QUESTIONS & ANSWERS WITH EXPLANATIONS-SOLVED BY DR: MANJUNATH, MBBS, MD (MAMC NEW DELHI) DIRECTOR DOCTORS ACADEMY DAVANAGERE & SHIMOGA (www.doctorsacademydvg.com) 38. IRON METABOLISM AND REGULATION ARE IMPORTANT FOR RBC PRECURSOR CELL. WHICH OF THE FOLLOWING HELPS IN REGULATION OF IRON METABOLISM BUT IS NOT SPECIFIC FOR IRON? • A. HEPCIDIN • B. DMT-1 • C. FERROPORTIN • D. FERRITIN • ANS- B. DMT-1 [REF: WINTROBE 13ED/PG 811] DR MANJUNATH, DOCTORS ACADEMY, DAVANAGEREDR. MANJUNATH, DOCTORS ACADEMY 81
  • 82. AIIMS NOVEMBER 2015-QUESTIONS & ANSWERS WITH EXPLANATIONS-SOLVED BY DR: MANJUNATH, MBBS, MD (MAMC NEW DELHI) DIRECTOR DOCTORS ACADEMY DAVANAGERE & SHIMOGA (www.doctorsacademydvg.com) EXPLANATION: DR MANJUNATH, DOCTORS ACADEMY, DAVANAGEREDR. MANJUNATH, DOCTORS ACADEMY  *HEPHAESTIN IS SIMILAR TO CERULOPLASMIN, THE COPPER-CARRYING PROTEIN.  **DIVALENT METAL TRANSPORTER TYPE 1 (DMT-1): ALSO KNOEN AS NATURAL RE- SISTANCE MACROPHAGE-ASOCIATED PROTEIN TYPE 2 (NRAMP 2) OR DCT-1  HEPCIDIN: PRINCIPAL IRON REGULATORY HORMONE; AND IS NEGATIVELY REGULATED BY FERROPORTIN  DMT1 HAS A STRICT REQUIREMENT FOR DIVALENT CATIONS, AND IT WILL NOR TRANSPORT THE FE3+ FROM OF IRON 82
  • 83. AIIMS NOVEMBER 2015-QUESTIONS & ANSWERS WITH EXPLANATIONS-SOLVED BY DR: MANJUNATH, MBBS, MD (MAMC NEW DELHI) DIRECTOR DOCTORS ACADEMY DAVANAGERE & SHIMOGA (www.doctorsacademydvg.com) 39. THERE ARE DIFFERENT CHECK POINTS IN CELL GROWTH AND REGULATION. WHICH ONE IS THE PRIMARY POINT FOR REGULATION OF CELL GROWTH? A. END OF G1 B. START OF G2 C. END OF S1 D. END OF M ANS: A. END OF G1 REF: ROBBIN’S 9/25; COMPLETE REVIEW OF PATHOLOGY 1/E P197 DR MANJUNATH, DOCTORS ACADEMY, DAVANAGEREDR. MANJUNATH, DOCTORS ACADEMY 83
  • 84. AIIMS NOVEMBER 2015-QUESTIONS & ANSWERS WITH EXPLANATIONS-SOLVED BY DR: MANJUNATH, MBBS, MD (MAMC NEW DELHI) DIRECTOR DOCTORS ACADEMY DAVANAGERE & SHIMOGA (www.doctorsacademydvg.com) EXPLANATION CELL CYCLE Definition: Cell cycle is the sequence of events that results in cell division DR MANJUNATH, DOCTORS ACADEMY, DAVANAGEREDR. MANJUNATH, DOCTORS ACADEMY  PHASES: G0→G1→S→G2→M  G1: PRE-SYNTHETIC GROWTH  S: DNA SYNTHESIS (MOST RADIO-RESISTANT PHASE)  G2: PRE-MITOTIC GROWTH  M: MITOTIC PHASE (MOST RADIO-SENSITIVE PHASE)  G0: QUIESCENT CELLS THAT ARE NOT ACTIVELY CYCLING ARE SAID TO BE IN GO STATE 84
  • 85. AIIMS NOVEMBER 2015-QUESTIONS & ANSWERS WITH EXPLANATIONS-SOLVED BY DR: MANJUNATH, MBBS, MD (MAMC NEW DELHI) DIRECTOR DOCTORS ACADEMY DAVANAGERE & SHIMOGA (www.doctorsacademydvg.com) 40. IDENTIFY THE CRYSTALS IN THE URINE ANALYSIS? A. OXALATE B. URIC ACID C. PHOSPHAHATES D. CYSTEINE ANS: A. OXALATE DR MANJUNATH, DOCTORS ACADEMY, DAVANAGEREDR. MANJUNATH, DOCTORS ACADEMY 85
  • 86. AIIMS NOV 2015-QUESTIONS & ANSWERS WITH EXPLANATIONS-SOLVED BY DR: MANJUNATH, MBBS, MD (MAMC NEW DELHI) DIRECTOR DOCTORS ACADEMY DAVANAGERE & SHIMOGA (www.doctorsacademydvg.com) EXPLANATION Crystals you can get in Urine DR MANJUNATH, DOCTORS ACADEMY, DAVANAGEREDR. MANJUNATH, DOCTORS ACADEMY TRIPLE PHOSPHATE  SEEN IN ALKALINE URINE  RECTANGULAR SHAPE URIC ACID  SEEN IN ACIDIC URINE  BROWN LEMONSHAPED OR STAR SHAPED  BIREFRINGENT WITH POLARIZED LIGHT CALCIUM OXALATE  ENVELOPE SHAPED  SEEN IN ACIDIC URINE 86
  • 87. AIIMS NOV 2015-QUESTIONS & ANSWERS WITH EXPLANATIONS-SOLVED BY DR: MANJUNATH, MBBS, MD (MAMC NEW DELHI) DIRECTOR DOCTORS ACADEMY DAVANAGERE & SHIMOGA (www.doctorsacademydvg.com) 41. CAVITATORY LESION IN RIGHT LOWER LUNG WITH DYSPONEA WITH FOLLOWING HISTOPATHOLOGICAL APPEARANCE? A. ECHINOCOCCUS WITH 2 LAYERS B. STRONGYLOIDES WITH 2 LAYERS C. PARAGONIMUS WITH 2 LAYERS D. CYSTICERCOSIS WITH 3 LAYERS ANS: C. PARAGONIMUS WITH 2 LAYERS REF: HARRISON’S 19/E P1429; CDC WEBSITE DR MANJUNATH, DOCTORS ACADEMY, DAVANAGEREDR. MANJUNATH, DOCTORS ACADEMY 87
  • 88. AIIMS NOV 2015-QUESTIONS & ANSWERS WITH EXPLANATIONS-SOLVED BY DR: MANJUNATH, MBBS, MD (MAMC NEW DELHI) DIRECTOR DOCTORS ACADEMY DAVANAGERE & SHIMOGA (www.doctorsacademydvg.com) EXPLANATION: THE FIGURE SHOWS: EGG OF PARAGONIMUS SP. TAKEN FROM A LUNG BIOPSY STAINED WITH HEMATOXYLIN AND EOSIN DR MANJUNATH, DOCTORS ACADEMY, DAVANAGEREDR. MANJUNATH, DOCTORS ACADEMY GEOGRAPHIC DISTRIBUTION  PARAGONIUM WESTERMANI IS DISTRIBUTED IN SOUTHEAST ASIA AND JAPAN. CLINICAL PRE- SENTATION  ACUTE PHASE: DIARRHEA, ABDOMINAL PAIN, FEVER, COUGH, URTICARIA, HEPATOSPLENOMEGALY & EOSINOPHILIA.  DURING THE CHRONIC PHASE, PULMONARY MANIFESTATIONS INCLUDE COUGH, EXPECTORATION OF DISCOLORED SPUTUM & HEMOPTYSIS MODE OF TRANSMISSION  HUMANS ACQUIRE LUNG FLUKE INFECTION BY INGESTING INFECTIVE METACERCERCARIAE ENCYSTED IN THE MUSCLES AND VISCERA OF CRAYFISH AND FRESHWATER CRABS.  IN ENDEMIC AREAS, THESE CRUSTACEANS ARE CONSUMED RAW, MATINATED, OR PICKLED.  ONCE THE ORGANISM REACH THE DUODENUM, THEY EXCYST, PENETRATE THE GUT WALL, AND TRAVEL THROUGH THE PERITONEAL CAVITY, DIAPHRAGM, AND PLEURAL SPACE TO REACH LUNGS  MATURE FLUKES ARE FOUND IN THE BRONCHIOLES SURROUNDED BY CYSTIC LESIONS  PARASITE EGGS ARE EITHER EXPECTORATED WITH SPUTUM OR SWALLOWED AND PASSED TO THE OUTSIDE ENVIRONMENT WITH FACES  THE LIFE CYCLE IS COMPLETED IN SNAILS AND FRESHWATER CRUSTACEANS. 88
  • 89. AIIMS NOV 2015-QUESTIONS & ANSWERS WITH EXPLANATIONS-SOLVED BY DR: MANJUNATH, MBBS, MD (MAMC NEW DELHI) DIRECTOR DOCTORS ACADEMY DAVANAGERE & SHIMOGA (www.doctorsacademydvg.com) DR MANJUNATH, DOCTORS ACADEMY, DAVANAGEREDR. MANJUNATH, DOCTORS ACADEMY MORPHOLOGY  ADULT LUNG FLUKES, WHICH ARE 7-12 MM IN LENGTH, ARE FOUND ENCAPSULATED IN THE LUNGS OF INFECTED PERSONS. RARELY, FLUKES ARE FOUND ENCYSTED IN CNS (CEREBRAL PARAGONIMIASIS) OR THE ABDOMINAL CAVITY.  WHEN MATIRUG FLUKES LODGE IN LUNG TISSUES, THEY CAUSE HAEMORRHAGE AND NECROSIS, RESULTING IN CYST FORMATION  THE ADJACENT LUNG PARENCHYMA SHOWS EVIDENCE OF INFLAMMATORY INFILTRATION, PREDOMINANTLY BY EOSINOPHILS.  CYSTS USUALLY MEASURE 1-2 CM IN DIAMETER AND MAY CONTAIN ONE OR TWO WORMS EACH  OLDER CYSTS DEVELOP THICKENED WALLS, WHICH MAY UNDER GO CALCIFICATION.  DURING THE ACTIVE PHASE OF PARAGONIMIASIS, LUNG TISSUES SURROUNDING PARASITE CYSTS MAY SHOW EVIDENCE OF PNEUMONIA, BRONCHIECTASIS, AND FIBROSIS. 89
  • 90. AIIMS NOV 2015-QUESTIONS & ANSWERS WITH EXPLANATIONS-SOLVED BY DR: MANJUNATH, MBBS, MD (MAMC NEW DELHI) DIRECTOR DOCTORS ACADEMY DAVANAGERE & SHIMOGA (www.doctorsacademydvg.com) DR MANJUNATH, DOCTORS ACADEMY, DAVANAGEREDR. MANJUNATH, DOCTORS ACADEMY DIAGNOSIS  PRODUCTIVE COUGH WITH BROWNISH SPUTUM OR FRANK HEMOPTYSIS ASSOCIATED WITH PERIPHERAL-BLOOD EOSINOPHILIA IS USUALLY THE PRESENTING FEATURE.  CHEST EXAMINATION MAY SIGNS OF PLEURISY, BRONCHITIS OR BRONCHIECTASIS MAY PREDOMINATE, RARELY LUNG ABSCESS.  IMAGING OF LUNGS SHOWS PATCHY DENSITIES, CAVITIES, PLEURAL EFFUSION, AND RING SHADOWS.  PULMONARY PARAGONIMIASIS IS DIAGNOSED BY DETECTION PARASITE OVA IN SPUTUM, LUNG BIOPSY AND/OR STOOLS.  SEROLOGY IS OF CONSIDERABLE HELP IN EGG-NEGATIVE CASES AND IN CEREBRAL PARAGONIMIASIS.  THE DIFFERENTIAL DIAGNOSIS INCLUDES ACTIVE TUBERCULOSIS. BACTERIAL LUNG ABSCESS, AND LUNG CARCINOMA. TREATMENT THE DRUG OF CHOICE FOR TREATMENT IS PRAZIQUANTEL 90
  • 91. AIIMS NOV 2015-QUESTIONS & ANSWERS WITH EXPLANATIONS-SOLVED BY DR: MANJUNATH, MBBS, MD (MAMC NEW DELHI) DIRECTOR DOCTORS ACADEMY DAVANAGERE & SHIMOGA (www.doctorsacademydvg.com) 42.ABSENT UROBILINOGEN IN URINE WITH ICTERUS INDICATES? A. PERIHEPATIC OBSTRUCTION B. HEMOLYSIS C. HEPATITIS D. LIVER FAILURE ANS: A. PERIHEPATIC OBSTRUCTION [REF: ROBBIN’S 9/830-840] DR MANJUNATH, DOCTORS ACADEMY, DAVANAGEREDR. MANJUNATH, DOCTORS ACADEMY 91
  • 92. AIIMS NOV 2015-QUESTIONS & ANSWERS WITH EXPLANATIONS-SOLVED BY DR: MANJUNATH, MBBS, MD (MAMC NEW DELHI) DIRECTOR DOCTORS ACADEMY DAVANAGERE & SHIMOGA (www.doctorsacademydvg.com) EXPLANATION: UROBILIONGEN APPEARS IN URINE ONLY AFTER ENTROHEPATIC CIRCULATION, SO ITS ABSECENCE WITH ICTERUS INDICATES POST HEPATIC JAUNDICE (PERIHEPATIC OBSTRUCTION) DR MANJUNATH, DOCTORS ACADEMY, DAVANAGEREDR. MANJUNATH, DOCTORS ACADEMY CONDITION SERUM BILIRUBIN URINE UROBILINOGEN URINE BILIRUBIN FECAL UROBILINOGEN NORMAL DIRECT 0.1-0.4 MG/DL INDIRECT 0.2-07 0-4 MG/24H ABSENT 40-280 MG/24H HEMOLYTIC ANEMIA ↑ INDIRECT INCREASED ABSENT INCREASED HEPATITIS ↑ DIRECT AND INDIRET DECREASED IF MICROOBSTRUCTIO N IS PRESENT PRESENT IF MICROOBSTRUCTIO N DECREASE OBSTRUCTION JAUNDICE ↑ DIRECT ABSENT PRESENT TRACE TO ABSENT 92
  • 93. AIIMS NOV 2015-QUESTIONS & ANSWERS WITH EXPLANATIONS-SOLVED BY DR: MANJUNATH, MBBS, MD (MAMC NEW DELHI) DIRECTOR DOCTORS ACADEMY DAVANAGERE & SHIMOGA (www.doctorsacademydvg.com) 43. FOR WHICH PROCEDURE IS THE FOLLOWING INSTRUMENT USED? A. BONE MARROW EXAMINATION B. LIVER BIOPSY C. PLEURAL BIOPSY D. LUMBAR PUNCTURE ANS: A. BONE MARROW EXAMINATION [REF: DACIE PRACTICAL HEMATOLOGY] DR MANJUNATH, DOCTORS ACADEMY, DAVANAGEREDR. MANJUNATH, DOCTORS ACADEMY 93
  • 94. AIIMS NOV 2015-QUESTIONS & ANSWERS WITH EXPLANATIONS-SOLVED BY DR: MANJUNATH, MBBS, MD (MAMC NEW DELHI) DIRECTOR DOCTORS ACADEMY DAVANAGERE & SHIMOGA (www.doctorsacademydvg.com) DR MANJUNATH, DOCTORS ACADEMY, DAVANAGEREDR. MANJUNATH, DOCTORS ACADEMY 94
  • 95. AIIMS NOV 2015-QUESTIONS & ANSWERS WITH EXPLANATIONS-SOLVED BY DR: MANJUNATH, MBBS, MD (MAMC NEW DELHI) DIRECTOR DOCTORS ACADEMY DAVANAGERE & SHIMOGA (www.doctorsacademydvg.com) DR MANJUNATH, DOCTORS ACADEMY, DAVANAGEREDR. MANJUNATH, DOCTORS ACADEMY 95
  • 96. AIIMS NOV 2015-QUESTIONS & ANSWERS WITH EXPLANATIONS-SOLVED BY DR: MANJUNATH, MBBS, MD (MAMC NEW DELHI) DIRECTOR DOCTORS ACADEMY DAVANAGERE & SHIMOGA (www.doctorsacademydvg.com) 44. IN A PATIENT SUFFERING WITH SICKLE CELL ANEMIA, ELETROPHORETIC MOBILITY OF HBS IN RELATION TO HBA WILL BE? A. RETARDED B. ACCELERATED C. SAME D. WILL DEPEND UPON CONCENTRETATION OF HBS ANS: A. RETARDED [REF: COMPLETE REVIEW OF PATHOLOGY 1/343] DR MANJUNATH, DOCTORS ACADEMY, DAVANAGEREDR. MANJUNATH, DOCTORS ACADEMY 96
  • 97. AIIMS NOV 2015-QUESTIONS & ANSWERS WITH EXPLANATIONS-SOLVED BY DR: MANJUNATH, MBBS, MD (MAMC NEW DELHI) DIRECTOR DOCTORS ACADEMY DAVANAGERE & SHIMOGA (www.doctorsacademydvg.com) 45. WHICH OF THE FOLLOWING MECHANISM IS MAINLY INVOLVED IN EPIGENETICS? A. METHYLATION B. ACETYLATION C. DEAMINATION D. PHOSPHORYLATION ANS: A. METHYLATION (The term epigenetics refers to heritable changes in gene expression (active versus inactive genes) that does not involve changes to the underlying DNA sequence; a change in phenotype without a change in genotype.) [REF: ROBBIN’S 9/ EP5] DR MANJUNATH, DOCTORS ACADEMY, DAVANAGEREDR. MANJUNATH, DOCTORS ACADEMY 97
  • 98. AIIMS NOV 2015-QUESTIONS & ANSWERS WITH EXPLANATIONS-SOLVED BY DR: MANJUNATH, MBBS, MD (MAMC NEW DELHI) DIRECTOR DOCTORS ACADEMY DAVANAGERE & SHIMOGA (www.doctorsacademydvg.com) EXPLANATION DR MANJUNATH, DOCTORS ACADEMY, DAVANAGEREDR. MANJUNATH, DOCTORS ACADEMY EPIGENETICS DEFINITION HERITABLE CHANGES IN GENE EXPRESSION, NOT CAUSED BY ALTERATIONS IN DNA SEQUENCE. EPIGENETIC FACTORS HISTONES AND HISTONE MODIFYING FACTORS o HISTONE METHYLATION o HISTONE ACETLATION o HISTONE PHOSPHORYLATION 98
  • 99. AIIMS NOV 2015-QUESTIONS & ANSWERS WITH EXPLANATIONS-SOLVED BY DR: MANJUNATH, MBBS, MD (MAMC NEW DELHI) DIRECTOR DOCTORS ACADEMY DAVANAGERE & SHIMOGA (www.doctorsacademydvg.com) DR MANJUNATH, DOCTORS ACADEMY, DAVANAGEREDR. MANJUNATH, DOCTORS ACADEMY SIGNIFICANCE  EPIGENETIC DYSREGULATION→CENTRAL ROLE IN MALIGNANCY  MANY OTHER DIEASES ARE ASSOCIATED WITH INHERITED OR ACQUIRED EPIGENETIC ALTERATIONS. EG GENOMIC IMPRINTING IN QUIRED EPIGENETIC ALTERATION. EG GENOMIC IMPRINTING IN PRADER WILLI SYNDROME  EPIGENETIC ALTERATIONS LIKE HISTONE ACETYLATION AND DNA METHYLATION ARE REVERSIBLE AND ARE RESPONSIVE TO DRUGS; SO, HDAC INHIBITORS AND DNA METHYLATION INHIBITORS ARE BEING TESTED IN THE TREATMENT OF CANCER DIAGNOSIS  SEQUENCING  CHIP ON CHIP (MICROARRAY TECHNOLOGY)  USING METHYLATION SPECIFIC PRIMERS IN POLYMERASE CHAIN REACTION (PCR)  BISULPHITE METHOD: BISULPHITE CONVERTS UNMETHYLATED CYTOSINE TO URACIL, WHICH ACTS LIKE THYMINE IN DOWNSTREAM REACTIONS. THE UNMETHYLATED (MODIFIED) DNA IS DETECTED BY SEQUENCE ANALYSIS. 99
  • 100. AIIMS NOVEMBER 2015-QUESTIONS & ANSWERS WITH EXPLANATIONS-SOLVED BY DR: MANJUNATH, MBBS, MD (MAMC NEW DELHI) DIRECTOR DOCTORS ACADEMY DAVANAGERE & SHIMOGA (www.doctorsacademydvg.com) PHARMACOLOGY 47. NEW ORAL DRUG APPROVED FOR THE TREATMENT OF HEPATITIS C IS? A. RIBAVIRINE B. LEDIPASVIR C. LAMIVUDINE D. PEG INTERFERON ANS: B. LEDIPASVIR (Ledipasvir (formerly GS-5885) is a drug for the treatment of hepatitis C that was developed by Gilead Sciences) REF: HARRISON’S 19/E P2041-2047 DR MANJUNATH, DOCTORS ACADEMY, DAVANAGEREDR. MANJUNATH, DOCTORS ACADEMY 100
  • 101. AIIMS NOV 2015-QUESTIONS & ANSWERS WITH EXPLANATIONS-SOLVED BY DR: MANJUNATH, MBBS, MD (MAMC NEW DELHI) DIRECTOR DOCTORS ACADEMY DAVANAGERE & SHIMOGA (www.doctorsacademydvg.com) EXPLANATION: DISCUSSING ANOUT THE PTIONS ONE BY ONE: DR MANJUNATH, DOCTORS ACADEMY, DAVANAGEREDR. MANJUNATH, DOCTORS ACADEMY (A) RIBAVIRINE  IT IS A NUCLEOSIDE INHIBITOR  USED FOR SEVERE RSV, CHRONIC HEPATITIS C INFECTION  RIBAVIRIN SHOULD NERVE BE GIVEM AS MONOTHERAPY.  SHOULD BE GIVEN IN COMBINATION WITH PEGINTERFERON ALFA-2A;  DAILT DOSE IS 800-1200 MG ADMINISTERED ORALLY  MOST IMPORTANT SIDE EFFECT IS HEMOLYSIS  CONTRA-INDICATED IN PREGNANCY & HEMOGLOBINOPATHIES (B) LEDIPASVIR  LEDIPASVIR (FORMERLY GS-5885) IS A RECENTLY DEVELOPED DRUG FOR THE TREATMENT OF EPATITIS C  IT IS A POTENT INHIBITOR OF HCV NS5A, A VIRAL PHOSPOHOPROTEIN THAT PLAYS AN IMPORTANT ROLE IN VIRAL REPLICATION, ASSE,BLY, AND SECRETION.  SOFTOSBUVIR IS A NUCLEOTIDE ANALOG INHIBITOR OF HEPATITIS C VIRUS NS5B POLYMERASE  FIXED DOSE COMBINATION LADIPASVIR-SOFOSBUVIR IS FDA-APPROVED FOR THE TREATMENT OF CHRONIC HEPATITIS GENOTYPE 1  MOST COMMON ADVERSE EFFECTS ARE FATIGUE AND HEADACHE. 101
  • 102. AIIMS NOV 2015-QUESTIONS & ANSWERS WITH EXPLANATIONS-SOLVED BY DR: MANJUNATH, MBBS, MD (MAMC NEW DELHI) DIRECTOR DOCTORS ACADEMY DAVANAGERE & SHIMOGA (www.doctorsacademydvg.com): EXTRA EDGE NEW MCQS DR MANJUNATH, DOCTORS ACADEMY, DAVANAGEREDR. MANJUNATH, DOCTORS ACADEMY © LAMIVUDINE  LAMIVUDINE, IS A NUCLEOSIDE ANALOG THAT INHIBITS HIV REVERSE TRANSCRIPTASE AND HBC DNA POLYMERASE.  FOLLOWING ORALADMINISTRATION. LAMIVUDINE IS ABSORBED RAPIDLY WITH A BIOAVAILABILITY OF 80%  LAMIVUDINE IS A WELL TOLERATED DRUG  AMINOTRANFERASE RISES AFTER THERAPY MAY BE SEEN (D) PEG INTERFERON  INTERFERONS (IFNS) ARE POTENT CYROKINES THAT POSSESS ANTIVIRAL, IMMUNIMODULATING, AND ANTIPROLIFERATIVE ACTIVITIES  ORAL ADMINISTRATION DOES NOT RESULT IN DETECTABLE IFN LEVEL IN SERUM, HENCE GIVEN SUBCUTANEOUS OR INTRAMUSCULAR  ATTACHMENT OF IFN PROTEINS TO LARGE, POLYETHYLENE GLYCOL (PEG) MOLECULES SLOWS ABSORPTION, DECREASES CLEARANCE, AND PROVIDES HIGHER & MORE PROLONGED SERUM CONCENTRATIONS THAT ENABLE NCE-WEEKLY DOSING  SIDE EFFECTS:ACUTE INFLUENZA B, DISCOMFORT AT THE INJECTION SITE, AND LEUCOPENIA  ONCE-A-WEEK ADMINISTERED LONG-ACTING PEG IFN IS MORE EFFECTIVE THAN THE STANDARD IFN 102
  • 103. AIIMS NOV 2015-QUESTIONS & ANSWERS WITH EXPLANATIONS-SOLVED BY DR: MANJUNATH, MBBS, MD (MAMC NEW DELHI) DIRECTOR DOCTORS ACADEMY DAVANAGERE & SHIMOGA (www.doctorsacademydvg.com) 48. MECHANISM OF ACTION OF ASPIRIN FOR ANTIPLATELET ACTION IS? A. BLOCKS TXA2 SYNTHESIS B. COX 2 INHIBITOR C. LIPOXYGENASE INHIBITOR D. PLATELET DYSFUNCTION ANS: A. BLOCKS TXA2 SYNTHESIS REF: H19 746-747 DR MANJUNATH, DOCTORS ACADEMY, DAVANAGEREDR. MANJUNATH, DOCTORS ACADEMY 103
  • 104. AIIMS NOV 2015-QUESTIONS & ANSWERS WITH EXPLANATIONS-SOLVED BY DR: MANJUNATH, MBBS, MD (MAMC NEW DELHI) DIRECTOR DOCTORS ACADEMY DAVANAGERE & SHIMOGA (www.doctorsacademydvg.com) EXPLANATION: ASPIRIN DR MANJUNATH, DOCTORS ACADEMY, DAVANAGEREDR. MANJUNATH, DOCTORS ACADEMY IMPORTANT INFO IT IS THE MOST WIDELY USED ANTIPLATELET AGENT WORLDWIDE MECHANISM OF ACTION  ASPIRIN PRODUCES ITS ANTITHROMBOTIC EFFECT BY IRREVERSIBLY ACETYLATING & INHIBITING PLARELET CYCLOOXY-GENASE (COX)-1, A CRITICAL ENZYME IN THROMBOXANE A2 SYNTHESIS  AT HIGH DOSES, ASPIRIN ALSO INHIBITS COX-2, AN INDUCIBLE COX ISOFORM FOUND IN ENDOTHELIAL CELLS AND INFLAMMATORY CELLS. INDICATIONS  SECONDARY PREVENTION OF CARDIOVASCULAR EVENTS IN PATIENTS WITH CORONARY ARTERY, CEREBROVASCULAR OR PERIPHERAL VASCULAR DISEASE  ASPIRIN PRODUCES A 25% REDUCTION IN THE RISK OF CARDIOVASCULAR DEATH, MI, R STROKE  ASPIRIN IS ALSO USED FOR PRIMARY PREVENTION IN HIGH RISK PATIENTS 104
  • 105. AIIMS NOV 2015-QUESTIONS & ANSWERS WITH EXPLANATIONS-SOLVED BY DR: MANJUNATH, MBBS, MD (MAMC NEW DELHI) DIRECTOR DOCTORS ACADEMY DAVANAGERE & SHIMOGA (www.doctorsacademydvg.com) DR MANJUNATH, DOCTORS ACADEMY, DAVANAGEREDR. MANJUNATH, DOCTORS ACADEMY DOSAGE 75-325 MG ONCE DAILY ADVERSE EFFECTS SIDE EFFECTS OF ASPIRIN ARE DOSE-RELATED MOST COMMON ARE GASTROINTESTINAL (DYSPEPSIA, EROSIVE GASTRITIS, PEPIC ULCERS WITH BLLEDING AND PERFORATION) 105
  • 106. AIIMS NOV 2015-QUESTIONS & ANSWERS WITH EXPLANATIONS-SOLVED BY DR: MANJUNATH, MBBS, MD (MAMC NEW DELHI) DIRECTOR DOCTORS ACADEMY DAVANAGERE & SHIMOGA (www.doctorsacademydvg.com) 49. WHICH OF THE FOLLOWING IS THE LATEST APPROVED DRUG FOR TUBERCULOSIS? A. ETHIONAMIDE B. LEVOFLOXACIN C. BEDAQUILINE D. TIPRANAVIR • ANS: C. BEDAQUILINE • (Bedaquiline (trade name Sirturo, code names TMC207 and R207910) is a medication used to treat tuberculosis) DR MANJUNATH, DOCTORS ACADEMY, DAVANAGEREDR. MANJUNATH, DOCTORS ACADEMY 106
  • 107. AIIMS NOV 2015-QUESTIONS & ANSWERS WITH EXPLANATIONS-SOLVED BY DR: MANJUNATH, MBBS, MD (MAMC NEW DELHI) DIRECTOR DOCTORS ACADEMY DAVANAGERE & SHIMOGA (www.doctorsacademydvg.com): EXTRA EDGE EXPLANATION: SECOND-LINE DRUGS (HAVE LOWER EFFICACY & HIGHER TOXICITY, USED IN MDR & XDR TB): DR MANJUNATH, DOCTORS ACADEMY, DAVANAGEREDR. MANJUNATH, DOCTORS ACADEMY FLUOROQUINOLONES (LEVOFLOXACIN & MOXIFLAXACIN MOST EFFECTIVE) INHIBIT MYCOBACTERIAL DNA GYRASE & TOPOISOMERASE IV, PREVENTING CELL REPLICATION AND PROTEIN SYNTHESIS, AND ARE BACTERICIDAL. INJECTABLE AMINOGLYCONSIDES (KANAMYCIN, AMIKACIN & STREPTOMYCIN) EXERT MYCOBACTERICIDAL ACTIVITY BY BINDING TO THE 16S RIBOSOMAL SUBUNIT. ADVERSE EFFECTS OTOTXICITY, NEPHROTOXICITY,A AND NEUROTOXICITY. CAPREOMYCIN USEFUL WHEN ADDITIONAL RESISTANCE TO AMINOGLYCOSIDES IS DOCUMENTED. CAPREOMYCIN IS ADMINISTERED BY THE IM ROUTE; ADVERSE EFFECTS; HYPOKALEMIA, HYPOMAGNESEMIA, OTO-AND RENAL TOXICITY. ETHIONAMIDE AND PROTHIONAMIDE (BACTERIOSTATIC) INHIBITS INHA GENE PRODUCT ENOYL-ACYL CARRIER PROTEIN (ACP) REDUCTASE, INVOLVED IN MYCOLIC ACID SYNTHESIS. ADVERSE EFFECTS: SEVERE GI REACTIONS (INCLUDING ABDOMINAL PAIN, NAUSEA, VOMITING), NEUROLOGIC SIDE EFFECTS, REVERSIBLE HEPATITIS, HYPERSENSITIVITY REACTIONS, AND HYPOTHYROIDISM. 107
  • 108. AIIMS NOV 2015-QUESTIONS & ANSWERS WITH EXPLANATIONS-SOLVED BY DR: MANJUNATH, MBBS, MD (MAMC NEW DELHI) DIRECTOR DOCTORS ACADEMY DAVANAGERE & SHIMOGA (www.doctorsacademydvg.com) DR MANJUNATH, DOCTORS ACADEMY, DAVANAGEREDR. MANJUNATH, DOCTORS ACADEMY CYCLOSERINE AND TERIZIDONE (THERIZIDONE) PREVENTS CELL WALL SYNTHESIS & INHIBITS THE ACTION OF ALANINE RACEMASE, INVOLVED IN PRODUCTION OF PEPTIDOGLYCANS IT IS WELL ABSORBED AFTER ORAL ADMINISTRATION AND IS WELL DISTRIBUTED AFTER ORAL ADMINISTRATION AND IS WELL DISTRIBUTED THROUGHOUT BODY FLUID, INCL CSF SIDE EFFECTS INCLUDE SEIZURES, PSYCHOSIS, PERIPHERAL NEUROPATHY, HEADACHE, SOMNOLENCE & ALLERGIC REACTIONS. PAS (PARA-AMINOSALICYLIC ACID) IT IS AN ORAL AGENT USED IN TREATMENT OF MDR-& XDR-TB. ITS BACTERIOSTATIC ACTIVITY IS DUE TO INHIBITION OF FOLATE SYNTHESIS AND OF IRON UPTAKE. ADVERSE EFFECTS INCLUDE NAUSEA, VOMITING, DIARRHEA, HEMOLYSIS IN C6PD DEFICIENCY. 108
  • 109. AIIMS NOV 2015-QUESTIONS & ANSWERS WITH EXPLANATIONS-SOLVED BY DR: MANJUNATH, MBBS, MD (MAMC NEW DELHI) DIRECTOR DOCTORS ACADEMY DAVANAGERE & SHIMOGA (WWW.DOCTORSACADEMYDVG.COM) 50. DRUG USED FOR DETOXIFICATION IN CHRONIC ALCOHOLISM ARE ALL EXCEPT? A. NALTREXONE B. FLUMAZENIL C. DISULFIRAM D. ACAMPROSATE ANS: B. FLUMAZENIL REF:HARRISON 19/E P2727 DR MANJUNATH, DOCTORS ACADEMY, DAVANAGEREDR. MANJUNATH, DOCTORS ACADEMY 109
  • 110. AIIMS NOV 2015-QUESTIONS & ANSWERS WITH EXPLANATIONS-SOLVED BY DR: MANJUNATH, MBBS, MD (MAMC NEW DELHI) DIRECTOR DOCTORS ACADEMY DAVANAGERE & SHIMOGA (www.doctorsacademydvg.com) EXPLANATION: MEDICATIONS FOR REHABILITATION IN CHRONIC ALCOHOLISM DR MANJUNATH, DOCTORS ACADEMY, DAVANAGEREDR. MANJUNATH, DOCTORS ACADEMY NALTREXONE  AN OPIOID ANTAGONIST  MAY BE TAKEN DAILY ORALLY, OR AS MONTHLY INJECTION  SHORTENS SUB-SEQUENT RELAPSES  BY BLOCKING OPIOID RECEPTORS, NALTREXONE DECREASES ACTIVITY IN THE DOPAMINE-RICH VENTRAL TEGMENTAL REWARD SYSTEM & DECREASES THE FEELING OF PLEASURE IF ALCOHOL IS IMBIBED. ACAMPROSATE  IT INHIBITS NMDA RECEPTORS, DECREASING MILD SYMPTOMS OF PROTRACTED WITHDRAWAL.  DOSE~2 G.D DIVIDED INTO THREE ORAL DOSES DISULFIRAM  IT IS AN ALDH INHIBITOR, USED AT DOSES OF 250 MG/D  IT PRODUCES VOMITING & AUTONOMIC INSTABILITY IN THE PRESENCE OF ALCOHOL AS A RESULT OF RAPIDLY RISING BLOOD LEVELS OF ACETALDEHYDE.  THIS REACTION CAN BE DANGEROUS, ESPECIALLY FOR ATIENTS WITH HEART DISEASE, STROKE, DIABETES MELLITUS, OR HYPERTENSION  IT ALSO CARRIES POTENTIAL RISK OF DEPRESSION, PSYCHOTIC SYMPTOMS, PERIPHERAL NEUROPAHY, AND LIVER DAMAGE 110
  • 111. AIIMS NOV 2015-QUESTIONS & ANSWERS WITH EXPLANATIONS-SOLVED BY DR: MANJUNATH, MBBS, MD (MAMC NEW DELHI) DIRECTOR DOCTORS ACADEMY DAVANAGERE & SHIMOGA (www.doctorsacademydvg.com) DR MANJUNATH, DOCTORS ACADEMY, DAVANAGEREDR. MANJUNATH, DOCTORS ACADEMY OTHER DRUGS UNDER INVESTIGATION  NALMEFENE (OPIOID ANTAGONIST)  VARENICLINE (NICOTINIC RECEPTOR AGONIST)  ONDANSETRON (SEROTONIN ANTAGONIST),  PRAZOSIN (Α-ADRENERGIC AGONIST)  BACLOFEN (GABA B RECEPTOR AGONIST),  TOPIRAMATE (ANTICONVULSANT) 111
  • 112. AIIMS NOV 2015-QUESTIONS & ANSWERS WITH EXPLANATIONS-SOLVED BY DR: MANJUNATH, MBBS, MD (MAMC NEW DELHI) DIRECTOR DOCTORS ACADEMY DAVANAGERE & SHIMOGA (WWW.DOCTORSACADEMYDVG.COM) 51. ETANERCEPT IS? A. COX 2 INHIBITOR B. TNF INHIBITOR C. IL 6 INHIBITOR D. IFN GAMMA INHIBITOR ANS: B. TNF INHIBITOR REF: HARRISON 19/E P272E30 DR MANJUNATH, DOCTORS ACADEMY, DAVANAGEREDR. MANJUNATH, DOCTORS ACADEMY 112
  • 113. AIIMS NOV 2015-QUESTIONS & ANSWERS WITH EXPLANATIONS-SOLVED BY DR: MANJUNATH, MBBS, MD (MAMC NEW DELHI) DIRECTOR DOCTORS ACADEMY DAVANAGERE & SHIMOGA (www.doctorsacademydvg.com) EXPLANATION INDICATIONS OF ETANERCEPT:- DR MANJUNATH, DOCTORS ACADEMY, DAVANAGEREDR. MANJUNATH, DOCTORS ACADEMY • ANKYLOSING SPONDYLITIS • ADULT RHEUMATOID ARTHRITIS • PSORIATIC ARTHRITIS • PLAQUE PSORIASIS 113
  • 114. AIIMS NOV 2015-QUESTIONS & ANSWERS WITH EXPLANATIONS-SOLVED BY DR: MANJUNATH, MBBS, MD (MAMC NEW DELHI) DIRECTOR DOCTORS ACADEMY DAVANAGERE & SHIMOGA (www.doctorsacademydvg.com) 52. WHICH OF THE FOLLOWING ADVICE WILL YOU GIVE TO A PATIENT WHEN HE IS GIVEN BISPHOSPHONATES? A. TAKE TABLET BEFORE FOOD WITH FULL GLASS OF WATER B. DISCONTINUE IF GASTRITIS PERSISTS C. TAKE TABLET AFTER FOOD WITH FULL GLASS OF WATER D. WITHDRAW THE DRUG WHEN BONE PAIN OCCURS ANS: A. TAKE TABLET BEFORE FOOD WITH FULL GLASS OF WATER REF: HARRISON 19/E P2499 DR MANJUNATH, DOCTORS ACADEMY, DAVANAGEREDR. MANJUNATH, DOCTORS ACADEMY 114
  • 115. AIIMS NOV 2015-QUESTIONS & ANSWERS WITH EXPLANATIONS-SOLVED BY DR: MANJUNATH, MBBS, MD (MAMC NEW DELHI) DIRECTOR DOCTORS ACADEMY DAVANAGERE & SHIMOGA (www.doctorsacademydvg.com) EXPLANATION: BISPHOSPHONATES DR MANJUNATH, DOCTORS ACADEMY, DAVANAGEREDR. MANJUNATH, DOCTORS ACADEMY NAME OF DRUGS ALENDRONATE, RISEDRONATE, IBANDRONATE, AND ZOLEDRONIC ACID INDICATIONS  PREVENTION AND TREATMENT OF POST- MENOPAUSAL OSTEOPOROSIS.  TREATMENT OF STEROID-INDUCED OSTEOPOROSIS,  PREVENTION OF STEROID-INDUCED OSTEOPOROSIS  PAGET’S DISEASE  HYPERCALCEMIA OF MALIGNANCY MECHANISM OF ACTION  INHIBIT OSTEOCLAST MEDIATED BONE RESORPTION  INCREASES APOPTOSIS OF OSTEOCLASTS  INHIBIT IL6, THEREBY SUPPRESSES DIFFERENTIATION OF OSTEOCLAST PRECURSOR TO MATURE OSTEOCLAST 115
  • 116. AIIMS NOV 2015-QUESTIONS & ANSWERS WITH EXPLANATIONS-SOLVED BY DR: MANJUNATH, MBBS, MD (MAMC NEW DELHI) DIRECTOR DOCTORS ACADEMY DAVANAGERE & SHIMOGA (www.doctorsacademydvg.com) DR MANJUNATH, DOCTORS ACADEMY, DAVANAGEREDR. MANJUNATH, DOCTORS ACADEMY PROCESS OF ADMINISTRATION  ALENDRONATE SHOULD BE GIVEN WITH A FULL GLASS OF WATER BEFORE BREAKFAST, BECAUSE BISPHOSPHONATES ARE POORLY ABSORBED.  IT IS RECOMMENDED THAT PATIENTS REMAIN UPRIGHT FOR AT LEAST 30 MIN AFTER TAKING THE MEDICATION TO AVOID ESOPHAGEAL IRRITATION. CONTRAINDICATION  BECAUSE OF THE POTENTIAL FOR ESOPHAGEAL IRRITATION, ALENDRONATE IS CONTRAINDICATED IN PATIENTS WITH HAVE STRICTURE OR INADEQUATE EMPTYING OF THE ESOPHAGUS. 116
  • 117. AIIMS NOV 2015-QUESTIONS & ANSWERS WITH EXPLANATIONS-SOLVED BY DR: MANJUNATH, MBBS, MD (MAMC NEW DELHI) DIRECTOR DOCTORS ACADEMY DAVANAGERE & SHIMOGA (www.doctorsacademydvg.com) DR MANJUNATH, DOCTORS ACADEMY, DAVANAGEREDR. MANJUNATH, DOCTORS ACADEMY 117
  • 118. AIIMS NOV 2015-QUESTIONS & ANSWERS WITH EXPLANATIONS-SOLVED BY DR: MANJUNATH, MBBS, MD (MAMC NEW DELHI) DIRECTOR DOCTORS ACADEMY DAVANAGERE & SHIMOGA (www.doctorsacademydvg.com) 53. DRUG OF CHOICE IN PREGNANCY INDUCED HYPERTENSION? A. ACE INHIBITOR B. ATENOLOL C. ALPHA METHYL DOPA D. NITROPRUSSIDE ANS: C. ALPHA METHYL DOPA REF:HYPERTENSION IN PREGNANCY (REPORT OF THE ACOG TASK FORCE ON HYPERTENSION IN PREGNANCY) OBSTETRICS & GYNECOLOGY VOL. 122, NO. 5 NOVEMBER 2013 DR MANJUNATH, DOCTORS ACADEMY, DAVANAGEREDR. MANJUNATH, DOCTORS ACADEMY 118
  • 119. AIIMS NOV 2015-QUESTIONS & ANSWERS WITH EXPLANATIONS-SOLVED BY DR: MANJUNATH, MBBS, MD (MAMC NEW DELHI) DIRECTOR DOCTORS ACADEMY DAVANAGERE & SHIMOGA (WWW.DOCTORSACADEMYDVG.COM) 55. WHICH OF THE FOLLOWING IS UREIDOPENICILLIN? A. PENICILLIN V B. METHICILLIN C. PIPERCILLIN D. CLAVULANIC ACID ANS: C. PIPERCILLIN DR MANJUNATH, DOCTORS ACADEMY, DAVANAGEREDR. MANJUNATH, DOCTORS ACADEMY 119
  • 120. AIIMS NOV 2015-QUESTIONS & ANSWERS WITH EXPLANATIONS-SOLVED BY DR: MANJUNATH, MBBS, MD (MAMC NEW DELHI) DIRECTOR DOCTORS ACADEMY DAVANAGERE & SHIMOGA (WWW.DOCTORSACADEMYDVG.COM) 56. WHILE PILOCARPINE ACTS ON SPHINCTER PAPILLAE. WHICH OF THE FOLLOWING HAS ANALOGOUS EFFECT ON DILATOR PUPILLAE? A. EPINEPHRINE B. TIMOLOL C. COCAINE D. LATANOPROST ANS: A. EPINEPHRINE (REF: K.D. TRIPATHI) DR MANJUNATH, DOCTORS ACADEMY, DAVANAGEREDR. MANJUNATH, DOCTORS ACADEMY 120
  • 121. AIIMS NOV 2015-QUESTIONS & ANSWERS WITH EXPLANATIONS-SOLVED BY DR: MANJUNATH, MBBS, MD (MAMC NEW DELHI) DIRECTOR DOCTORS ACADEMY DAVANAGERE & SHIMOGA (www.doctorsacademydvg.com) EXPLANATION: PUPILLARY MYDRIASIS DR MANJUNATH, DOCTORS ACADEMY, DAVANAGEREDR. MANJUNATH, DOCTORS ACADEMY THIS IS CAUSED BY DRUGS THAT ACTIVELY DILATE THE PUPIL (ACTING ON THE DILATOR PAPILLA) ON PRESENT THE ACTIVE CONSTRICTION OF THE PPIL (PREVENTING ACTION ON THE SPHINCTER PAPILLA). THUS, DRUGS WHICH DILATE THE PUIL BY ACTIVELY ACTING ON THE SYMPATHETIC INNERVATIONS OF THE DILATOR PAPILLA ARE CALLED SYMPATHOMIMETIC DRUGS AND DRUGS WHICH RESULT IN DILATATION OF THE PUPIL BY PREVENTING THE PARASYMPATHETIC ACTION OF THE SPHINCTER ARE CALLED PARASYMPATHOLYRIC DRUGS. 121
  • 122. AIIMS NOV 2015-QUESTIONS & ANSWERS WITH EXPLANATIONS-SOLVED BY DR: MANJUNATH, MBBS, MD (MAMC NEW DELHI) DIRECTOR DOCTORS ACADEMY DAVANAGERE & SHIMOGA (WWW.DOCTORSACADEMYDVG.COM) 57. PHASE 1 TRIAL IS DONE FOR? A. HUMAN PHARMACOLOGY & SAFETY B. THERAPEUTIC EXPLORATORY C. MICRODOSING D. PHARMACOKINETICS ANS: A (REF: K. D TRIPATHI 7/E P63-64) DR MANJUNATH, DOCTORS ACADEMY, DAVANAGEREDR. MANJUNATH, DOCTORS ACADEMY 122
  • 123. AIIMS NOV 2015-QUESTIONS & ANSWERS WITH EXPLANATIONS-SOLVED BY DR: MANJUNATH, MBBS, MD (MAMC NEW DELHI) DIRECTOR DOCTORS ACADEMY DAVANAGERE & SHIMOGA (www.doctorsacademydvg.com) EXPLANATION DR MANJUNATH, DOCTORS ACADEMY, DAVANAGEREDR. MANJUNATH, DOCTORS ACADEMY PHASE NAME CONDUCRED ON PURPOSE I HUMAN PHARMACOLOGY AND SAFETY HEALTH VOLUNTEERS (20-100)  SAFETY AND TOLERABILITY  TO KNOW MAIMUM TOLERABLE DOSES (MTD) II THERAPEUTIC EXPLORATORY 100-150 PATIENTS  TO ESTABLISH THERAPEUTIC EFFICACY  DOSE RANGING AND CEILING EFFECT III THERAPEUTIC CINFIRMATORY UPTO 5000 PATIENTS FROM SEVERAL CENTRE  TO CONFIRM THERAPEUTIC EFFICACY IV POST MARKETING SURVEILI LUNCH TREATED BY PRACTICING PHYSICIANS  TO KNOW RARE AND LONG TERM ADVERSE EFFECTS  SPECIAL GROUPS LIKE CHILDREN, PREGNANCY ETC CAN BE TESTED 0 (ZERO) MICRODOSING STUDIES HEALTHY VOLUNTEERS (SMALL NUMBER)  VERY LOW DOSE 1/100 OF HUMAN DOES; MAX 100 µG 123
  • 124. AIIMS NOV 2015-QUESTIONS & ANSWERS WITH EXPLANATIONS-SOLVED BY DR: MANJUNATH, MBBS, MD (MAMC NEW DELHI) DIRECTOR DOCTORS ACADEMY DAVANAGERE & SHIMOGA (WWW.DOCTORSACADEMYDVG.COM) 58. WHICH ANTIDOTE IS REQUIRED FOR FIBRINOLYTIC THERAPY? A. tPA B. PROTAMINE SULFATE C. STREPTOKINASE D. E-AMINO CAPROIC ACID ANS: D. E-AMINO CAPROIC ACID REF: K. D TRIPATHI 6/E P608-610 DR MANJUNATH, DOCTORS ACADEMY, DAVANAGEREDR. MANJUNATH, DOCTORS ACADEMY 124
  • 125. AIIMS NOV 2015-QUESTIONS & ANSWERS WITH EXPLANATIONS-SOLVED BY DR: MANJUNATH, MBBS, MD (MAMC NEW DELHI) DIRECTOR DOCTORS ACADEMY DAVANAGERE & SHIMOGA (www.doctorsacademydvg.com) EXPLANATION DISCUSSING THE OPTIONS ONE BY ONE, DR MANJUNATH, DOCTORS ACADEMY, DAVANAGEREDR. MANJUNATH, DOCTORS ACADEMY (A) EACA EPSILON AMINO CAPROIC ACID (EACA) & TRANEXAMIC ACID ARE SPECIFIC ANTIDOTES OF FIBRINOLYTIC THERAPY (B) PROTAMINE SULFATE IT IS AN ANTIDOTE OF HEPARIN (C ) STREPTOKINASE IT FORMS A COMPLEX WITH PLASMINOGEN & EXPOSES ITS ACTIVE SITE. THIS ALTERED PLASMINOGEN ACTS LIKE TPA & ACTIVATES OTHER PLASMINOGEN MOLECULES TO PLASMIN THUS, STREPTOKINASE ACTS AS A FIBRINOLYTIC (D) TPA TISSUE PLASMINOGEN ACTIVATOR SELECTIVELY ACTIVATES PLASMINOGEN THAT IS BOUND TO FIBRIN (IN THE THROMBUS) 125
  • 126. AIIMS NOV 2015-QUESTIONS & ANSWERS WITH EXPLANATIONS-SOLVED BY DR: MANJUNATH, MBBS, MD (MAMC NEW DELHI) DIRECTOR DOCTORS ACADEMY DAVANAGERE & SHIMOGA (WWW.DOCTORSACADEMYDVG.COM) 59. WHICH ANTICHOLINERGIC IS EXCLUSIVELY USED IN PRE- ANAESTHETIC MEDICATION? A. ATROPINE B. PROMETHAZINE C. GLYCOPYRROLATE D. SCOPOLAMINE ANS: C. GLYCOPYRROLATE (TO DECREASE SECRETIONS) REF: K. D TRIPATHI 7/E P110-112 DR MANJUNATH, DOCTORS ACADEMY, DAVANAGEREDR. MANJUNATH, DOCTORS ACADEMY 126
  • 127. AIIMS NOV 2015-QUESTIONS & ANSWERS WITH EXPLANATIONS-SOLVED BY DR: MANJUNATH, MBBS, MD (MAMC NEW DELHI) DIRECTOR DOCTORS ACADEMY DAVANAGERE & SHIMOGA (WWW.DOCTORSACADEMYDVG.COM) 60. DRUGS SOLD ONLY ON PRESCRIPTION OF REGISTERED MEDICAL PRACTITIONER BE LONGS TO WHICH SCHEDULE? A) H B) X C) J D) P ANS: A) H (REF: TEXTBOOK OF FORENSIC MEDICINE AND TOXICOLOGY BY N.G.RAO P356) DR MANJUNATH, DOCTORS ACADEMY, DAVANAGEREDR. MANJUNATH, DOCTORS ACADEMY 127
  • 128. AIIMS NOV 2015-QUESTIONS & ANSWERS WITH EXPLANATIONS-SOLVED BY DR: MANJUNATH, MBBS, MD (MAMC NEW DELHI) DIRECTOR DOCTORS ACADEMY DAVANAGERE & SHIMOGA (www.doctorsacademydvg.com) EXPLANATION THE DRUGS AND COSMETIC RULES 1945 DR MANJUNATH, DOCTORS ACADEMY, DAVANAGEREDR. MANJUNATH, DOCTORS ACADEMY SCHEDULE DESCRIPTION C BIOLOGICAL AND SPECIAL PRODUCTS E LIST OF POISONS F VACCINES AND SERA G HORMONE PREPARATION; H DRUGS (POISON TO BE SOLD ONLY ON THE PRESCRIPTION OF A REGISTERED MEDICAL PRACTITIONER); J LIST OF DISEASES FOR THE CURE OF WHICH NO DRUG SHOULD BE ADVERTISED L ANTIBIOTICS, ANTIHISTAMINICS AND OTHER RECENT CHEMOTHERAPEUTIC AGENTS 128
  • 129. AIIMS NOV 2015-QUESTIONS & ANSWERS WITH EXPLANATIONS-SOLVED BY DR: MANJUNATH, MBBS, MD (MAMC NEW DELHI) DIRECTOR DOCTORS ACADEMY DAVANAGERE & SHIMOGA (WWW.DOCTORSACADEMYDVG.COM) 61. A PATIENT WITH EPILEPTIC DISORDER PRESENTED WITH CONCENTRIC VISUAL FIELD DEFECTS. WHICH DRUG WAS HE TAKING ? A. VALPROATE B. ETHOSUXIMIDE C. PHENOBARBITOL D. VIGABATRIN ANS: D. VIGABATRIN (REF: NELSON 20/E P2846) DR MANJUNATH, DOCTORS ACADEMY, DAVANAGEREDR. MANJUNATH, DOCTORS ACADEMY 129
  • 130. AIIMS NOV 2015-QUESTIONS & ANSWERS WITH EXPLANATIONS-SOLVED BY DR: MANJUNATH, MBBS, MD (MAMC NEW DELHI) DIRECTOR DOCTORS ACADEMY DAVANAGERE & SHIMOGA (www.doctorsacademydvg.com) EXPLANATION DR MANJUNATH, DOCTORS ACADEMY, DAVANAGEREDR. MANJUNATH, DOCTORS ACADEMY ANTIEPILETIC DRUG SIDE EFFECTS ACETAZOLAMIDE MINOR: DIZZINESS, POLYURIA, ELECTROLYTE IMBALANCE SERIOUS: STEVENTS- JOHNSON SYNDRINE BENZODIAZEPINES MINOR: DOES-RELATED NEUROTOXICITY (DROWSINESS, SEDATION, ATAXIA), HYPERACTIVITY, DROOLING INCREASED SECRETIONS SERIOUS APNEA CARBAMAZEPINE NUISANCE: TICS, TRANSIENT LEUCOPENIA, HYPONATREMIA, WEIGHT GAIN, NAUSEA, DIZZINESS SERIOUS: STEVENS-JOHNSON SYNDROME, AGRANULOCYTSIS, APLASTIC ANEMIA, LIVER TOXICITY GABAPENTIN IN CHILDREN: ACUTE ONSET OF AGGRESSION, HYPERACTIVITY IN ADULTS: EUPHORIA AND BEHAVIOURAL DISINHIITION, WEIGHT GAIN LAMOTRINGINE MINOR: HEADACHE, ATAXIA, DIZZINESS, TREMOR, SERIOUS, STEVENS-JOHNSON SYNDROME, RARELY LIVE TOXICITY LEVETIRACETAM CNS ADVERSE EVENTS: SOMNOLENCE ASTHENIA DIZZINESS, BUTUSUALLY LESS THAN OTHER AEDS IN ADULTS: BEHAVIOURAL SYMPTOMS ARE COMMON, DEPRESSION 130
  • 131. AIIMS NOV 2015-QUESTIONS & ANSWERS WITH EXPLANATIONS-SOLVED BY DR: MANJUNATH, MBBS, MD (MAMC NEW DELHI) DIRECTOR DOCTORS ACADEMY DAVANAGERE & SHIMOGA (www.doctorsacademydvg.com) DR MANJUNATH, DOCTORS ACADEMY, DAVANAGEREDR. MANJUNATH, DOCTORS ACADEMY PHENOBARBITONE MINOR: NEUROTOXICITY INSOMNIA, HYPERACTIVITY, SIGNS OF DISTRACTIBILITY, FLUCTUATION OF MOOD, AGGRESSIVE OUTBURSTS SERIOS:LIVER TOXICITY, STEVENS-JOHNSON SYNDROME PHENYTOIN AND OTHER HYDANTOINS MINOR: GINGIVAL HYPERPLASIA, COARSENING OF FACIES, HIRSUTISM, CEREBELLOVESTIBULAR SYMPTOMS (NYSTAGMUS AND ATAXIA) SERIOUS: STEVENS-JOHNSON SYNDROME LIVE TOXICITY PREGABALIN MINOR: DIZZINESS, PERIPHERAL EDEMA, BLURED VISION WEIGHT GAIN, THROMBOCYTOPENIA SERIOUS: HYPERSENSITIVITY REACTIONS, RHABDOMYOLYSIS TOPIRAMATE MINOR: CONGNITIVE DYSFUNCTION, WEIGHT LOSS, RENAL CALCULI, HYPOHIDROSIS, FEVER SERIOUS: PRECIPITATION OF GLAUCOMA VALPROCIC ACID MINOR: WEIGHT GAIN; HYPERAMMONEMIA TREMOR, ALOPECIA, MENSTRUAL IRREGULARITIES SERIOUS: HEPATIC AND PANCREATIC TOXICITY VIGABATRIN MINOR: HYPERACTIVITY SERIOUS: IRREVERSIBLE VISUAL FIELD DEFICTS, RETINOPATHY THAT REQUIRES FREQUENT OPHTHALMOGIC EVALUATIONS AND FOLLOW UP 131
  • 132. AIIMS NOV 2015-QUESTIONS & ANSWERS WITH EXPLANATIONS-SOLVED BY DR: MANJUNATH, MBBS, MD (MAMC NEW DELHI) DIRECTOR DOCTORS ACADEMY DAVANAGERE & SHIMOGA (WWW.DOCTORSACADEMYDVG.COM) 62. DRUG WHOSE CONCENTRATION INCREASES AFTER A FATTY MEAL ARE ALL EXCEPT? A. AMPHOTERICIN B B. AMPICILLIN C. PPI D. NIMESULIDE ANS: C. PPI (REF: NUTRITION IN PEDIATRICS /E P196) DR MANJUNATH, DOCTORS ACADEMY, DAVANAGEREDR. MANJUNATH, DOCTORS ACADEMY 132
  • 133. AIIMS NOV 2015-QUESTIONS & ANSWERS WITH EXPLANATIONS-SOLVED BY DR: MANJUNATH, MBBS, MD (MAMC NEW DELHI) DIRECTOR DOCTORS ACADEMY DAVANAGERE & SHIMOGA (www.doctorsacademydvg.com) EXPLANATION DR MANJUNATH, DOCTORS ACADEMY, DAVANAGEREDR. MANJUNATH, DOCTORS ACADEMY DRUG ABRORPTION REDUCED/DELAYED BY FOOD DRUG ABSORPTION INCREASED BY FOOD AMPICILLIN ASPIRIN ATENOLOL AZITHROMYCIN CAPTOPRIL CEFACLOR CEFIXIME CEPHALEXIN CIPROFLOXACIN DIDANOSINE ATOVAQUONE CARBAMAZEPINE CHLOROTHIAZIDE CEFUROXIME CLOFAZIMINE DIAZEPAM ERYTHROMYCIN ESTOLATE GANCICLOVIR HYDROCHLOROTHIAZIDE ITRACONAZOLE 133
  • 134. AIIMS NOV 2015-QUESTIONS & ANSWERS WITH EXPLANATIONS-SOLVED BY DR: MANJUNATH, MBBS, MD (MAMC NEW DELHI) DIRECTOR DOCTORS ACADEMY DAVANAGERE & SHIMOGA (www.doctorsacademydvg.com) DR MANJUNATH, DOCTORS ACADEMY, DAVANAGEREDR. MANJUNATH, DOCTORS ACADEMY INDINARIR ISONIAZID LORATIDINE NAFICILLIN PENICILLIN G OR V PHENOBARBITAL PHENYTOIN RIFAMPIN SUCRALFATE TERACYCLINE DOXYCYCLINE KETOCONAZOLE LITHIUM LOVASTATIN METHYLPHENIDATE METOPROLOL NELFINAVIR NITROFURANTOIN PROPRANOLOL PROPOXYPHENE RITONAVIR SAQUINAVIR SPIRONOLACTONE HYDRALAZINE 134
  • 135. AIIMS NOV 2015-QUESTIONS & ANSWERS WITH EXPLANATIONS-SOLVED BY DR: MANJUNATH, MBBS, MD (MAMC NEW DELHI) DIRECTOR DOCTORS ACADEMY DAVANAGERE & SHIMOGA (WWW.DOCTORSACADEMYDVG.COM) 63. ALL OF THE FOLLOWING INHIBITS NUCLEIC ACID SYNTHESIS EXCEPT? A. QUINOLONES B. NALIDIXIC ACID C. LINEZOLID D. RIFAMPICIN ANS: C. LINEZOLID (LINEZOLID INHIBTS THE PROTEIN SYNTHESIS) (REF: HARRIOSN 19/E P931) DR MANJUNATH, DOCTORS ACADEMY, DAVANAGEREDR. MANJUNATH, DOCTORS ACADEMY 135
  • 136. AIIMS NOV 2015-QUESTIONS & ANSWERS WITH EXPLANATIONS-SOLVED BY DR: MANJUNATH, MBBS, MD (MAMC NEW DELHI) DIRECTOR DOCTORS ACADEMY DAVANAGERE & SHIMOGA (www.doctorsacademydvg.com) EXPLANATION DR MANJUNATH, DOCTORS ACADEMY, DAVANAGEREDR. MANJUNATH, DOCTORS ACADEMY ANTIBACTERIAL AGENT MAJOR CELLULAR TARGET MECHANISM OF ACTION Β-LACTAMS (PENICILINS CEPHALOSPORINS) CELL WALL INHIBIT CELL-WALL CROSS-LINKING VANCOMYCIN CELL WALL INTERTERES WITH ADDITION OF NEW CELL WALL SUBUNITS (MURAMY PENTAPEPTIDES) BACITRACIN CELL WALL PREVENTS ADDITION OF CELL-WALL- SUBUNITS BY INHIBITING RECYCLING OF MEMBRANE LIPID CARRIER MACROLIDES (ERYTHROMYCIN) PROTEIN SYNTHESIS BIND TO 50S RIBOSOMAL SUBUNIT LINCOSAMIDES (CLINDAMYCIN) PROTEIN SYNTHESIS BIND TO 50S RIBOSOMAL SUBUNIT BLOCK PEPTIDE CHAIN ELONGTATION CHLORAMPHENICOL PROTEIN SYNTHESIS BIND TO 50S RIBOSOMAL SUBUNIT BLANK AMINOACYL TRNA ATTAXHMENT 136
  • 137. AIIMS NOV 2015-QUESTIONS & ANSWERS WITH EXPLANATIONS-SOLVED BY DR: MANJUNATH, MBBS, MD (MAMC NEW DELHI) DIRECTOR DOCTORS ACADEMY DAVANAGERE & SHIMOGA (www.doctorsacademydvg.com) DR MANJUNATH, DOCTORS ACADEMY, DAVANAGEREDR. MANJUNATH, DOCTORS ACADEMY TETRACYCLINE PROTEIN SYNTHESIS BIND TO 50S RIBOSOMAL SUBUNIT BLOCK AMINOACYL TRNA AMINOGLYCOSIDES (GENTAMYCIN) PROTEIN SYNTHESIS BIND TO 50S RIBOSOMAL SUBUNIT INHIBIT TRANSLOCATION OF PEPTIDYL- TRNA LINEZOLID PROTEIN SYNTHESIS BIND TO 50S RIBOSOMAL SUBUNIT INHIBITS INITIATION OF PROTEIN SYNTHESIS SULFONAMIDES AND TRIMETHOPRIM CELL METABOLISM COMPETITIVELY INHIBIT ENZYMES INVOLVED IN TWO STEPS OF FOLIC ACID BIOSYNTHESIS RIFAMPICIN NUCLEIC ACID SYNTHESIS INHIBITS DNA-DEPENDENT-RNA POLYMERASE METRONIDAZOLE NUCLEIC ACID SYNTHESIS INTRACELLULARLY GENERATES SHORT- LIVED RE-ACTIVE INTERMEDIATES THAT DAMAGE DNA BY ELECTRON TRANSFER SYSTEM QUINOLONESS (CIPROFLOXACIN) DNA SYNTHESIS INHIBIT ACTIVITY OF DNA GYRASE (A SUBUNIT) AND TOPOISOMERASE IV 137
  • 138. AIIMS NOV 2015-QUESTIONS & ANSWERS WITH EXPLANATIONS-SOLVED BY DR: MANJUNATH, MBBS, MD (MAMC NEW DELHI) DIRECTOR DOCTORS ACADEMY DAVANAGERE & SHIMOGA (www.doctorsacademydvg.com) DR MANJUNATH, DOCTORS ACADEMY, DAVANAGEREDR. MANJUNATH, DOCTORS ACADEMY NOVOBIOCIN DNA SYNTHESIS INHIBIT ACTIVITY OF DNA GYRASE ( B SUBUNIT) POLYMYXINS CELL MEMBRANE DISRUPT MEMBRANE PERMEABILITY BY CHARGE ALTERATION GRAMICIDIN CELL MEMBRANE FORM PORES CHLORAMPHENICOL PROTEIN SYNTHESIS BIND 50S RIBOSOMAL SUBUNIT BLOCK AMINOACYL TRNA POSITIONING OXAZOLIDINOMES (LINEZOLID TEDIZOLID) PROTEIN SYNTHESIS BIND 50S RIBOSOMAL SUBUNIT INHIBIT IN;TIATIO OF PEPTIDE SYNTHESIS MUPIROCIN PROTEIN SYNTHESIS BLOCK ISOLEUCYL TRNA SYNTHEASE SULFONAMIODES (SULFADIAZINE SULFLSOXAZOLE AND SULFAMETHOXAZOLE) FOLATE SYNTHESIS INHIBITS DIHYDROFOLATE SYNTHETASE TRIMETHOPRIN FOLATE SYNTHESIS INHIBITS DIHYDROFOLATE REDUCTASE QUINOLONERS (NORFLOXACIN, CIPROFLOXACIN, OFLOXACIN LEVOFLOXACIN MOXIFLOXACIN GEMIFLOXACIN) DNA SYNTHESIS INHIBIT DNA GYRASE AND DNA TOPOISOMERASE IV ENZYME-DNA- DRUG COMPLEX:BLOCK REPLICATION APPARATUS 138
  • 139. AIIMS NOV 2015-QUESTIONS & ANSWERS WITH EXPLANATIONS-SOLVED BY DR: MANJUNATH, MBBS, MD (MAMC NEW DELHI) DIRECTOR DOCTORS ACADEMY DAVANAGERE & SHIMOGA (www.doctorsacademydvg.com) DR MANJUNATH, DOCTORS ACADEMY, DAVANAGEREDR. MANJUNATH, DOCTORS ACADEMY RIFAMYCINS (RIFAMPIN FIFABUTIN, RIFAPENTNE) RNA SYNTHESIS INHIBIT RNA POLYMERASE NITROFURANTOIN NUCLEIC ACID SYNTHESIS REDUCE REACTIVE DRUG DERIVATIVES THAT DAMAGE DNA METRONIDAZOLE NUCLEIC ACID SYNTHESIS BIND LPS AND MEMBRANE CHANNEL AND MEMBRANE LEAKAGE POLYMYXIN E (CLISTIN) CELL MEMBRANE BIND LPS AND DISRUPT: BOTH OUTER AND SYTOPLASMIC MEMBRANE DAPTOMYCIN CELL MEMBRANE PRODUCES MEMBRANES CHANNEL AND MEMBRANE LEAKAGE 139
  • 140. AIIMS NOV 2015-QUESTIONS & ANSWERS WITH EXPLANATIONS-SOLVED BY DR: MANJUNATH, MBBS, MD (MAMC NEW DELHI) DIRECTOR DOCTORS ACADEMY DAVANAGERE & SHIMOGA (www.doctorsacademydvg.com) DR MANJUNATH, DOCTORS ACADEMY, DAVANAGEREDR. MANJUNATH, DOCTORS ACADEMY 140
  • 141. AIIMS NOV 2015-QUESTIONS & ANSWERS WITH EXPLANATIONS-SOLVED BY DR: MANJUNATH, MBBS, MD (MAMC NEW DELHI) DIRECTOR DOCTORS ACADEMY DAVANAGERE & SHIMOGA (WWW.DOCTORSACADEMYDVG.COM) 64. GENDER SPECIFIC SIDE EFFECT OF VALPROATE IS? A. WEIGHT GAIN B. TREMORS C. PCOD D. ALOPECIA ANS: C. PCOD (REF: K.D TRIPATHY 7/E P405-409) DR. MANJUNATH, DOCTORS ACADEMY 141
  • 142. AIIMS NOV 2015-QUESTIONS & ANSWERS WITH EXPLANATIONS-SOLVED BY DR: MANJUNATH, MBBS, MD (MAMC NEW DELHI) DIRECTOR DOCTORS ACADEMY DAVANAGERE & SHIMOGA (WWW.DOCTORSACADEMYDVG.COM) 65. GLUCOCORTICOID IS NOT USED IN WHICH OF THE FOLLOWING CONDITIONS? A. MULTIPLE MYELOMA B. KAPOSI SARCOMA C. HODGKIN LYMPHOMA D. CLL ANS: B. KAPOSI SARCOMA (REF: HARRISON’S 19/E P716) DR MANJUNATH, DOCTORS ACADEMY, DAVANAGEREDR. MANJUNATH, DOCTORS ACADEMY 142
  • 143. AIIMS NOV 2015-QUESTIONS & ANSWERS WITH EXPLANATIONS-SOLVED BY DR: MANJUNATH, MBBS, MD (MAMC NEW DELHI) DIRECTOR DOCTORS ACADEMY DAVANAGERE & SHIMOGA (www.doctorsacademydvg.com) EXPLANATION DISCUSSING THE DRUGS USED IN THE TREATMENT OF ABOVE MALIGNANCIES ONE NY ONE DR MANJUNATH, DOCTORS ACADEMY, DAVANAGEREDR. MANJUNATH, DOCTORS ACADEMY (A) MULTIPLE MYELOMA THALIDOMIDE, DEXAMETHASONE, LENALIDOMIDE (A DERIVATIVE OF THALIDOMIDE), BORTEZOMIB (A PROTEASOME INHIBITOR), CYCLOPHOSPHAMIDE (B) KAPOSI SARCOMA LIPOSOMAL DAUNORUBICIN, LIPOSOMAL DOXORUNICIN, VINBLASTINE, AND PACLITAXEL-HAVE BEEN APPROVED BY FDA FOR THIS INDICATION (C) HODGKIN LYMPHOMA COMBINATION OF DOXOTUBICIN (ADRIAMYCIN), BLEOMYCIN VINBLASTINE, AND DACARBAZINE (ABVD) (D) CLL THE MOST COMMON TREATMENTS FOR PATIENTS WITH TYPICAL B- CELL CLL/SMALL LYMPHOCYTIC LYMPHOMA HAVE BEEN CHLORAMBUCIL OR FLUDARABINE, ALONE OR IN COMBINATION. OTHER DRUGS USED: BENDAMUSTINE, CVP (CYCLOPHOSPHAMIDE, VINCRISTINE, AND PREFNISONE) OR CHOP PLUS RITUXIMAB. ALEMTUZUMAB (ANTI-CD52) 143
  • 144. AIIMS NOV 2015-QUESTIONS & ANSWERS WITH EXPLANATIONS-SOLVED BY DR: MANJUNATH, MBBS, MD (MAMC NEW DELHI) DIRECTOR DOCTORS ACADEMY DAVANAGERE & SHIMOGA (WWW.DOCTORSACADEMYDVG.COM) 66. WHICH OF THE FOLLOWING IS A CALCINEURIN INHIBITOR? A. CYCLOSPORINE B. METHOTREXATE C. LEFLUNOMIDE D. MYCOPHENOLATE MOFETIL ANS: A. CYCLOSPORINE REF: K.D TRIPATHY 7/E P838-839 DR MANJUNATH, DOCTORS ACADEMY, DAVANAGEREDR. MANJUNATH, DOCTORS ACADEMY 144
  • 145. AIIMS NOV 2015-QUESTIONS & ANSWERS WITH EXPLANATIONS-SOLVED BY DR: MANJUNATH, MBBS, MD (MAMC NEW DELHI) DIRECTOR DOCTORS ACADEMY DAVANAGERE & SHIMOGA (www.doctorsacademydvg.com) EXPLANATION: DR MANJUNATH, DOCTORS ACADEMY, DAVANAGEREDR. MANJUNATH, DOCTORS ACADEMY BIND TO IMMUNIPHILLIN ↓ INHIBITS ACTIVATION OF NUCLEAR FACTOR OF ACTIVATED T CELLS (NFAT) ↓ INHIBITION OF TRANSCRIPTION FACTOR OF IL-2 ↓ IMMUNISUPPRESSIVE ACTION 145
  • 146. AIIMS NOV 2015-QUESTIONS & ANSWERS WITH EXPLANATIONS-SOLVED BY DR: MANJUNATH, MBBS, MD (MAMC NEW DELHI) DIRECTOR DOCTORS ACADEMY DAVANAGERE & SHIMOGA (WWW.DOCTORSACADEMYDVG.COM) 67. DRUG OF CHOICE FOR BIRD FLU? A. OSELTAMIVIR B. RIBAVIRIN C. ENTECAVIR D. ACYCLOVIR ANS: A. OSELTAMIVIR REF: HARRISON’S 19/E P1209-1213 DR MANJUNATH, DOCTORS ACADEMY, DAVANAGEREDR. MANJUNATH, DOCTORS ACADEMY 146
  • 147. AIIMS NOV 2015-QUESTIONS & ANSWERS WITH EXPLANATIONS-SOLVED BY DR: MANJUNATH, MBBS, MD (MAMC NEW DELHI) DIRECTOR DOCTORS ACADEMY DAVANAGERE & SHIMOGA (WWW.DOCTORSACADEMYDVG.COM) 68. AT A DOSE OF 3- 5MCG/KG/MIN, DOPAMINE CAUSES? A. β1 RECEPTOR STIMULATION B. INCREASE IN RENAL FLOW C. INCREASE BLOOD PRESSURE D. VASOCONTRICTION ANS: B. INCREASE IN RENAL FLOW (REF:GOODMAN & GILMAN PHARMACOLOGY/CH 10) DR MANJUNATH, DOCTORS ACADEMY, DAVANAGEREDR. MANJUNATH, DOCTORS ACADEMY 147
  • 148. AIIMS NOV 2015-QUESTIONS & ANSWERS WITH EXPLANATIONS-SOLVED BY DR: MANJUNATH, MBBS, MD (MAMC NEW DELHI) DIRECTOR DOCTORS ACADEMY DAVANAGERE & SHIMOGA (WWW.DOCTORSACADEMYDVG.COM) 69. URIPRISTAL ACETATE IS A ? A. SELECTIVE ESTROGEN RECEPTOR MODULATOR B. SELECTIVE PROGESTERONE RECEPTOR MODULATOR C. GNRH ANATAGONIST D. GNRH AGONIST ANS: B. SELETIVE PROGESTERONE RECEPTOR MODULATOR (SPRM) (REF: HARRISON 19/E P2391) DR MANJUNATH, DOCTORS ACADEMY, DAVANAGEREDR. MANJUNATH, DOCTORS ACADEMY 148
  • 149. AIIMS NOV 2015-QUESTIONS & ANSWERS WITH EXPLANATIONS-SOLVED BY DR: MANJUNATH, MBBS, MD (MAMC NEW DELHI) DIRECTOR DOCTORS ACADEMY DAVANAGERE & SHIMOGA (WWW.DOCTORSACADEMYDVG.COM) MICROBIOLOGY 70. MOST COST EFFECTIVE WAY TO PREVENT INFECTION? A. HANDWASHING B. ANTIBIOTIC C. CHEMOPROPHYLAXIS D. CULTURE SENSITIVITY ANS: A. HANDWASHING (REF: HARRISON’S 19/E P152E9) DR MANJUNATH, DOCTORS ACADEMY, DAVANAGEREDR. MANJUNATH, DOCTORS ACADEMY 149
  • 150. AIIMS NOV 2015-QUESTIONS & ANSWERS WITH EXPLANATIONS-SOLVED BY DR: MANJUNATH, MBBS, MD (MAMC NEW DELHI) DIRECTOR DOCTORS ACADEMY DAVANAGERE & SHIMOGA (WWW.DOCTORSACADEMYDVG.COM) 71. IDENTIFY THE ORGANISM IN THE GIVEN FIGURE? A. HISTOPLASMOSIS B. CANDIDA C. CRYPTOCOCCUS D. COCCIDIODOMYCOSIS ANS: C. CRYPTOCOCCUS (REF: ANANTNARAYAN P611) DR MANJUNATH, DOCTORS ACADEMY, DAVANAGEREDR. MANJUNATH, DOCTORS ACADEMY 150
  • 151. AIIMS NOV 2015-QUESTIONS & ANSWERS WITH EXPLANATIONS-SOLVED BY DR: MANJUNATH, MBBS, MD (MAMC NEW DELHI) DIRECTOR DOCTORS ACADEMY DAVANAGERE & SHIMOGA (www.doctorsacademydvg.com) EXPLANATION: DIAGNOSIS OF CRYPTOCOCCUS INFECTION DR MANJUNATH, DOCTORS ACADEMY, DAVANAGEREDR. MANJUNATH, DOCTORS ACADEMY INDIA INK STAINING  A DIAGNOSIS OF CRYPTOCOCCOSIS REQUIRES THE DEMONSTRATION OF YEAST CELLS IN NORMALLY STERILE TISSUES.  VISUALIZATION OF THE CAPSULE OF FUNGAL CELLS IN CEREBROSPINAL FLUID (CSF) MIXED WITH INDIA INK IS A USEFUL RAPID DIAGNOSTIC TECHNIQUE.  CRYPTOCOCCAL CELLS IN INDIA INK HAVE A DISTINCTIVE APPEARANCE BECAUSE THEIR CAPSULES EXCLUDE INK PARTICLES.  HOWEVER, THE CSF INDIA INK EXAMINATION MAY YIELD NEGATIVE RESULTS IN PATIENTS WITH A LOW FUNGAL BURDEN.  THIS EXAMINATION SHOULD BE PERFORMED BY A TRAINED INDIVIDUAL, SINCE LEUKOCYTES & FAT GLOBULES CAN SOMETIMES BE MISTAKEN FOR FUNGAL CELLS. 151
  • 152. AIIMS NOV 2015-QUESTIONS & ANSWERS WITH EXPLANATIONS-SOLVED BY DR: MANJUNATH, MBBS, MD (MAMC NEW DELHI) DIRECTOR DOCTORS ACADEMY DAVANAGERE & SHIMOGA (www.doctorsacademydvg.com): CRYPTOCOCCUS DR MANJUNATH, DOCTORS ACADEMY, DAVANAGEREDR. MANJUNATH, DOCTORS ACADEMY 152
  • 153. AIIMS NOV 2015-QUESTIONS & ANSWERS WITH EXPLANATIONS-SOLVED BY DR: MANJUNATH, MBBS, MD (MAMC NEW DELHI) DIRECTOR DOCTORS ACADEMY DAVANAGERE & SHIMOGA (WWW.DOCTORSACADEMYDVG.COM) 72. STOOL SAMPLE FROM A PATIENT SHOWS THE FOLLOWING. HOW DO HUMANS GET THIS INFECTION? A. INGESTION WITH FOOD B. WORK IN DIRTY WATER C. IMPROPERLY COOKED BEEF D. DIRECT CONTACT ANS: A. INGESTION WITH FOOD (ENTAMOEBA CYST) (REF: K. D CHATTERJEE COLOUR PLATES) DR MANJUNATH, DOCTORS ACADEMY, DAVANAGEREDR. MANJUNATH, DOCTORS ACADEMY 153
  • 154. AIIMS NOV 2015-QUESTIONS & ANSWERS WITH EXPLANATIONS-SOLVED BY DR: MANJUNATH, MBBS, MD (MAMC NEW DELHI) DIRECTOR DOCTORS ACADEMY DAVANAGERE & SHIMOGA (www.doctorsacademydvg.com) EXPLANATION: DR MANJUNATH, DOCTORS ACADEMY, DAVANAGEREDR. MANJUNATH, DOCTORS ACADEMY CHARACTERISTIC FEATURES DIAGNOSIS  10-15 UM SIZED  ROUND YELLOW COLORED STRUCTURE SEEN  BROWN COLOURED 1-4 NUCLEI AND GLYCOGEN MASS CYST OF ENTAMEOBA  PEAR SHAPED YELLOW COLOURED SEEN WITH VISIBLE AXOSTYLE TROPHOZOITE OF GIARDIA  6-10 UM SIZED DISTINCT CYST WALL SURROUND WITH VISIBLE AXOSTYLE CYST OF GIARDIA 154
  • 155. AIIMS NOV 2015-QUESTIONS & ANSWERS WITH EXPLANATIONS-SOLVED BY DR: MANJUNATH, MBBS, MD (MAMC NEW DELHI) DIRECTOR DOCTORS ACADEMY DAVANAGERE & SHIMOGA (www.doctorsacademydvg.com) DR MANJUNATH, DOCTORS ACADEMY, DAVANAGEREDR. MANJUNATH, DOCTORS ACADEMY  40X60UM SIZED OVAL SHAPED YELLOW COLORED STRUCTURE SEEN WITH LIGHTLY STAINED BLASTOMERE  SHELL MEMBRANE AND SEGMENTED OVUM WITH LIGHT YELLOW STAINED BLASTOMERE  CKEAR SHELL MEMBRANE AND BLASTOMERE HOOKWORM EGG  60X40 SIZED ROUND OVAL SHAPED YELLOW COLOURED STRUCTURE SEEN WITH YELLOW STAINED OUTER CORTICAL THICK CELL WALL  UNSEGMENTED OVUM AND ALSO THE SPACE BETWEEN THE SHELL AND OVUM AT EACH POLE ARE STAINED YELLOW. FERTILIZED EGG OF ASCARIS LUMBRICOIDS  25X50 UM SIZED YELLOW COLORED,  BARREL SHAPED STRUCTURE SEEN WITH LIGHTLY STAINED MUCUS PLUGS AT EACH POLE  EGG SHELL IS STAINED BROWN AND ENCLOSES THE LIGHT YELLOW STAINED UNSEGMENTED OVUM EGGS OF TRICHURIS TRICHURA 155